Sunteți pe pagina 1din 115

Anurag Mishra Electricity and Magnetism with www.puucho.

com

ELECTRIC CURRENT current in a conductor is given by the direction of motion of


The electric current characterizes the flow of charge positive charge carriers. For example, the current in Fig. 2.1
through a material Fig. 2.1 (a) shows a section of (a) is to the right.
conducting wire with positive charge carriers moving to the Consider the effect of the sign of the charge carriers on
right. Let dQ be the magnitude of the charge that passes the sense of the current. Positive carriers moving to the right
through the plane cross-sectional surface labeled S in time tend to cause the; region to the-right to become more positive
dt. . and the region to' the left to become more negative. In Fig.
The electric current I in the wire is the rate at which 2.1 (b), negative carriers moving to the left also tend to
charge passes through this surface: cause the region to ihe right to become more positive and
I~ dQ the region to the left to become more negative. That is, the
dt carrier motion shown in both Fig. 2.1 (a) and (b) gives the
same result. Thus the sense of the current is the same in Fig.

d+--:___. +=.(s) +__.+~


+____..., + ___..
V 2.1 (a) as it is in Fig. 2.1 (b) in each case it is to the right.
This means that we need not be concerned with

v.-
E

(a)
Sense of/
the sign of the carriers when dealing with the
external effects of a current; these effects are the
same for carriers having either sing.

----- ------ ~
+
Concepts: 1. Conductors are materials in which
.J +
+
(s) electric charges readily flow. In many metals, these electrons
are free to move when an electric field is applied to the
E materiaL Under static conditions the electric field in the
Sense of/ ,interior of a conductor is zero, even if the conductor carries a
(b) net charge. Otherwise the free electrons would be accelerated,
Fig. 2.1 which would violate the assumption of a static charge·
distribution. 1
The SI unit of electric current is the ampere (A), equal
2. In an insulator, on the other hand, the electrons are
to one coulomb per second: lA =lC/s.
,bound rather tightly to the atoms and are not free to move,
Electric current I is a scalar quantity. Even though the
under the electric fields. An insulator can carry any
electric current is not a vector quantit:Y, it is common
'distribution of electric charges on its surface or in its interior,
practice to speak of the "direction" of the current. This
·and the electric field in the interior of an insulator can have
direction corresponds to the direction of flow of positive
charge carriers. To emphasize that current is a scalar, we '.non-:z~ro val~es.
shall refer to the sense of the current. The sense of the

www.puucho.com
Anurag Mishra Electricity and Magnetism with www.puucho.com

ELECTRICITY & MAGNETISM '

3. In an insulating material, molecules are not easily


ionized, the electrical properties may depend on the electric
dipole moment of the molecules. Materials in which the tir - . - ~
molecules have permanent dipole moments are called polar,
and electric fields can align the dipole moments of
molecules. In some materials, the alignment of the dipoles
remains even when we remove the applied field; these
-- -ta it ! Device for
circulating
charges

materials are called ferroelectric. /


Even in non-polar materials the applied electric field can
induce a dipole moment in the molecules. In an insulator, a
sufficiently large electric field can ionize the atoms, and as a
r rr rrr
result there are electrons available to move through the
material. Under these circumstances an insulator can behave ' V7
more like a conductor. This situation is called breakdown
,and requires fields typically in the range of 10 6 V/m in air .
4. --> .
--> Fig. 2.3. The electric field E0 moves electrons
Ea through the slab of copper. The electrons
collected at the top of the slab are transported
through an external path to the bottom of the
copper slab by a device.

~-
I. f~i -f- fl
+ + + + + + + + ~
ELECTRIC CURRENT DENSITY
Let the density of charge carriers be n (number of charge
.'
V
\7 ' V7
carriers per unit volume of the material). Let q be the charge
on each charge carrier. We imagine the current carrying wire
'' Electrons in the material move (b) Electrons accumulate on the top
upward in response to the field. surface and positive ions on to be cylindrically shaped. We consider a reference location,
(a) the bottom4 Induced charges set as shown in Fig. 2.4. All the charges within v d dt to the left
up a field E'. of the reference location will pass the location during the
-->
Ea time 'interval dt. The charge conservation implies that net
charge entering and leaving any cross-section must be the
same. The number of charge carriers within this volume is :
,. Conventional
., \--- ~o --- ) current Electron flow
Inside the slab, the
·,
.,',
,.
f+ + + + + + + + +) net field is zero.

I I =---J
+ 1 o.L. I
Cross-sectional area A
,I
(c)
I'
,' Fig. 2.2
In Fig. 2.2 (a) electrons move from the bottom of the slab'
of copper to· the top under the action of the applied electric
A B
field, until the concentration of electrons at the top of the slab:
,(and of positive ions at the bottom) creates afield that cancels:
,the applied field in the interior of the copper and prevents the: ~ - - - - - - i + V0 - f - - - - - - - - - - '
flow of additional electrons. Suppose there were a mechanism;
to remove electrons from the top of the slab, cany them l Fig.2.4
'around an external path, and re-inject them at the bottom of, No. of charge carriers per unit volume x volume
lthe slab (shown schematically in Fig. 2.3). In this case, there: = nAvddt
iwould be no build-up of charge on the top and bottom of the:, If each charge carrier has charge. q, the charge dQ within
,slab. this volume is
· The continuous loop of flowing electrons is a simple dQ = V ddt anA
representation of an electric circuit, and the flow of electrons;
!(or other !'ha~ged_particles) is called an electric cu_rrent. ' As current is defined as I = _9_, ·so we have
dt
I= qnAvd

www.puucho.com
Anurag Mishra Electricity and Magnetism with www.puucho.com

ELECTRIC CURRENT
If the charge _, _, _,
flowing through an J = n.q. v aa+ nbqb v db
area varies with where the subscripts. a and b designate the quantities for
position across that each type of charge carrier.
area as shown in Fig. AreaA If the drift velocity of the carriers varies from point to
2.5, the upper part has
point within a material, as shown in Fig. 2.7, then the
more charges crossing
current density varies correspondingly. In this case the
per unit area than the
current I through a surface can be found from the surface
lower part. Analogous _,
Fig. 2.5 integral of the current density J:
to the idea of electric _, _,
flux, the electric field I=JJ.dS
crossing a unit area, we define the current I as the flux of
current density through an area. ·
r =J area
1 ds
Regardless of the orientation of the cross-sectional area
through the flux is taken, we obtain the same current or flux
_,
of the current density J. (See Fig. 2.6.)
--
Fig. 2.7 The drift velocity is shown as it
varies in a conductor with varying c ~
section.The arrows indicate values of vd
at a few representative points. If the
carriers are positive,...Wese arrows can be
used to represent J also. What if the
carriers are negative.

The current through a surface is the flux of the current


density for that surface .
• ,., >
Fig.2.6 Model of a Metal
--+ --+ --+ --+
To describe the flow of charge at points with\n a From combining eqn. J = nq v d• and eqn. J = cr E, we
conductor, we use the current density J, which is a vector _, _,
quantity. If the current density is uniform the magnitude J,o.f see that v d and E are proportional. That is, an applied field
the current density is the current I divided by the causes the carriers to move with a constant average velocity
_,
cross-sectional area A of the wire: v d. But if the field were to furnish the only force on a carrier,
I _,
J = - (uniform J) then the carrier's acceleration would be constant, not its
A velocity. This means that when a carrier moves through the
J in terms of the drift speed v d: metal; there must be other forces on it. Indeed, since the
J = nAvdJqJ =nvdJqJ average velocity is constant, the sum of all forces on a carrier
A must be zero on the average.
This result can be expressed as a vector equation using The situation is analogous to a marble rolling down a
_,
the drift velocity v d: pegboard (Fig. 2.8). When first released, a marble will
accelerate down the board because of the unbalanced
component of the gravitational force down the board. As it
Note that the absolute value sign has been taken_,from collides with the pegs, we notice that its motion, averaged
over many collisions, is characterized by a constant average
Iq J. Thus the current density points in the direction of v d for velocity down the board. Averaged over many collisions, the
_,
positive carriers, and it points opposite v d for negative force on a marble due to the pegs is equaland opposite the
_, . component of the gravitational force down the board. A free
carriers. Consequently, the direction of J coincides with the electron is similar to a marble, the lattice ions are similar to
sense of the current in a wire. the pegs, and the applied field is similar to the componenrnf
the gravitational field down the board.
Concept: If the conductor contains more than
_, one type
of charge carrier, then there is a contribution to J from each
type of carrier. Suppose there are two types of charge carriers,
a and b. Then,

www.puucho.com
Anurag Mishra Electricity and Magnetism with www.puucho.com

[2"14 ELECTRICITY & MAGNETISM


. - ' -+ -+ --+
The drift velocity is-v d = (v) = (vx) i, so that
-+ -eE-r:-:
Vd = - - 1
m
.... .... .
Using 'eqn. J = nqv d• we obtain

.. ·_.,- J=n(-e{-;:'1)
Fig. 2.8
ne 2 T -: ne 2 T-+
=--El =--E
m m
The free electrons in metals are the valence electrons .... ....
that are weakly bound to the atoms when the atoms are Comparing this with eqn. J = er E, gives
isolated (not part of a metal). When the atoms are side by ne 2 t
er=-- ... (1)
side in a solid, these electrons are free to move through the m
material. Thus the number density n of carriers is the Since p =1/ er, we also have
product of a small integer and the number density of atoms m
p=-- ... (2)
in the material. ne2't
....
The average velocity (v) of the free electrons is the ....
-+ -+ -+ '-+ -+ ' Concepts: If er= ne 2 ,jm is independent of E, then the
carrier drift velocity v d: (v) = v d· Since E ex: v d, the average
.... model yields Ohm's law. The factors n, e, and m are plainly·
velocity is zero, when there is no applied field. That is, if E =
....
independent of E, but what about , ? Expect , to depend on
0, then (vx) = (vy) = (vz) = 0. The behavior of the free ....
electrons is similar to the behavior of the molecules of a gas (v 0 ), and E may change (v) by no more than v d· But noted
in that their velocities are randomly directed. earlier.that (v) "10 6 m/sand vd "'10""' m/s Because of this•
. .... -; .
The force by the applied field E on an electrons is vast difference, we expect , to be essentially independent of E. ·
.... .... ' .
-e .
F = E . Since this is the only force on a free electron
.... .... We can express er and p in terms of the average speed (v)
--

between collisions, Newton's second law, i: F = ma, gives by introducing the average distance an electron travels
between collisions, the mean free path '}._ :
the acceleration between collisions as ; = -e 11/m. If we
'}._ =(v),
....
align our x-axis along E, then the x-component of a free Substitution for , into eqn. (1) and (2) gives
electron's velocity at a time' t after a collision is ne 2 '}._ m(v)
er=-- and p=--
vx =vxo+axt=Vxo-(:} m(v) ne 2 '}._
Concept: It is inappropriate to apply Newton's second'
where v xo is the x-component of the electron's velocity
law to the motion of an electron in a metaL An electron in a·
immediately after the collision. On the average, we have
'metal must be described according. to quantum .mechanics.
(vx) = (vxo) -( : }
CURRENT DENSITY AND DRIFT SPEED
where , is the time· interval between collisions. This time As electrons are accelerated by an electric field, which
interval is often-called the mean free time, or the relaxation
· time.
exerts a force -e Eon the electrons. Here static conditions do
-;
-;

not apply and E can be non-zero inside a conductor.


Concept: For a rather sizable current, the drift speed v d The electrons collide with the ions of the lattice and
is only about 10""' m/s, whereas the average speed (v) of the transfer energy to them. The motion of individual electrons
free electrons is about 10 6 m/s. Since (v) is a factor of about is therefore very irregular, consisting of a short interval of
_10 10 larger than v d, the contribution to the motion of the free· acceleration in a direction opposite to the electric field,
electrons due to the applied field is negligible at .the' followed by a collision with an ion that might send the
'microscopic level. Therefore it is valid to assume that the· electron into motion in any direction, followed by another
velocity of an electron immediately' after each collision is acceleration, and SC' on. The net effect is a drift of electrons
' ....
randomly directed relative to E, or (v xo) = 0. in a direction opposite to the · field. There is no net
acceleration of electrons, because they continually lose
Thus energy in collisions with the lattice of copper ions. In effect,
energy is transferred from the applied field to the lattice (in
the form of internal energy of the conductor, often observed

www.puucho.com
Anurag Mishra Electricity and Magnetism with www.puucho.com

. ·-· - . . 7
_21~_J
as a temperature increase). On the average, electrons can be The resistance of an object is independent of
.... the magnitude or sign of the applied potential
described as moving with a constant drift velocity v d in a
difference.
direction opposite to the field.
Concepts:1. Potential difference 1W, I and R are
Concepts: 1. If the electrons drift at such a low ~peed, macroscopic quantities, applying to a particular body or
why do electrical effects seem to occur immediately after a
extended region. The corresponding microscopic quantities
switch is thrown, such as when you tum on the room lights ? -> ->
Distinguish between the drift speed of the electrons and the are E, j and p ( or a); they have values at every point in a
speed at which changes in the electric field configuration body. The macroscopic quantities are related by /!,.V =IR and
_, _,
travel along wires. This latter speed approaches that of light. the microscopic quantities by E =p j
Similarly, when you tum the valve on your garden hose, with
the hose full of water, a pressure wave travels along the hose The macroscopic quantities /!,.V, I and R are the quantities
at the speed of sound in water. The speed at which the water whose values are indicated on meters. The microscopic
_, ....
moves through the hose is much lower. quantities E, j, and pare of primary importance when we are
2. Between collisions with the lattice ions, the electrons in concerned with the fundamental behavior of matter (rather
.... than of specimens of matter).
a conducting material are accelerated by the electric field E,
.... 2. Ohm's law is not a fundamental law of
and so their drift velocity is proportional to E The current electromagnetism because it depends on the properties of the
.... .... ....
density J is also proportional to v d, so J should be conducting medium.
.... In a metal, the valence electrons are not attached to
proportional to E. The proportionality constant between the individual atoms but are free to move about within the lattice
current density and electric field is the electrical conductivity a, and are called conduction electrons.
of the material:
-> -> In the free-electron model, the conduction electrons are
J = crE ... (1) assumed to move throughout the conducting material,
A large value of a indicates that the material is a good, somewhat like molecules of gas in a container. In fact, the
conductor of electric current. The conductivity is a property of assembly of conduction electrons is sometimes called an
the materia~ not of any particular sample of the material electron gas.
Also resistivity, which is the inverse of the conductivity: In the absence of an electric field, the electrons move
p = 1/cr randomly, an electron collides with an ionic core of the lattice,
-> -> suffering a sudden change in direction in the process. We can
Hence E =p J ... (2) associate a mean free path 1. and a mean free time , to the
1
Note that 1 ohm = (1 siemensY- average distance and time between collisions. (Collisions
.Equation (1) and (2) are valid only for isotropic between the electrons themselves are rare and do not affect
materials, whose electrical properties are the same in all the electrical properties of the conductor.)
.... In an ideal metallic crystal at, 0 K, electron-lattice
directions. In these materials, J will always be in the same
.... collisions would ncit occur, according to the predictions of
direction as E . quantum physics; that is, 1. -. co as T -. 0 K for ideal crystals.
For certain materials, the resistivity does not depend on Cause of collisions in actual crystals .
the strength of the applied field for a wide range of applied, (1) the ionic cores at any temperature T are vibrating
fields. For such materials, a plot of E againstj gives a straight about their equilibrium positions in a random way;
line, whose slope is the resistivity p. These materials are (2) impurities may be present; and
known as ohmic materials. Such materials are said to satisfy' (3) the crystal may contain lattice imperfections, such as
Ohm's law. missing atoms and displaced atoms. Consequently, the
The resistivity (or conductivity) of a material is resistivity of a metal can be increased by (i) raising its
independent of the magnitude and direction of the applied temperature, (ii) adding small amounts of impurities, and
electric field. (iii) straining it severely, to increase the number of lattice
We obtain an expression for the resistance R, imperfections.
L When we apply an electric field to a metal, the electrons.
R=p- .modify their random motion in such a way that they drift
A
slowly, in the opposite direction to that of the field, with an
The resistance R is characteristic of a particular object
'average drift speed v d.
and depends on the material of which It is made as well as
on its length and cross-sectional area; the resistivity p is
characteristic of the material in general.

www.puucho.com
Anurag Mishra Electricity and Magnetism with www.puucho.com

j21s ELECTRICITY & MAGNETISM

Fig. 2.9 suggests the relationship between these two therefore Va=( e!~}
speeds. The solid lines suggest a possible random path'
followed by an electron in the absence of an applied field. The also ' ' v,
. .. .
..
l=neAva m(e)
dashed lines show how this same event might have occurred if therefore
A:

:A
~ i
I =ne
' ' ' '•
2
:an electricfieldE had been applied. When afield is applied to: A= eti.V, ne (ti.V)A,
l'N
·an electron in the metal, it experiences a force eE, by Newton's' ml ml Fig. 2.10
second law, ' ne 2 A,
a=-
eE -I = - -
ti.V ml
m
I ne 2 A,
-=-- => R =2~
R ml · ne A,
But we have already derived that,
·p-(- ne m )
2 -r
___ _y'
therefore R =pl
A
For l=l and A=l,R=p
i.f., the specific resistance is the resistance offered by a
conductor of unit length and unit cross-section.

X
Note: The formula R =pl is applicable only when
A
Fig. 2.9 cross-sectional area A and resistivity p are same
Consider an electron that has just collided with an ion: throughout the conductor.
!core. The electron has a truly random direction after the,
collision. During the time interval to the next collision, the' If conducting rod of uniform cross-section is stretched or
electron's speed changes, on the average, by an amount: compressed (volume remains constant) then,
a(1/Vav) or a,, where , is the mean time between collisions.' volume = Al => constant
1We identify this with the drift spe~d va, R=p..!_·xl
efa A l
vd = a t = -
m 12
We mqy also express v a in terms of the current density'.
R=p- R oc 12
Al
iwhich gives ' l A
J efa R=p-x-
vd =-=-·-_ A A
ne m lA l
R=p- Roc-
Combining this with (p = E/ ,1), we finally obtain A2 A2
m A graph of the current through a circuit element versus
P = ne 2, the potential dffference is called the characteristic curve
The quantity , depends on the speed distribution of the, of_ the circuit element.
,conduction electrons that, is affected only very slightly by the I , Current '
'application of even a relatively large electric field, since v av is· :through
Slope=k
·of the order of 10 6 m/s, and v a is only of the order of lo--4 0
resistor
:m/s. Whatever the value of, is in the absence of a field, it
l=~V
,remains unchanged when the field is applied. Thus the p is
1indep_e"'d~nt of~ IJ!!cd the_"'oat_erial obeys Ohm's_law. _ ! Potential
--+-t-V~•~--;~V
difference
EXPRESSION FOR RESISTANCE across resistor
We have already derived that, Fig. 2.11. The characteristic Fig. 2.12. The characteristic
curve of a resistor curve of ali independent'
Va=(:} __ voltage source ___ -· ... '

but

www.puucho.com
Anurag Mishra Electricity and Magnetism with www.puucho.com

ELECTRIC CURRENT ___ _ 21u


The total energy consumed by any device is simply its
power consumption. multiplied by the time it works. The
household electricity connections use the unit
kilowatt-hour (kWh).
V
One kWh = (1000) (3600) = 3.60 x 10 6 J
The average electric current is defined as the net
amount of charge that passes through the wire's full
Fig. 2.13. The characteristic curve of a Zener diode
cross-section at any point per unit time.
Fig. 2.13 shows the characteristics of a non-linear I = /!,.Q (average current)
device. We define the resistance of the device as the inverse av At
of the slope of the characteristics curve at any point. where l!.Q is the charge passing through cross-section
R=-l-=dV during time interval l!.t. If the amount of charge /!,.Q charges
dI/dV dI in time, we define the instantaneous current as
I= lim /!,.Q = dQ
Concept: The vibrational energy of the lattice increases AHO /!,.t dt
with temperature; as a result the atoms are moving more,
The SI unit of current is ampere.
rapidly and are arranged in a less orderly manner. So
interference in the flow of electron is increased. Therefore, the 1 A=lC/s
resistivity of metals increases with temperature. The Conventionally the direction of current is taken to be the
resistivity of metals usually increases approximately linearly" direction of flow of positive charge.
with temperature. Resistors in Series
In the series connection current through each resistor is
p
same. The potential difference across each resistor is found
p(T) ~ Po [1 + Oav (T- To)] from Ohm's law.
V1 =IR 1 , V2 =IR 2 , V3 =IR 3
- R1 R2 R3
A + + +
Approximately
linear region
Po
' Ap
1 Slope = AT = PoClav
', T
1 To
:__ _____ __ Fig. 2.!4 Fig. 2.15

p = Po[l + Uav (T-To)] We wish to replace a series of resistors with a single


where uav is the mean temperature ·coefficient of equivalent resistor R,q. connected between the same two
resistivity. terminals A and B. The total potentia_I difference between A
1 /!,.p and Bis
Pav =;;-;;- /!,.T V=V1 +V2 +V3
Since resistance is proportional to resistivity, we have Since the potential difference between A and B across
R,q.. Also must be V, we have
R =Ro[l + UavCT-To)l
IR,q. =IR 1 + IR 2 + IR 3
Electric Power
R,q. =R 1 +R 2 +R 3 ... (i)
The energy transformed when an infinitesimal charge
As current is sa!'le through each resistor in series
dq moves through a potential difference V is dU = V dq. The
combination..
power P is the rate at which energy is transformed.
Thus we have therefore V =IR
dU dq gives VocR
P=-=-V=IV ... (i) i.e., V1 : V2 : V3 = R1 : R2 : R3
dt dt
= I(IR) = I 2R ... (ii) and if V1 +V2 +V3 =V
R,
(~)v= :2 ... (iii)
then ½ --~--v
R1 +R2 +R3
Similarly expression for V2 and V3 can be obtained.
Eqns. (ii) and (iii) appiy to resistors only, whereas (i) is Power dissipated in the resistor in time 't',
applicable to any device. The SI unit of electric power is
P =I 2 R
same as for any kind of power, the watt (1 W = 1 J/s).
www.puucho.com
Anurag Mishra Electricity and Magnetism with www.puucho.com

'218 ELECTRICITi&_MjlGNETIS_M _]

PocR produces a pressure difference !lp = Phi - p 10 between its


i.e., P1 :P2 :P3 =R1 :R 2 :R 3 . outlet O and inlet I. Driven by this pressure difference, water
Dividing equation (i) by V2 , flows through the pipes in the sense of decreasing pressure,
R R1 R2 R3 as shown by the arrows. If its pumping capacity is sufficient,
-=-+-+-
v2 v2 v2 v2 the pump will maintain the pressure difference !lp even if
1 1 1 1
the valve is opened wider.
-=-+-+-
w W1 W2 W 3
Resistors in Parallel
If three resistors are· connected to same nodes A and B,
the potential difference V across each resistor is the same,
t
Value
We wish to replace the set with an equivalent resistor with
resistance R,q. so that the ,potential difference across R,q. is
same as across any of the resistors in parallel.
I
Pump
A

G:
1, 12
+
--+-
-
Fig. 2.17

I
B
1. External mechanical energy is converted by the pump
Fig. 2.16 int!) JJOtential energy of water.
2. The potential energy is converted into kinetic energy
The current I reaching node A is divided into I 1 , I 2 and of .flow as the water moves through the pipes outside the
I 3 in the branches containing resistors R1,R 2 and R 3
pump.
respectively.
3. The kinetic energy is converted into heat energy by
... (1)
frictional processes. ·
and V=I 1R 1 , V=J 2R 2 , V=I 3 R 3 ... (2)
4. The process continues as long as external mechanical
V V V, V3
- - = -1 + - + - energy is supplied to the pump.
R,q_ R1 R 2 R 3 The device that produces the emf called a source of
1 1 1 1 emf is usually part of an electric circuit, which is a
--=-+-+-
R,q. R1 R 2 R 3 closed path for circulation of charge, just as the system of
Figure is a closed path for circulation of water. Indeed, we
In parallel combination, the equivalent resistance is
can describe the cycle of charge circulation in terms closely
smaller than the smallest one in the combination.
analogous to those used for the water system:
Since V is same for all resistors in parallel.
1. External energy is converted by the source of emf into
1
Therefore I oc - potential energy of electric charge.
R
2. The potential energy is converted into kinetic energy
or J 1 :J 2 :I 3 = - : - : -
1 1 1 of flow as the charge moves through the part of the circuit
R1 R 2 R 3 external to the source of emf.
if I 1 +I 2 +I 3 =I 3. The kinetic energy is converted into heat by the
1 "frictional" process that is equivalent to the phenomenon of
I - Ri xI electrical resistance.
then I - 1 1 1 4. The process continues as charge continuously arrives
-+-+-
R1 R 2 R3 at the source of emf.
A battery is connected to a "device." The device may be
Power (P) = V 2 /R
a single cirCuit element, such as a resistor or a capacitor, or it
P oc (1/R) may be a combination of circuit elements. The battery
p =P1 +P2 +P3 maintains the upper terminal at a potential V+ and the lower
ELECTROMOTIVE FORCE terminal at a potential V_. For an ideal battery, the potential
difference V+ - V_ between its terminals is independent of
AND ITS SOURCES the amount of current that it is providing to the circuit.
Fig. 2.17 schematically shows the water analogy that
will guide us in our study of electric currents. The pump

www.puucho.com
Anurag Mishra Electricity and Magnetism with www.puucho.com

Current provided to them by the· source of emf. Eventually, they


return to the negative terminal, from which the emf raises
Wire
them to the positive terminal again, and the cycie continues.
When a steady current has been established in the
circuit a charge dq passes through any cross-section of the
1
Battery•
circuit in time dt. The emf & of the source is defined as the
work per unit charge, or
&=dW/dq
The unit of emf is the joule/coulomb, which is the same
as the volt.
The source of emf provides energy to the circuit. Its
Wire
energy might be obtained from a variety of processes:
chemical (as in a battery or a fuel cell), mechanical (a
Fig. 2.18 generator), thermal (a thermopile), or radiant (a solar cell).
The current in the circuit transfers energy from the
The battery can be considered a "pump" for charge, as if
source of emf to the device. If the device is a resistor, the
it were bringing positive charge through the battery from
transferred energy appears as internal energy (observed
the negative terminal to the positive one. In actuality, it is
perhaps as an increase in temperature). If the device is a
usually the motion of the negatively charged electrons that
capacitor, the energy transferred is stored as potential
is responsible for the current flow. ·
energy in its electric field.
Concept: The function of the battery in the circuit is to A battery can either be charged (meaning an external
maintain the potential difference that enables the fl.ow .of source adds to the battery's supply of energy, not that we are
charge. The battery is not a source of electrons. Electrons pass forcing more charge into the battery) or discharged
through the battery and have their energy raised as the move (meaning we take energy from the battery).
inside the battery from the positive to· the negative terminal. Symbols for Circuit Diagrams
Conductor with negligible resistance
When we say that a battery is "drained," we do not R
mean that it has "used up" its supply of electrons; instead, --~'W-1'~--- Resistor
we mean that we have exhausted the source of energy (often
a chemical reaction) that was responsible for raising the. ----l+ I Fe_ __ Source of emf (longer vertical line always
represents the positive terminal, usually the
energies of the electrons. Note in Fig. 2.18 that the electrons terminal with higher potential)
move throughout the entire circuit; they do not "come from" - .!l'.1+
--~'IVV'l4'---- Source of emf with internal resistance r
the battery. (r can be placed on either side)
The direction of current is the direction that positi11e -----,(y)1---- Voltmeter (meas_ures potential difference
charges would move, even if the actual charge carriers are between its terminals)
negative. -----,@ Ammeter (measures current through ii)
We assume that, under steady conditions, charge does
not collect at or drain away from any point in our idealized Concept: 1.
wire. Battery and EMF: When a device is connected to a
Concept: The electric current T is the same for all ·circuit element it maintains a potential difference between
cross-sections of a conductor, even though the cross-sectional ,terminals of element. The battery has an internal mechanism
--> •
area may be different at different points. The current density that exerts force on charges, we refer is as Fb .
-->

~
J (current per unit area) will change as the cross-sectional
·area changes, but the current I remains the same.
- -
At any junction in an electric circuit, the total current
entering the junction must be equal to the total current leaving
the junction.
A source of emf must be able to do work on charge
---rInitially neutral
conductors
~3r
...
Fb is force of battery
on positive charges
Fig. 2._19 Fig. 2.20_
carriers that enter it. The source acts to move positive
charges from a point of low potential (the negative
terminal) through its interior to a point of high potential
(the positive terminal). The charges then move through the
external circuit, in the process dissipating the energy
www.puucho.com
Anurag Mishra Electricity and Magnetism with www.puucho.com

220 E~ECTRICllY_& MAGN_~T!S~_j


-> The wires in our circuits are conductors, and so there
Force of battery Fb is a non-electrostatic force. The
->
must be an electric field present to establish and sustain the
battery that can maintain Fb for q longer duration is a good, current. Where does this electric field come from ?
-> ' The battery provides the emf to the circuit; its role is to
battery. Due to Fb positive charge accumulate on terminal A "pump" charges from low potential to high potential. The emf
and negative charge on B. Due to this charge separation on is defined as the work per unit charge done by the battery.
-> ->
electric field E is developed in the material from A to B. With the conventional definition of work done by a force F as
=f F· d S,
-> ->

~~~
W the work per unit charge W / q (the emf) must
->
then be related to the force per unit charge F/q:
1. -> ->
E =:y(F/ q)-dS
Fig. 2.21 ->
-> ->
The force F in this case is the one that acts inside the
The charge separation occurs till the Fb = q E when ·a' source of emf; it might be a force of mechanical, chemical,
-> ' thermodynamic, or magnetic origin, ·but it is not necessarily
charge q is transferred from A to B workdone by F b is Fb d and 0
associated with an electric field. The emf depends only on the
emf is defined as ·net effect of the source on a charge that makes a complete loop
E =Fbd around the circuit. Conservative external fields cannot give an
->
q , F ->
In the absence of any entered circuit current emf, because integral
. q
f- ·
d·S vanishes for such fields.
Fbd
E = ~ =qEd=qV Inside the wires, there is an electric field_ This field must
q be present for charge to flow in the wires.
Where Ed is potential difference between terminals A and ' When the battery is first connected to the circuit, initial
B. ,transient currents are established. These currents distribute
--->
E charge along the surfaces of the wires in just the precise way

r necessary to establish the electric field that maintains the


steady current in the wires- and the entire process takes place
·'in a time that
f is- .typically
. of the . of nanoseconds !
,. order
~II~
. + E + +
AI B
'

Fig.2.22 +
When an external circuit is connected across terminals A
and B. Free electrons more toward A, at the same instant
electron are driven at each cross-section of circuit. When an Resistor
Battery
electron reaches A and another moves. away from B, the
-> ->
electrostatic field E r/ecreases. Now 'electrostatic force q E
->
decrease but force F b due to internal mechanics of battery,
remains uncharged_ Thus there, is a net force on the positive
charges of material from B to A. Thus the potential difference - ---> -
E
between A and B is maintained_
Fig. 2.23
2. When a current is driven into battery; positive charges
enter battery from positive terminal and leave at B such a' 4. In practice, the current is the same all the way around
process is called charging of battery. When we draw current it the loop, at any given moment.
leaves out of positive termina~ this process is called If the current is not the same all the way around then
discharging of battery. charge may collect somewhere, and the electric field of this
3. Electric Fields in Circuits: The relationship· accumulating charge is in such a direction as to even out the
-> ->, flow. Suppose for the instance, that the current into the bend
,between current and electric field in a conductor: E =p J in Fig. 2.24 _is.grea~er than the current out.
->
where p is the_resistivity of the material and J is the current
density (current per unit cro~s-sectional area).

www.puucho.com
Anurag Mishra Electricity and Magnetism with www.puucho.com

' -
I_ ELECTRIC CURRENT 221
current inside the battery is from the negative terminal
toward the positive terminal.
+
+ 1\vo important characteristics of a

+
+ battery are its emf & and its internal
resistance r. The emf characterizes the
A
~w
+ + energy that the battery provides the
charge carriers, and the internal
resistance is the battery's own
resistance. Fig. 2.26 shows how the emf i
and internal resistance can be Fig. 2.26
Fig. 2.24 determined. A voltmeter placed across
the battery measures the battery's terminal potential
Then charge collects at the ''knee," and thi.s produces a difference V and an ammeter measures the current I. The
field directed away from the kink. Thi.s field opposes the current can be changed by changing the resistance of the
current fl.owing in and promotes the current fl.owing out until variable resistor (shown as .Nf,r) Fig. 2.28 shows a graph
these currents are equal, at which point there i.s no further of V versus I that is typical for such measurements. The
accumulation of charge, and equilibrium i.s establi.shed. equation that gives this graph is
5. In a similar fashion, we can ask how the current · V=&-lr ... (1)
''knows" to change direction when it encounters a bend in the V
·wire. Fig. 2.25 shows a schematic view of a right-angle bend,,
for which the surface charges must be di.stributed roughly as
shown. The negative charge sets up a field near the bend that E, r
opposes the motion of the oncoming current, and the positive A R
charge provides an initiql ''push" in the new direction.
+++ {',, s
----{>
-
E Fig. 2,27_
0
Fig. 2.28
The graph's intercept equals the value of the emf E, and
E its slope gives the internal resistance r. Thus the battery's
emf E is its terminal potential difference when the current in
the battery is zero:
&=V (when!= 0)
The emf of a battery can be measured by placing a
Fig, 2.25 high-resistance voltmeter across its terminals while the
terminals are not connected to anything else. In this way the
The batte,y provides the initial ''burst" of current to the
current is so small that the Ir term in eqn, (1) is negligible
circuit, and almost instantly the charge finds its way to the.'
compared with&.
locations where it guides the steady current and prevents:
further build-up ofcharge on the surface of the wires. This emf has the same dimension as electric potential,
equilibrium i.s maintained as long as the battery continues to namely; energy per unit charge. The dimension energy per
pump charge around the ci~cuit. unit charge indicate~ !"'e physical__nature of emf.
Concept: If we consider a small current so that we can
EMF AND INTERNAL RESISTANCE
:neglect the Ir term then we can describe the emf of a battery as
OF A BATTERY ,the electric potential energy per unit charge given to the
For an electric circuit to have a continuous current, the charge carriers by non-electrostatic forces in the battery as the'
circuit must contain an element that is a source of electric ,carriers pass from one terminal to the otlzer. These force are a
energy. Such an element is called a source of emf. !result of the chemical action of the battery.
The battery produces steady current by maintaining a
nearly constant potential difference across its terminals. The In Fig. 2.29, we show the internal resistance separate
terminal that is at the higher potential is called the positive from the emf, even though they cannot be physically
terminal and the terminal that is at the lower potential is separated. Traversing the battery along the sense of the
called the negative terminal. Thus the sense of the current current, we find that the potential increases by the amount E
outside the battery (through the resistor) is from the positive because of the chemical action of the battery and decrease
terminal toward the negative terminal, and the sense of the by Ir because of the resistance of the battery; which
illustrates the relation V = & - Ir

www.puucho.com
Anurag Mishra Electricity and Magnetism with www.puucho.com

1222 ELECTRICllY & MAGNETISM


l-. ----1 b where the potential is Vb (Va > Vb). Th~ change in the
, ·E r . : + electric point energy AU of the carriers is
-• ,; I ~
1
1 I
I I • ! AU= VbAQ - V0 AQ
]...&...+I 1--:,...
.... ·- -:- ,:~~-,:"'--.Battery
. =-{Va -Vb)AQ=-VAQ
' '
I I '
I•
.. The. quantity AU is negative because the potential
' '
V !' : ·': I decreases along the sense of the current. The rate at which
I I --1----':-
the· carriers lose electric potential energy is -AU/ M, and we
v.l--.--!-..1----"--=---' ~f~- call this rate the power PR 'dissipated in the resistor. Thus
lr ' - AU -VAQ AQ
p =--=---=V-
V- 1-'Y'--'-:____._ R M M M
Since I = AQ/ M,
Fig. 2.29 A battery with current
I in it. The sense of.I is the same ~=W ,,.m
as the sense of &. The terminal We can write PR in terms of the resistance R of the
p_~t~~tial f!iff~r_eD~~J~ y_ = '{+ :- V_ . resistor by using V =IR. We have PR =IV= I(IR), or
If the sense of the current in a battery is the same as the :, PR=I 2 R ,.,(2)
sense of its emf, as seen in Fig. 2.29, then the battery is said Alternatively, PR = VI = V(V/R),
to be "discharging," and _ _ _ __ v2
eq. V="-lr
c::;.,
is_valid
.
for + ,
, _ E · ~ i, -
·or PR =- ... (3)
R
this case. On tlie other • ; ,I ~
hand, if the sense of the ! ;. I ~ : i · Eq11_a~on (l),_ (2) and (3) is_called Joule's law.
current is opp·osite the v ·-i--i-!-'-"'- · Concept: The carriers lose energy in the collisions that
emf, as shown in Fig. 2.30, : : : Battery -'are: ·responsible for the resistance of the resistor. When a
then the battery is said to , v. ' ' '
f-~tc-1
-1:'.::--f- 'resistor carries a current, the temperature of the resistor tends
be "charging" or is "being I :to increase as a result of these collisions. If the temperature of
charged." From the figure v : :e
we see that ifwe traverse a , 1- i i
i
charging battery along the v______ : ______ ::-, __,.._~-
1 l'..
'the resistor rises above that of its surroundings, then the
,resistor transfers heat to its surroundings. (Recall that heat is'
,the transfer of energy due to a temperature difference.), Under;
sense of the current, then steady conditions the energy_ is continuously transferred as,
the potential decreases by ;Fig. 2.30 A battery being· charged. , :heat to the surroundings. These effects are sometimes called
'. The sense of I is opposite the sense •
the 11mount Z b ecause Of :0 1e.The terminal potential difference ' )
2
R heating, sometimes called Joule heating or sometimes
chemical reactions in the ;;~ \/ :' v.2 '{__ __ _ __ __ _ _ _ : !called ohmic heating.
battery. As with a , _
discharging battery, the potential decreases by Ir because of Energy to or From a Battery
the battery's resistance. Thus . the_ terminal potential . . · If charge carriers pass through a battery in the direction
difference V across a battery that is be(ng charged is such that the sense of the current is .the same as the sense of
V=Z+Ir ' the emf (the battery is discharging as in Fig, 2,26 and 2.29)
ELECTRIC EN.ERGY AND POWER then their electric potential energy increases. Let AQ be the
When a cu~ent exists in a circuit element, energy is ainount of charge that passes through the battery in time M.
transformed. We now investigate energy- transformations The change in the electric potential energy of the carriers is
due to currents in circuit elements.· · · AU = V AQ, where V is the terminal potential difference
across the battery. In this case, AU is positive because the
Energy Dissipated in a Resistor, electric potential increases through the battery along the
Consider the energy ,---A0 -- ---- ··-;;,a-- sense of the· current (see Fig, 2.29) Tiie rate at which the
transformed when a resistor of j ·- v/Jw - carriers gain electric potential energy is AU/ M, and we call
· resistance R carries a current I, 1, v a ~ :,b this rate the power output P0 from the battery:
as shown in Fig. 2.31. The 1, ,
potential difference across the 1 P =AU =VAQ =W
resistor is V = Va - Vb and the IV,
[ t :, 0
M M
sense of the current is from a to V :
Using the expression for the terminal potential
b. In a time interval M, a
number of carriers with total , ,
!v. J___ --------,!---
0
difference across a discharging battery, V = Z - Ir, ·
wehave P,=IV=I(Z-Ir)
charge AQ enter the resistor at a' i,'
point a where the potential is Fig, 2.31
v. and a number of carriers ---- - - - --- '
with an equal total charge AQ leave the resistor at point b

www.puucho.com
Anurag Mishra Electricity and Magnetism with www.puucho.com

ELECTRIC CURRENT 223


Concept: ... (4) Hence V = & -Ir
The term & I in eqn. (4) represents the rate at which the Thus during the discharging of a cell the terminal
electric potential energy of the earners is increased by potential difference is less than the emf of the cell and if the
chemical reactions in the battery. We shall call this the power circuit is open i.e., no current is drawn from the cell,
PE expended by the emf of the battery: P& = & I. We recognize then V=&
2
the I r terms as the power P, dissipated in the battery·due to The internal resistance of the cell (r) can be expressed in
its resistance r. (The temperature of the battery ten<¼ · to terms of E and V as,
increase.) This term represents a rate of loss of electric
potential energy for the earners and properly enters the r=R(~-1)
expression with a minus sign. Thus eqn. (4) states that the During the discharging of a cell, it continuously losses
power output P, of a battery is equal to the power P& the energy.
expended by the emf minus the power P, dissipated as heat:
Energy lost by cell= q& (where' q' is the charge flown)
P,=Pe-P,.
= Idt &f
Equation (4) is valid for a discharging battery. If a
Power Transferred to the Load Resistance
battery is being charged, then the sense of the current is
opposite the sense of the battery's emf. In this case the P =I 2R
potential decreases along the sense of the current and the & &2 )2
electric potential energy of the carriers decreases as they =( R+r R= (R+r) 2 R
pass through the battery. The power input P1 to the battery is p
&2
equal to the rate at which the carriers lose electric potential i-----,,-r-.;Pmax=4r
energy in passing through the battery: P1 = IV. Since the
terminal potential difference across a charging battery is
V=&+~ .
we have P1 =IV =I(& +Ir),
or P1 = & I+ I 2r 0 R=r R
In this case the product & I represents the po\A/er Fig. 2.33
delivered to the emf of the battery by the charge carriers. For P to be maximum,
Energy to or From Any Circuit Element dP
-=0· R=r
Next we consider the rate of energy transformation Pin dR '
any type of circuit element. If V is the potential difference Therefore the power transferred to the load by a cell is
across the element and I is the current in the element, then maximum when the load resistance of the circuit equals the
P = IV : .. (5) internal resistance of the cells the statements is named as
because Vis the change in the potential energy per unit maximum power transfer theorem.
charge for carriers that pass through the element and I is Total Power Consumed in the Circuit
rate at which charge passes through the element. The Total power,
product IV gives the rate at which the electric potential ·::;2
energy of the carriers changes as they pass through the =-=&I
R+r
element. If the sense of I is along the direction the potential
For Ptotal to be maximum, R = 0,
decreases (as in a resistor or a charging battery), then P &2
gives the rate at which the carriers lose electric potential (Ptotal ) max .=.-
energy. If the sense of I is along the direction the potential r
increases (as in a discharging battery), then P gives the rate And simultaneously the current is also maximum as,
at which the carriers gain electric potential energy. &
I=-~
Discharging of a Cell R+r
&
In the discharging of a cell the !max=-
current flows from cathode to anode r
inside the cell (in the direction of or R=O
~ I
emf). Efficiency of a Cell (ri) or Power Transfer Efficiency
Applying the Kirchoffs law, E Power developed in R (load resistance)
Fig. 2.32 ~=-----~---------
&-IR-Ir= 0 Total power supplied
& =I(R+r) 2
I R R 1
Where IR(= V) is the terminal potential difference and Ir = =--=---
I2(R+r) R+r. l+(r/R)
is the internal drop.

www.puucho.com
Anurag Mishra Electricity and Magnetism with www.puucho.com

ELECTRICITY & MAGNETISM

When the power is maximum i.e., R = r, then,


+
A A+
r l Remaining Short~I VAe=O Remaining Open IAe= 0
11 =2r=2 • , SC
circuit circuit lsc -- !AB -- 00 circuit circuit RAB= oo
T\ = 50% -
B B
~ p
(a) (b) .
1.0 Pmax
Fig. 2.37

0.5 .-Whole of the applied voltage is felt across the 'open',


i.e., across terminals A and B.
R A
R=r R 0 R=r
Fig. 2.34 B Req = R1

Efficiency will be maximum when the resistance (load) V R2 I=..'£.


R1
is infinitely large or (r/R) is very small. Short
C circuit Va=Vo
Charging of a Cell
I
• 1·1~1 D

ti Fig. 2.38

.-Bulb B1 acts as open circuit, bulb B1 will not glow.


+ V - However, other bulb B2 remains connected across the
Fig. 2.35 voltage supply; it will operate normally. ·
During charging current inside the cell is from anode to R

cathode and from the figure.


V=&+Ir 8
: 1 0 0 V = V n ~V . 100V VA8 =100V 1
i.e., during the charging of a cell the terminal . T _
~r~uit
potential difference (V) is greater than emf of the cell. ' ~
When cell is short circuited- the resistance outside Fig. 2.39
the cell becomes zero,
A

1{
1·1=1 ti 220.v
Open filament
'v Ra
Short
circuit

R=O B
Fig. 2.36 Fig. 2.40 Fig. 2.41

therefore I = __E_ = & .-short circuit across R3 shorts R1 and R 2 as well, short
R+r r across one brancb in parallel means across all
and V-IR=O branches. There is no current in shorted resistors. The
V=&-Ir = 0 shorted components are not damaged, they will
Hence current is maximum and terminal potential function normally when short circuit is removed.
difference (V) is zero. ..-Jn Fig. 2.42 shown, short circuit across R 3 may short
out R 2 but not R1 , because it is protected by R 4 .
Short and Open Circuits . . .

When two points of a circuit are connected together by a


conducting wire, they are said to be short-circuited. The
connecting wire is assumed to have zero resistance. No
voltage can exist across a 'short', secondly current through it
is very large (theoretically infinity) .
.-Two points are said to be open-circuited when F\g. 2.42
there is no direct connection between them, a break
in th,e continuity of circuit exists. Due to this break the
resistance between the two points is infinite and there
is no flow of current between the two points.

www.puucho.com
Anurag Mishra Electricity and Magnetism with www.puucho.com

ELECTRIC CURRENT

Voltage Divider
In a series circuit, current through each ,-;:::==;7
H ;J Rt
; VIt
2
j
. (using V ;JR)
resistor is same. , I ; cv
2
/R)t
V;V1 +V2 +V3

J;
;JR1 +JR2 +JR3
V
!v--
'I
As resistance of a given electrical appliance (bulb,
heater, press etc.) is constant, it is given by; R ; V, ·
2

R1 +R 2 +R 3 w
"•1 -- VR, Where V, and W are the voltage and wattage specified
R1 +R 2 +R 3 on the appliance.
Fig. 2.43 An electric appliance consumes the specified power W
V 2 ; - VR2 -~-- only if it runs at the specified voltage V,. If the applied
R1 +Rz +R3 voltage (VA) is greater than specified voltage (V,) then the
V:3 -- VR3 appliance will get damaged as in this situation the current
R1 +R2 +R3 J ; (VA /R) will exceed its current capacity (V, /R). If an
Each resistor has its own potential drops. Voltage across appliance is made to run at a lower voltage than specified,
each resistor is additive. then the power consumption will be, .
Current Divider
From Fig. 2.44 we have P; 1 CV,;~);(~ )\v.-
J ;J, +R 2 ... (1)
SERIES AND PARALLEL
J1R1 ;J2R2 ... (2)
On solving eqns. (1) and (2), we COMBINATION OF BULBS
get Parallel Combination
2 If the different bulbs are in parallel and a voltage
J, ;(R,: R2} VA (:S: V,) is applied across them as in case of house wiring
then the resistance of a bulb. ·
J2 ;(R,: R2}
1
R cc-
w
1
... (1)

The division of current in the


Since R; v,2
branches of a parallel circuit is w
inversely proportional to their V, : is specified voltage.
resistances. I U1 R, and W: is specified wattage.
1
Li:;g'--'.2~4,,_s_---J J CC- ... (2),
R
and 1
Pcc- ... (3)
R

Since p; v;
R
From eqn. (1), (2) and (3), we get
1
PccJccWcc-
R
In parallel grouping of bulbs the bulb of greater wattage
will give more bright light and will pass greater current·
It is easy to remember the expression for J 1,J 2 and J 3, through it and will have smaller resistance and if one bulb
notice which resistance is missing in the numerator. gets fused other will still work.
Heat Energy Produce in a Resistance Series Combination
The heat produced in a resistance in time 't', if a current, In series combination same current passes through each·
'I' flows through it and the potential difference across its bulb,
two ends is V is given by; R cc_!_ ... (1)
;J
H; J Rdt
2 w
If current is constant then, Since R; V,2
w
H ;J 2 RJ;dt V cc R (' J' is constant) ... (2)
www.puucho.com
Anurag Mishra Electricity and Magnetism with www.puucho.com

1226 ELECTRICllY & MAGNETISM j


P oc R ('I' is constant) ... (3) (c) When wires are connected in parallel, each
Since P =l 2R receives full battery voltage V. Thus the power consumed by
From equation (1), (2) and (3), we get each bulb remains the same as when only one bulb is in the
. l circuit, so the brighmess does not change.
P ocVocRoc-
W p = y2 = (48)2 = 9.6 w
In series combination the bulb of greater wattage will R 240
give less bright light and will have smaller resistance and =~-- -2.....,,, , .
-c.-_;:,;::,=.-.=e=m-:w=·=:P.1=1~1f-=~--"".
potential difference across it. If one bulb gets fused the other
will not work.
In an extended medium of conductivity er, the power
¼potential difference Vis applied to a copper wire of diametM
dissipated in an element of cross-section AS and length AZ is,
Id ~nd length L What is the effect on the electron drift speed ofi
l£ioublfogJiJvolwgeV, (iiJJmgthL ancL@iJ diameter d ? I
t.P = AVJAS = Et.UM
or Power dissipated per unit volume, · . Solution: Accordi~g to 'electron theory of i'rietals' the
= JE = aEE = crE 2 · · · drift velocity of an electron inside a metal in presence of an
electric field E is given by:
-ese;'m'f~!e\~ Va =(:)E =(:)(~) [asE =f]
)A circuit contains a 48 V battery and a si~gle bulb whose
resistance is 240f! A seco·nd 'identical bulb can be connected So (i) As v a oc V, on doubling V, drift velocity will be
either in series or parallel with the first one. Determine the
doubled.
power in a single bulb when the circuit.contains (a) only one (ii) As va oc (1/L), on doubling L, drift velocity will be
bulb, (b) two ·bu.lbs in series and (c) two .bulbs in paralleL halved.
Assume that the battery is ideal without any internal (iii) As drift velocity is independent of diameter d, it will
not change on doubling the diameter.
!resistance., -----------~
'
Solution: Power consumed by a light bulb is related
to)ts·resi.stance Rand the voltage across'it·by· ·
if=~~ '
-.. -.- -, . - .. ' ·v·2 , , ·. . . IIThe. area of,c~oss-sectio~; :l~ngth and density of a piece of~
P"=- metal of atomic weight 60 are 10-6m 2, 1 m and Sx 103 kg/
·- R
m 3 respectively. Find the number of free electrons per unit
(a) When only the bulb is connected· in the circuit,
1volume if every atom contributes one free electron.
the power it consu11].es is . 1 • • , • •
l.4lso find the drift velocity of electrons in the metal when a
. y2 (48)2 . .
, , P ;=-.-. ""'-.- = 9.6W · . jcurrent of 16 A pQ-lses \h,ough it. Given that Avogadro's
R 240 number N ~ = 6x:i~ 23/mo1 and charge on an electron
!e=l.6xl0~ 9 c.... ,
0

(b) The more• "the power dissipateci• in a light bulb,


the brighter it is. . . ., .
·· ·When ·identical bulbs·are wired in series, each receives Solution: As according to Avogadro's hypothesis,
one half the battery voltage V. The power consumed by each
N
-=-
m ",·.;t·~-.-
bulb is
' ' ·, ,• \l:V)2
- ·p =:...L:....,__
.
,..!~
2 ,
,·, - - ' so,
,- ,,.l-,asd=-m]
. 'I
• ' ~, V
. .,1\ 4 1\
The power dissipated in each bulb is,reduced to only. . -,
one-fourth the power,dissipated in a single bulb circuit. Thus
the brighmess of each bulb decreases. Now as each atom contributes one electron, the number
The equivalent resistance of two bulbs is Req. = R + R. of electrons per unit volume
Th~,cµrrent in the circuit is.given by I =.V/Req.· n, =lx n = lx Sx 10 28 = Sx 1028 /m 3
. , The power consumed .by one of t:1,e light.bulbs can be
expressed as .. . .' ' Further as here:
2 ' ' ' 2 . I 16 6 A
P~I R=(-v-)
.- . -.·
2
R+R ,4R•.
R'=y
J=-=--=l6xl0 -

andasJ=neva · .
A .10-6
''
m2

' · N8) 2 • · 'J ·' l6xl0 6 ·,,.. -.-.·


. , . =2.4W. . Vd=-
_ :· ,4(240) ne (Sx 10 28 )x (l.6x 10-19 )
! . • • • ••• '
-., The power dissipa~ed in one-fourth a.s C(/mpared to that =2x 10-3 m/s
by a single bulb.
www.puucho.com
Anurag Mishra Electricity and Magnetism with www.puucho.com

ELECTRIC CURRENT ' 227 j


~~a~~Jjg--:-;'41:>
·~-- --·---- -----~~~
~""'° ff;-- · -- -- . ·-- ·····---···1 LlE~Ii~~~J
·· ·-- -· ~·------ I
6 I~ ,
IA copper wire of cross-sectional area 3.00 x 10-6 m 2 carries a, !If a copper wire is stretched to make it 0.1 % longer. Find the\
[current of 10 A. find: · l ipercentage change in its resistance:_------·-··-- ____ ,
'.(a) The drift speed of the electrons in the wire. Assume that!
,each copper atom contributes one free electron to the body of Solution: For a given wire, R = pL
!material. t
s
'.(b) The average time between collisions for electrons in the! with L x s = volume = V = constant
:copper at 20 °C. The density of copper is 8.95 g/cm 3, motor! So that, R = pL
:mass of copper 63.5 glmol, Avogadro number 6.02 x 1023 : s
;~lecifon{.F_!l_o.!_ a_1_1_cl__r:~i§_qyjty 9f copper0 _ •• . . ·-- _ •• • •• __ ! t,R = 2 M, = 2(0.1 %) = 0.2% increase
R L
Solution (a) The volume. occupied by 63.5 g of
copper ~~~~;~~
M 63.5 . s I r - ·- - ··---- ------ -·-1
V=-=--=7.09cm /mo Wig. 2E. 7 shows a thick copper rod X and a thin copper ."'.i~e YI
p 8.95
As each copper atom contributes one free electron to the
!joined in series. They cany a current which is sufficient toi
body of the material, the density of free electrons is
· 6.02xl0 23
n=---- 6
7.09x 10- .
· '
:i :' l' ·:
!make Y much hotter than X. > - - - - - - ~
11
'
. j:.: .,·",
_,·; ,

= 8.48 x 1028 electron/m 3


The drift speed
I
.

X
, 'Y·
.
·,. ·.:,,:.j.
I • , I•

vd = - -
Fig.2E,7
'I ' ' - ..I . J
neA 1 !
----·----·-:-- -·-_-- - - - - - - · ·•·-·-·-_ · - - - - • ' • ' I
• · 10.0 Which one of the following is. correct? ·•' · ' . I
I
I
. _va = 8.48x
. ,, =2.46xl0 m1·S·,' . ' •
' ' -4
10 28
x J.60x 10-19 x 300x 10-6 .·
' i
II
Number of density· ~ ~ean tiilje·b~i,i,~eri _c6lli~ionsof
conduction .. electro_ns . ·'\ :. · the elect,:ons . ., ; '{ -
I
' ,. ,. . 1
I
(b) Average. time between collision for electrons
I (a) Same inX and,Y. ~ss inX than in Y :
. , me
t=--
ne2p .. ' ( '. (b) Same inX and Y Same in x· arid Y ' .'. -j>
'·· •.
r. ~ ' ·=--------------'--,-~
28
8.48x 10
= 2.Sx 10-14
19 2
X
8
(l.6x 10- ) X l.7x l(}C
(c) More in X than' Y, ·
(d) More ' in X than .Y ' .
More in X than in Y
Less in X and_ Y
SameinXandY
.
· · ·
j!

F~Et>i~·r->li~ 5
~;g>-.,g -.:: r,c;,.£;- ~
----- - ---· - " -- -
f.;':,·~
-- - --;-,-- - ·-1
____,_ .. -- ---- _. -
-- -- ---- - - . --~ _. -- - - - - - '.
-----------~---------·----- - __ . _____ . ____ , __
..
.____
-
'.}
'Find the total momentum of electron .in .a straight wire of., Solution: (c) The number density n of condu'ctio~
~ngthl=l000m_canyingcurreniI_=,7'04A. _____ .-, __ i electrons in the copper is a characteristic of the copper and is
about 1029 at room temperature for both the copper rod X
Soluti9_n: _Let n be no. of electron per unit volume. and the thin copper wire Y. ,, · ·
No. of electron in I length : ·. - .. Both X and Ji. carry the same current I since they are
N = nSI · 'cs is cross-sectional area) joined in series.
Momentum of one electron = mv a From I= nAv.q
Total momentum P = (nSl)mv a Where q is the electron charge of 1.6 x 10-19 c, v is the
I
as vd = - - drifr velocity- hi 'i:Jie conductor and' A: 'is ·ihe cross-sectional
neS area of the conductor. ·' ' · · ·' · ··
l. '_ - mU
P = (nSl)m-- = - We may conclude that rod X has a lower drift velocity of
(neS) e electrons compared to wire Y since rod X has larger
On substituting numerical values, we get cross-sectional area. This is so because the electrons in X
P=0.40µNs ' collide more ofren with one another· and with the copper
ions when drifting towards the positive end. Thus, the mean
time between collisions of the electrons is more in X than in
,, '
Y.
www.puucho.com
Anurag Mishra Electricity and Magnetism with www.puucho.com

· ct2:;::_2s=--:_ _ _ _ _ _ _ _ _ ---- ---- ELECTRICITY &MAGNETISM

~~ma ~
What is the equivalent resistance between points A and B in
.
lwhen a metal rod is heated, not only its resistance but also its
the networks shown in Fig. 2E.8 (a) and (b) if each resistance ength and area of cross-section changes. Fjnd the per cent
is R ? change in .R, I and A of a copp.er wire for a temperature rise of,
•··-r-
1°C. Coefficient of linear expansion for copper is

.> R2
.·!R1 l
1.7 x 10-5/"C. and its thermal coefficient of resistance is
3_J) X 10-3,/°C.

Solution: If coefficient of linear expansion is a,


.. A .R,. . B .. . 0 ... A~
R, R4 percentage change in length is
R2
.
.),R4
.> R3 Al x 100 =aATx 100
I
Coefficient of area expansion is 2a, so percentage
(a) (b)
change in area is
Fig. 2E.8 M x 100 =2aATx 100
A
' .
. Solution:
percentage change in resistivity is
' ' . .. - A careful study of the given networks
~

reveals that:
Ap X 100 =CLR ATx 100
p
(A) In this network one end of all the four resistances is where a R denotes thermal coefficient of resistance.
connected to A while the other to B, i.e., potential difference All the variables ~ A and p are function of T.
across each resistance is same and equal to applied between Consequently, R is also a function of T. We find, therefore,
A and B while current divides~''--------
dR d (pl)
dT=dT A
. I· dp p di d I (I)
lf.---'V\tv---+--O=-v,·w---4A_B = A'dT+ AdT+p dT A ··
R4 . '( p 1 .
= CJ.RP-+ al- --2aApl
·. ,_A_. A A2 •

,_'---<,/\/\,----' .
= (CLR-+·a-2a -= CLR -CL)-
)pl ( . pl
(c) (d) A A
dp . ,',
Fig, 2E.8 where we have'· written - =CJ.RP fr.om equation
'--------'---'- --~---'-------' dT
V1 = V2 = V3 = V4 = V . and I = I 1 + I 2 .+ I 3 + I 4 di dA ...
Ap = a RP AT; similarly - = al and - = 2CX{l..
So the given four resistances are in parallel [as shown in . rIT dT ..
Fig. 2E.8 (c)] and,hence, . Thus change in resistance due to temperature change is
1 1 1 1 1 given by
-=-+-+-+-
Req·">R, R 2 R 3 R 4 . . .AR = (aR - a)R AT
But as. here, . We find that since a is quite small as compared to a R•
R1 =R~ '=R 3 =R 4 =R, R. 4 = (R/4) we have approximately
(B) In this network one end of each R1 , R 2 and R 3 are AR = a RR AT .. _..
connected to pointO while the other to B, so that R1 , R2 and and therefore, percentage change in R is the same as
R3 are in parallel resulting in a single resistance (R/3) which that in resistivity; changes in dimensions are not important.
is in series with resistance R4 between points O and A as
shown in Fig.2E.8 (d) so that:
1 4
G~'*~~I. 10 ~
R. 4 = R+R= R k lon;·round conductor of cross-sectional area S is made of, 0

,· 3 3 material whose resistivity depends only on a distance r from


the axis of the conducto~ p
, -
=~,
r2
where a is a constant. Find

,the resistance per unit length ·of conductor anq the electric
!field strength due tE._ which a current I flows'in"it.

www.puucho.com
Anurag Mishra Electricity and Magnetism with www.puucho.com

ELECTRIC CURRENT

Solution: ~.,...._:.- 1 ·-=-----=. ---


o O !
I lwha;-~ - ;;,.:C~tage change in its resistance if rad~~
l!!,creased by 1 %and area is increased b}' 1 0/c=-o?_ _ _ ____,
----H---' I
~ dR =-2dA
V
L Fig. 2~~~ ('.l) Solution: ·: R=p-
V

l@,·
A2 R A
We consider a cylindrical shell of radius r As cross-section area A = nr 2
and thickness dr, the entire conductor consists
dA = 2 dr
of such shell in parallel arrangement because i
p.d. across each is same. : I
Cl A r
dR dA dr
pl al I '
1 so -=-2-=-4-
dR=~~- Fig.
'---
2E.10(b)
- - - - R A r
3 )
(2nrdr) 2nr dr
If radius is increased by 1%, the resistance is decre_ased
_! = Jr=a 3
2nr dr = 2na 4 by4%.
R r=O al 4a/ If area is increased by 1%, the resistance is decreased by
or R = 4al4 as S = na 2 2%.
2na The significance of negative sign is that change in
R 2m1 resistance is opposite to that of radius and area. Note that
z=s2 calculus can be employed for small percentage change pnly.
In accordance with Ohm's law. p~%':Qm>~ 13 ~ · .. ' .. ,
E = V =IR =(~Ir= 21taI
1 1 1 s2 ! - ·-·-
IAn electric kettle has two coils. When one coil is switched on,

~~~J~~
it takes 15 minutes to boil water and when the second coil is
---- --- - ----------- - --- -------------- ------ -----
' ~ lswitched on it takes 30 minutes. How long will it take to boil
;water when both the coils are used in (a) series, (b) parallel?
lA portion of length L is cut out of a conical solid wire. The two
Jends of this portion have circular cro~s-sections of radii r1 and Solution: Heat produced in a resistance R in time Us
,r2 (r2 > r1 ). It is connected lengthwise to a circuit and a y2
!current I is flowing in it. The resistivity-of the material of thej H =Pt = - t
R
!wire is p. Calculate the resistance of the considered portion y2
!!!!id the voltag~developed across it. __ ,. _______________ J 1

For coil 1,- · · · H 1 = - (15 x 60) ... (1)


R1
Solution: Consider a y2 '
disc of thickness dx, at and for coil 2, H 2 =-(30x 60) ... (2)
distance x from end AB. R2
' '
Radius ofthe element But according to given proble~.!f 1 =H 2 , i.e.,
(r2 -r1 ) )
=( r1r+~~~x I' 15 30 .'
- = - , i.e:, ·R 2 =2R1
R1 R2 ,
... (3)
L
D,
·.\ (al Both the coils are used in series :
"----L-----"' ;
. _... - .. I
1 Fig. 2E.11 .
i
I
y2
Hs = - - - - t s
'
- - -~--- - --- - --- - _.I (R 1 + R2 )
y2 . . ·.
Resistanc~ ~f this element =p ' · dx 2 =-Xts [asR 2 =' 2R1 ]
- '{r,'+ (r2 ~r1 ) x] But as here,
3R1
Hs =H 1 (=H 2 )
y2 y2
Resistance of the conical wire = JL pdx ~ So, -(15x 60)=-~ts·

.
o{· Cr2 -r,lx] ..
R1 3R1 ,
, .• ' r, + L ts= (45x 60)s =45 min
~ ' I •

=----
pi" ,, . (b) Both the co:P:e[~i:i~J']a:a::el:
m-1 Tz
R1 R2
. • pL
Voltage drop across it = IR = I ---- 3V 2
. ,. 7tr1 r =--Xtp [asR 2 = 2R1 ]
2
2R,

www.puucho.com
Anurag Mishra Electricity and Magnetism with www.puucho.com

'h ----..!1!-1- .• - -·-


'ELE~TRICITY ~ MjlqNEJl~M I
According to given problem, Solution: When a current is passed through a wire,
Hp =H1 its temperature rises with time due to heat produced. Since
3V 2 V2 the fuse wire is exposed to the surroundings it also loses
or - - x t =-x (15x 30) heat mainly due to radiation, heat loss depends upon
2R, P R,
temperature and it increases as temperature rises ..The
or tp = (lOx 60)s ='.10 min temperature attains a steady value when the rate at which
lr~lil~ heat is produced in the wire is equal to the rate at which
heat is lost due to radiation. Let the steady temperature be
IA~;e ofle;gthl.Om and radi~10-3 mis canying-a•heayy equal to the melting point of the wire and l be the current at
current and . is, assumed to radiate as a black ,body. At the melting point. The rate at which heat is produced-in the
~qujlibrium·· its temperature is 900 K while that of the
wire is given by
2
surroundings _is 300 _K. The. resistivity of the.material of the P "'l R
wire at 300K is rr 2 x 10_,, Qm and. its temperature coefficient! where R is the resistance of the wire. The resistance of
of resistance is).8 x 10~3 per °C. Find the current in the;wire. wire is
lg_ejizn's constant - 5.68x 10-"wm-2K-4.. : ,____....J R·=_e_!_
rrr 2
Solution: Resistance of the wire at 300 K is where p = resistivity; 1 = length and r = radius of the
Ro=pL=pL wire. Thus ·
• ,.: A rrr 2 12p 1
i1,.·"''.··'''
2
n x10-~xl.0 rrx 10-2 Q P=- __ ,(l)
. nr2
,, ·1·1J:.·1·,· . .!.. , ,'t .:: ',nx(l0-3}2 ..
Now let h be the rate ofloss of heat per unit surface area
·• •! 'Resistance at 900 K is: of the wire. The rate' of \o~s of heat.from the;wi~e is
,'.· ·' '· Ri =R 0
(1+at) ·. , , :_P'=21trlxh .-.· . . . . (2)
= rrx 10-2[1 + 7.8x 10-3 x (900-300)] In the steady state, P = P'. Therefore equaµng eqns. (1)
= 5.68rr-x 10-2n -. -· : and (2), we have
The rate at which the wire radiates, el).ergy is _, 1 2 l
_P_ =2nrlh
W =P =1 2R, . 1tr2 -

-:: 5.68~x 10-21 2 ~att ... (1) ' 1_=(21t2r3h)1/2


or '
... (3)
The rate at which a black body radiates energy is given
' -' p
,b)[,S,t<tf~'l} law .. ,. _•,.· _ Thus the current at which the melting point is reached is
4 4
.,. ,, . , W.=e_<tAs.CT,.-T 0 ),,-. --,•.·. _·. independent of the length .of the wire, it depends upon the
__ .,. :~he~e As = ~urfa;7.~r_ea of.the wjre =.(~rrr,~L, values ofr, p and h. If the two wires have the same value ofr,
T = 900 K, T0 = 300 K they are made of the same material (p is the same for both
and -cr;,,,5.68xl0-8 Wm-2K°"1·., wires), the rate of loss of heat per unit surface area (h)
Thus W = 5.68x 10-B x (2rrx 10-3)x 1.0[(900) 4 -(300) 4 ] depends upon the material, of the wire and the melting point
= 5.68 x 12.96itwatt ... (2) which is same for both the wires, both fuses will melt at the

r
Equating eqns. (1) and (2), we get · same value of current, irispite of different lengths.
.'c •,', ,_,,_'' '5.6°81t~ 10-2 ~ 1 2 = 5.68,<"f2.96it From eqn. (3), we have
3 2
'' · • • · ·'·-· 1 2 = 1296 (. " .
•.,,. "I," • ' ...... ,,.., . '. ..., "
·-·· · ,, .. ,-, ,_ l =( 2it; h · av,•-;:,
or l =36A·:· -,;; ·

fwa:G)iiilii 15 ~ . ' ··--.-i


or
J2 '21t2h
- = - - = constant
r3 .. p .
l(ciJ _Two wires ,nade ofsame tinn~d cdpper alloy having equal1 Thus
12 12
__!_ = ___±._
ic~o,ss-sectional areqs but-of different lengths.a~~ to be l!,Sed as] .. r3
• 1
r:3
2
!fuses." Show that the -fuses will melt at the same value of,! 3
curre'!t ~hich is independent o/'the lengt1!§ ofth~ wifes.' -
or r2=r1xe:r
(b) A fuse wij:e of radius 0. lO·IJlni blows when a current ofl
lOA passes .through it. What should be the ra'dius of,a fuse • --. Substituting numerical values ,,-."'
wire made of the same material_-wfiii;h will blow at a cµrrent -- ,·'· ' --r = 0.10mm, 1 =lOA'·
~ - ,, 1 ' • .J
1 1
~0if_ 20A
- .? . - ' · ~ - '·' ' . , ".. ,, - ' • ·,· ··and' · 11 ='20 A, we have''',,,·, ' ·
'.

www.puucho.com
Anurag Mishra Electricity and Magnetism with www.puucho.com

ELECTRIC CURRENT 231.


'------------------- -- --
3 I =I 0 e-'' r----- -. ------·- ,
r2 = 0.lOx 20)?/ = 0.10(2)?/3 I I /
( 10 ,_ = ln(2) I I
= 0.159mm = 0.16mm
where
to
! lo '

total charge,
~&'IDllB!!~l}6]~ q=f:Idt
I
:w,i;t-ar~ount of heat ;.,ill be generated in~-c~/zoJ ~~istancel
!R due to a charge q passing through it if the current in the I
!coil: j'
= J: I 0 e-i.tdt =(I~) I
:
-
Fig.2E.16(b)
-- ---- --------.
j
lo·
'
. ,- t :
· · , I,
~

1( a) decreases down to zero uniformly during a time interval


or I 0 =Aq
I I or I = (Aq)e-''
!to ? \
!Cb) decreases down ti)_Eero hal1;il}gj_~!'_C!_l!,le everyt 0 seconds? 1 Heat produced in time interval dt,
I I dH =I 2 Rdt = ,_2 q 2 e-2i.tRdt
I
I lo
ro H = A2 q 2R J: e-2"dt
q 2 AR q 2R ln(2) .•.
=--=~-- ~

2 2t 0
'---~-•t I
I I lo i ONE DIMENSIONAL CONDUCTION
! Fig. 2E.16 (a) 1 iI In this type of conduction, current flows parallel to the
- - - - - - - - - ____-:-=::=.=.=-....:.=;-:::::--..;:.;.-_=----------- - - - - ~
length of conductor. Plane perpendicular to the direction of
Solution: (a) The current decreases unifonnly with flow of current is always an equipotential plane ap.fl the

rn.-m!l)=• • [I:!
time, therefore I vs · t curve is a straight line as shown in Fig. decrease in potential per unit distance is maximum in the
2E.16(a) with slope m = -I 0 /t 0 • Current as function of time
can be written as
I=I 0 -G:}. (y=-mx+c) ... (1)

Area under I vs ·t graph gives the flow of charge q,


therefore ''
1 1~-_._ _ _ _ !')g.~_,~6 I
q =-CtolCio)
2 In one dimensional conduction the equipotential
2q surfaces are simple planes and parallel to each other.
Io=-
to Resistance of the tube shown hi ihe'figure betWeen the
Substituting in eqn. (1), we get · • . ' lI' I '

two circular faces is given by (using-R -= !:..)


.·., ·.
I - Zq(1-_E_J
to ·to . A
R=
!
pl ,

or I =(2qto - 2to;,t_-J '·.


,r(b2 -a2) -
where p js the resistivity which is_ uniform throughout.
Heat produced in a time interval dt is Resistance of a Conical Conductor Between its
dH=I 2Rdt . Two Circular Faces
Since the area of the
=(2q - 2;_t)Rdt equipotential surface (area of
to to cross-section) varies as we
or
0
J
H = to (2q -. 2qtJR dt
t 0 l:Q
,2
move from one. face to another,
so to calculate the resistance,
let us consider .a co-axial disc
2
4q R of radius 'r' and thickness dx at I
•. !
=---
3 t0 a distance ' x' from one end as
Fig. 2.47
-~
(b) Here, current decreases from I O to zero shown in the figure.
exponentialJy::,vith half-life ,of t 0 • The I-t equation in this Radius of the disc,
case is an exponential function like the radioactive decay r::;::;a+nx
law where 'n' is the increament in radius per unit length.

www.puucho.com
Anurag Mishra Electricity and Magnetism with www.puucho.com

1232 ELECTRICITY & MAGNETISM j


(b-a)
i.e., n=--
1
' '
The cross-section area and length of a cylindrical conductor,
The resistance of the element disc, are A and ~ respectively. The specific conductivity varies as j
dR=pdx
nr2 cr(x) = cr 0 -~., where x is the' distance along the axis of the!
vX I
Since all such discs are in series therefore the net cylinder from one of its ends [ see Fig. 2E.17 (a)]. '1

r~sistance of the conductor, (a) Compute the resistance of the system along the cylindrical
pdx pfl dx axis. I
R =Jdr =Jnr2 =-;;: o (a+ nx)2 (b) Compute the current density if the potential drop alongi
__ .!_p_(_l _.!.) the cylinder is V0 • What is the electric field at each point in the
1
nit a+nl a cylinder in, the case described?
--~------· - -- _.. _,_ -·----- - - - - ·- --- ·- ·- -- ,'
p nl pl Solution: (a) Consider the
7t a(a+ nl)n na(a+ b-a) cylinder as composed of thin discs of
R=__e!__ width dx connected in series [see
nab Fig. 2E. l 7 (a)]. The resistance of a
disc at a distance x away from the
Tow dimensional conduction (Cylindrical
cylinder end is :
Conduction), Resistance Between Inner and Outer
_ 1 dx _ -./xdx
Surface dR ------- ... (l) I Fig. 2E.17 (a) I
cr(x) A Alcr 0
'--·-·--·- ·-·- "

dx where A is the cross-section area of the disc and dx is its


C r- width. Since the discs are connected in series, the total

I end·view
resistance is

(b) From Ohm's


R =r
law,
o
we
dR

deduce that the current flowing across


= _l_ r -rx
Alcr 0 o
dx = 2.J[
3Acr 0
... (2)

Fig. 2.48 the cylinder is given by


To calculate the resistance let us consider an elemental I= VoR = 3AJro
2 1·,_
....(3)
concentric cyli!}drical shell of radius 'x' and thickness 'dx'.
The resistance of this elemental shell is given by, The current density is, therefore;
'
dR=-
·-pdx J =~ = 3;1zo ... (4) __ Fig, 2E.17 (b) ___ 1

2nxl
Since all such elemental shells are in series, therefore The electric field in the cylinder may be found by using
the net resistance of the conductor is, Ohm's law: ·
R =I dR =Jb pdx
a 21txl
=> R =..E__
2nl ~\a
,J!:) ... (5)

Equipotential Surfaces are Spherical


To calculate the resistance r----:::::=::;::----7
between t:Iie inner and outer surface,
let us consider a spherical shell of ------ -"---·-··--·--,
radius 'x' and thickness 'dx'. The space between two coaxial cylinders, whose radii are ai
and b (where a< b as in Fig. 2E.18), is filled with al
dR = pdx conducting medium. The specific conductivity of the medium!I
47tX2 . .
is·q.·
R= dR f ( a) Compute the resistance between the cylinders in the radial j
direction, using two distinct methods,Assume that the I
= :"s::.
R =1:_ (b-a)
Fig.2.49
cylinders are very long compared to their radi~ Le,, L » b,:
where L is the length of the cylinaers. :
4n ab (b) Calculate the resistance, assuming crvaries as cr(r)
________ .,r
= " 0• ;
I

www.puucho.com
Anurag Mishra Electricity and Magnetism with www.puucho.com

ELECTRIC CURRENT 233

I
•A conducting medium is shaped in the form of a quarter of an i'
annulus of radii b and a (b > a) and thickness h. The1·
L, m. edium's specific conductivity is cr. Compute the resistanc~
' between the three pairs of opposite faces in the f, z and 0

Fig. 2E.18
directions.
'------------------

Solution: (a) Calculate


for the z direction : In this
case, the electric current density
r, -·. -~--- - --- - ·
------------- --·------ ----- -
1

··--------·-=-s-=-·-=~•~----=~-------------
is z-independent; J = Jz, where
8
....-----····:;;· \
Solution: (a) From Ohm's law; we have 1
--> -->
... (1)
J = constant. The cross-section ::..---,,,....,_-....:..~J.h 1
;
J=crE } I '.

--> area is A= -:t(b -a


2 2
) and the I fI
Assuming radial current densit}, J becomes , Fig. 2E.19 (a) I
width of the medium through L_ , -- --- - -- ------·
J=- -r
2rrrL
1
for a<r<b ... (2) which the current passes is l = t. Therefore,
and, therefore, R • =.!.~ = 1
£ h 4h
2
(1) ·, •••

--> I • aA a_!_rr(b,-a2) na._(b 2 -a- ) .·, ,_,·,·-.


E=--r . .. (3)
2rrcrrL 4 ... -·~ ' ' • • ( I•

--> (b) r direction : In this case, the radial current


Here we have used the assumption that L >> b so that E
-->
. 1s.
dens1ty =- - r, where - - - - - - --- --- - - ;
. -->JI.
and J are in cylindrically symmetric form. The potential A(r)
drop across the medium is thus : A(r) is the cross,section area
at radius r through which the
Vab =-J a--> -->
E(r)-dr=--- I Jadr
-=-- I 1{b)
- ... (4)
b 2rraL b r 2rrcrL a current flows.Therefore,
1 ·:1f
The resistance is clearly: A(r) =-2nrh; we divide the , h I
4 ::t
R - vab - In(~) .
conductor of Fig. 2E.19 (b) dr i
ab - -I- - 2,mL ·
... (5) into radial slices of width dr, '-----·-- Fig. 2E.19 (b) _____ ,
as can be seen in, figure. The _, ,
Method 2 : We split the medium into differential area of such a slice ·is A =.!. ·2rrrh, and the width through ,
cylindrical shell elements, of width dr, in series. The current . 4 _ .·,,_, ·.r'1
flow is cylindrically symmetric (L >> b). The area through which the current passes is l = dr. ·Thus, the resistance of a
which the current flows across a shell of radius r is slice of radius r is , _; ' ,'",
A(r) = 2nrL. The length the current flows, passing through a 1 z 1 dr· -
dR =--=-___:..._a., .. ,·.' .... (2)
shell of radius r is dr. Therefore, the resistance of the shell of aA crnh!:... i ,

radius r is : i.
... (6) These slices. are connected in a series; since all the
current flowing o;,t of the slice at r enters the adjacent slice
Since the shells are connected in a series, we have : ofr+dr. Thus, ·. - ·.. ·,,,>,·,. · :1 ~,-,

~ i{~)
R,
= fb dR = __2_fb dr, = -~' 1J.!>.)(3) ...
R =fb dR a rrah a r 1tcrh 1 ' \ a
... (7)
ab a 2mrL (c) fl direction : In this r------------------:
(b) Using cr(r) = cro, we have from eqn. (6),
_, .
case J = J0, i.e., the current flow
I ,
r .. ' . lines are azimuthal. We split the
=---- ~ ... (·8;.:
dr ' dr · conductor into'radial slices. The
dR=
2rrrLcr(r) 2,rL'. r _,_cr_o 2rrLcr 0 • , •• current flows as· shown in the
r · ,' , figure. The slices are connected in
... (9)
parallel. The cross-section area of I I
Thus,
each slice is h dr, and the length I Jda
along which the current flows in iL _ _ _ _ Fig. 2E.19 (c)
_ - - - - - - - --.
:
. f d" · 1 2 rrr
t h es I1ceo ra 1usr1s-- rrr =-.
4 2
www.puucho.com
Anurag Mishra Electricity and Magnetism with www.puucho.com

~34 ELECTRICITY&_MAGNETISM I
Therefore,
a hdr
dR
r
1t-
=.!__1_ ... (4)
I,
I
is independent of
p does not dependent on E
E _. .,
Since the slices are connected in parallel, the equivalent for materials that obey ohm's law , is unchanged: when
resistance is given .by . I
!field is applied particular temp.--'e'-'ra"'tu=re"-._ _ _ _ _ _ __.
_1_ = J b _!_ =J b2ah dr = 2ah 1
Req. adR a nr 1t a
{7!_)
(S) ... KIRCHHOFF'S LAWS FOR
CIRCUIT ANALYSIS
or, R- =--"-- ... (6) Before moving on to the statement of Kirchhoffs law; we
• 2ahl{~) state some conventions to be followed in circuit analysis:
-----·--- _____ ___.., ---, (1) Direction of conventional current is from high
potential to low potential terminal.
Concepts: 1. The resistivity or conductivity is
independent of the magnitude and direction of electricfi.eld. (2) Current flows from high potential node A to low
potential node B. If we traverse from point A to B, there is
2. Resistance B is characteristic property of object. R is
drop of potential; similarly from B to A, there is gain of
independent of potential difference. ·
potential.
· 3. · AV, I .and R are macroscopic quantities applying to a
body or region. The corresponding macroscopic properties are If a source of emf is traversed from negative to positive
""7' ~ ',- . ' ' terminal, the change in potential is +E.
!E, 'J and p or a they have values at every, j,oi!'t in .a body. V .
:!:, .: . rResistivity, of a· metal can .be increased by oJ ra~ing
Potential gain
A - VB =+E
High Potential Low Potential t ' t
}its_ t~rrperature (2) a~ding ~mall amounts of impurities and A B Final _ . Initial
potential potential
(3) increase lattice imperfections by doping. Potential drop
· , , .5. ,AV, I, R are macroscopic quantities applying to a
.,par,tjcular body (!r extended region.. , , Nate that -• while ~--- ·---~'-"'·-----
. ., ... . ., discharging, current is A•
I
• +1 -
• •

I · 6; E, J and a are corresponding microscopic quantities ·drawn from the battery, the
B

· 'ithej hdveivalues at ',;i,ery point of a body. · current comes out - fioin


Dischargln9 Battery
' ..
I 7. Temperature variation of resistivity. positive terminal', · a:nd ·+1 ~ - L
f , ' • 1 dp .·eniers negative· terminal, A••-•----"I- , ' . B

! a =p_dt . . .·: . . . I while charging of battery


·. current is forced .from
Charging .a Battery

' 8. bhm's law is not a, fundr;11Jtentql law ·ofi


e(ectromagnetism ·beca~e It depends on ,properti~S of the . ·positive terminal •of the
, battery. to · negative
High Potential ' ; ~ Low Potential
I\, .',1 , ,
.
B I
conducting medium. : ,. , <, ., _,_, _.
terminal. Irrespective·, of Fig.,._?,-5.9c....-._ _ _~
/ : (p = const. fbr same mate,:ial not dependent· on electric
direction of current ,through a battery the sign convention
ifir1d.J . · --- -: . mentioned above holds.
··· 9. Electrons move with a uniform .average speed is resultl
of quantum mechanics. i
The positive plate of a capacitor is at high potential and
negative plate at low potential. If we traverse a capacitor
The effective speed obtained_. from. · the quantum from positive plate to negative plate, the change in potential
. distribution is nearly independent of temperature. is -Q/C.
'1 O. Collisio'n.s-bet:ween electrons themselves are rare do
Q
VB - VA : : : : ; - -
not effect the electrical properties of material.- ,
.t '·t C
11.· Energy-ii tran!;ferred·from the applied field to the High Low'
lattice [in from of internal energy of the conductor, often potential potential
observed as a temperature increase]. '
If we traverse a resistor in the direction of current, the change
12. Drift speed is different from speed at which changes in potential is -IR
in the electric fielfl, cgnfif';uration travels qlong_ wire.
..., ..., 1:'s. ,- VA = -IR
.., I•' •:
.j; •. ·-1-
13. J = aE is valid only for isotopic materials. whose ' ' Final ' I ~ ~ li:tltial
electric properties are- same in all directions. ' potential potential

· ' 14. In case of 'ohmic materials '· the· resistivity or , , . If we tranSVer~e, a ..resistor in the directio_n:,c_>pposite to the
' conductivity of a' material ~ indepenile'nt of'ihe magnitude ldirection of current, the change in potential is.,.+IR. 1 ,
and direction of the ·applied field. · VA,-. v. =+IR
.j,' ;j,
1_ _ [' ~ charact'ei-j/tic "of material. __ ' ·.' ----c----' Final• , 'Initial
potential potential

www.puucho.com
Anurag Mishra Electricity and Magnetism with www.puucho.com

I,ELECTRIC CURREfU 235]


If a capacitor is traversed from negative plate to positive
:g
I=--
plate, the change in potential is +Q/C. r+R.
,-,-·+-=--~ In the above analysis we chose to traverse the loop
VA - VB =+g 1
I I _1
-_ clockwise, but that choice is arbitrary. Suppose we traverse

High
t
Low
t C '7A B1'! the loop counterclockwise, beginning at point a. The loop
rule gives '
potential potential '
1
Fig. 2.51 (Vd -V.) + (V, -Vd) + (Vb -V;) +(Va-Vb)= 0
------- - --1
The potential difference (V, - v.)
is +IR because V, > Vd.
KIRCHHOFF'S RULES Also, v. - Vb = -{Vb - v.) = -{ff! -Ir) = -ff! + Ir. This gives
Two rules, called Kirchhoffs rules and named for G.R. IR-E +Ir= 0
Kirchhoff (1824-1887), guide us in finding the currents. Solving for I we have
These rules are referred to as the .Joop rule and the jun~tion I=--
ff!
rule. r+R
The Loop Rule This is the same result .as before. The answer is
The loop rule states: The sum of the potential independent of which way we go around the loop.
differences encountered in a round-ttip around any closed There are two rules we can use to give the algebraic sign
loop in a circuit is zero. of terms we enter into the loop-rule equation:
---·· --~--------~------------ 1. In traversing a resistance R along the sense of the
'
, Concept: Since the potential is directly related to thef current I, the potential difference across the resistance is
potential energy of the carriers, the loop rule is a statement of, entered as -JR. In traversing a resistance R opposite the
,conservation of energy. We can write the loop rule as I sense of the current I, the potential difference is entered as
___________________ ~V=O _.· ____________ _I +IR.
2. In traversing a source of emf along the sense· of the
As we consider the potential in going around a loop in a
emf, the potential difference across the source is entered as
circuit, the potential increa~es through some elements and
+E. In traversing a source of emf.opposite the sense of the
decreases ·through .others; the sum of the potential
emf, the potential difference across the source is entered as
differences for a complete round-ttip is.zero. -ff!. .
By convention, I has a positive value when the sense of
In using these rules, we treat the internal resistance of
the current corresponds to the, ·direction of motion of

i.;:31 :
the source as a separate resistance.
positive carriers. Now consider applving -,---- ----r-·· - - ------,
1 1
Consider using the loop rule to fihd the current in •the J· \ C ~ --~ b 1

circuit of Fig. 2.52 (a). The sense of the current I is shown·in


the figure. We begin at point a and traverse the loop in the ~~,,.:::: a:!, sh':."":: !'~ i
Fig. 2.52 (b). Since the senses _R2, . R, ·f1 1

clockwise sense. The loop rule gives ,· ,


;,-"":"'b·-- I_.. --,: -:-c7· of the emfs ·of the two I·· ' • · r, 1'
1•
batteries are opposite · one !'d . l..:, _ _J - _a :
I another, we are . not certain. ; ' .. · · £2- - ]
.'..,•
- ,,1
i r ,·R,
about the sense of I. Let us L I
___ __ Fig.2.52{b) _______ j'
I assume that the sense of I is ,
,-t..
I E counterclockwise, as shown in the figure. Starting at point a
' and going counterclockwise around the loop, we write the
a ...:-1 d sum of the potential difference as ·
Using the.loop rule (IR 1 ) + (+E1 ) + (-Ir1 )) + (-IR 2 ) + (-Ir2 ) + (-E2 ) = 0
___ Fig.2.52.(a) __ ____ i
Solving for the current I, we have
(Vb-Va)+(V,-Vb)+(Va -V,)+(Va -Vd)=O I= E1 ~s,
The potential difference across the section from a to b is R1 +R2 +r1 +r2
the terminal potential difference across the battery:
Notice that eq. (1) yields a positive value for I if E1 > E2
Vb - v. = ff! -Ir. The . connecting wires have negligible
and a negative value for I if E 2 > E1 . If E 2 > E 1, then the
resistance, so the potential difference V, - Vb and v. - Vd are
sense of the current is clockwise, opposite our assumed
each zero. '
sense for I in Fig. 2.52 (b). Therefore, if we assume a
The current through R is from ·c and d so that V, > Vd. particular sense for the current at the _outset of a problem
Therefore V{- V, = -IR. -·' and the value of the current turns O\lt to be negative, then
Substitution into the loop rule gives the actual sense of the current is opposite the assumed
(E-Ir) + (0) + (-IR)+ (OJ= 0 sense. That is, the equation automatically gives the sense of
Solving for I, we have the current.

www.puucho.com
Anurag Mishra Electricity and Magnetism with www.puucho.com

1236 ELECTRICITY & MAGNETISM' I


We can assume the current has a particular sense, and if from a to e to f to b. Also, for simplicity we include the
this assumption turns out to be wrong, then I has a negative internal resistance of each battery in the resistance that is in
value. In the case of Fig. 2.52 (b) the actual sense of the series with that battery: For example, the internal resistance
current is counterclockwise, the same as our assumed sense. of battery 1 is included in R1 .
The loop rule applied to loop 1 beginning at point a and
going around counterclockwise gives
(-I2R 2 )+ (-E2)+ (-I 1R1 ) + (E1) = 0
4 V, V
Rearranging, we have
E1 -E2 =I1R1 +I2R2 ... (3)
3
The loop .rule applied to loop 2 beginning at point a and
~ l---V, J_~~--~ V
going around counterclockwise gives
(E3)+ (-l3R3) +CE2) + CI 2 R 2) = O
a Ve- Vb d"j" ,' Rearranging, we have
01-a...,..,____+-+----+---'-'e------->-..•,-....1
, C d E3 +E2 =l3R3 -I2R2 ... (4)
We have three equations in three unknowns. The three
i-2 equations are the point-rule equation (2) and the loop-rule
! equations (3 and 4). The three unknowns are the currents
L___ Fig.2.53 ____ --·--· ··-·-- I 1 ,I 2 and J 3 • The point-rule equation is simpler than the
. · ··Fig. 2.53 shows a graph of the loop rule for the circuit in other two because each of the coefficient is 1. Using the
Fig. 2.52(b). In our mind's we break the circuit at point a point rule equation to eliminate I 1 in Eq. (3) gives
and· 'string it along· a straight line. Then we show the E1 -E2 =I2(R1 +R 2)+I 3R1 ... (5)
variation of the potential along the sense of the current. The Equations 4 and 5 represent two equations in two
potential of point a has been arbitrarily set to zero (that is, unknowns. Substitution of the numerical values from Fig.
point a i_s "grounded"). 2.54 into these equatiqns gives.
The Junction Rule COMBINATION OF CELLS,
The sum of the currents toward a junction point is equal Series Combination
to the sum of the currents away from the same point. If the negative ·-·:;1 e2 e3............. En
Points a and b in Fig. 2.54 are examples of points. Since terminal of a cell is ·f-W\.-J WV'vj f--vw--+
charge does not accumulate at any: ' point along the connected to the p·ositive r1 r2 r3 ............. rn

=J
1
1
connecting wires, the point rule is simply.a statement of the· terminal of the· second'
conservation of charge.'.The point rul~ can.be written as and the negative
TI towa<d "' TI away ... (1) terminal of the second to . __ _R_ _ _F',_,,9c,•2..c'c.55
For example, the point rule appli,ed .to point a in the positive terminal of the
circuit in Fig. 2.54 gives . . , third and so on then the cells are said to be in series.
,-. ,_Ii ."'I2 +I3 , .· ... (2) Applying Kirchhoffs law, we get
because current I 1 is toward a and current! 2 and I 3 are
away from a.
I . E1 +·E2+... En
r1 + r2 + r3 ... rn +R
r h
L:__J
We now. use the loop rule and the point rule to find the and equivalent emf.
currents Ii, I 2 and I 3 .in the circuit of Fig. 2.54. Let loop Eeq =E1 + E2 +E3+... En R
abcda be loop 1 and loop aefba be loop 2. From the figure, and r eq = r 1 + r2 + r3 +··· r n
1
Fig,~·=2-~5~6-~
our assumed sense' for J1 is chosen to be from b to c to d to a <, •• If
E1 =&; =&3 =E4 .. ,=En =E(sqy)'

C
- 1,
b
-
13 and
then
r1 =r2 =r3 =r4 ... =rn =r(say) '.· ·
I=....!!£_

·i
R, R3 nr+R

e,I_ R2 e;I. req = nr,


.'
'· case-1: ·
J12
I d
- - 1,
__ _f!g. 2:54
a
13
'.'
If
then
R »> nr
.·. nE
I,=R-
, ' .
i.e., current the r;ir;cuit is 'n' times the_ cuff!=nt due to a
(counterclockwise); our assumed sense for I 2 is chosen to be single cell. · ·
from a to b; and our assumed sense for I 3 is chosen to be
www.puucho.com
Anurag Mishra Electricity and Magnetism with www.puucho.com

ELECTRIC CURRENT 237

case-2: Equivalent emf


If R <<<nr i . D 11 r1 11 C ,

then l=n& =& &1


nr r B!
i.e., current in the circuit is same as due to a single cell. 12 '2 12 :
Thus when internal resistance is negligible in
&2
1:
comparison at the external resistance then cells are
connected in series to get maximum current. !M ~----P.AAl'---~IN.
R
Parallel Combination i
1_________ Flg.2.59_(a) __________ ,
&r
From Kirchhoffs first law at any junction,
I =I, +I 2 ... (1)
From second law for the loop DAMNBCD,
&1 -IR -I1 r1 = 0
- &, -IR ... (2)
I1 -
r,
Similarly from the loop AMNBA.
_&2-IR ... (3)
R I2-
-~g, 2.57_ _J r2
In parallel connection of cells aJl the positive terminals Putting the values of I 1 and I 2 from (ii) and (iii) . in
are connected together at one point and all the negative equation (i), we get
terminals at another point.
Let there be 'm' branches with one cell each then
I=(&,+ &2
r1
J-1il 2-+_!_J ,-- ----- -.- - - ·1
rz "\ r1 rz
l1C:J&,:•q. 1
applying Kirchhoffs law in any one lo'op.'·
I (1 1) =(&1- +&1J-
I+IR-+- r ,

'"l'C:J.
-IR - - r + & = 0 · ;- -- · -,,;;· -- - r1 r2 r1 Tz ' '
i .R. - i
I= & :1 &,, ·11 1
1[1 +ii_!_+ 2-JJ=(& + &2J,
+ -r) .·,
___ Fig0 2.59_(b)_
m .. \r1 rz r1 rz
I'
R
(
m I -
.. :· .
I
I
R I 1
r . '' I Fig.2.58 ·I [
1. 1 )"
IR+ -+-·
( r1 rz
'] =(& &
_ I +2 "J(--'-+-
1 l J-l .:.(4)
req = - . _,:---- - --- - ----~,l r1 Tz r1 Tz
m
I'= &,q Now from the equivalent'circuit; ·
R+r,q I(R +r,q) =&,q - ... (5)

rl
Comparing equations (4) and (5), we get
I = I' so &,q =&
Case-1:. 1
If R »> r/m,
, r,q =(:, +:, =r:: r2
&
then l=- and . &,q =(&1 +·8:2J('t'+_!_J-1
R
r1 Tz r1 . r2
i.e., the ~un;ep.t in the circuit is equal to the current due
to a single celL , · ·• &1r2 + &2r1
' -
Case-2: T1 + Tz
If R <«r/m In general for ' n' cells,
-1
l=m&
then
r &,q = (-+-+-...+&n-
&1 &2 &3
r1 r2 r3 rn
1
J(-+-+-...+-J
r1
1
r2 r3
1 1
Tn
'
i.e., the current in the circuit is 'm' times the current
produced by a single cell.
Hence when external resistance is negligible in
=(~!)(~~r,
comprising to the internal resistance (r/m) then the cells are Mixed Combination of Cells
connected in parallel to get maximum current. In the circuit N similar cells each of emf E and internal
If the cell·s have different emf, then the current in the
circuit is solved by using kirchhoffs two laws. · '
www.puucho.com
Anurag Mishra Electricity and Magnetism with www.puucho.com

[238 · ,,,:.i~-·'---------'-----
-" '.-,, -,,.,, 'ELECTRICllY &IMQNETISl.f I
resistance 'r' are connected as shown in figure. The Net Work Analysis
arrangement consists of 'm' rows of cells, the rows being The Kirchhoffs ·current law r---;:-=-=,w,;-=-=--=-:c··=·-o=-=-=-
(KCL) states that the algebraic sum I
of the currents e.ntering the
junction must equal the sum of the
currents leaving the junction. From
the standpoint of phy~i~s, KCL is a i
statement of charge conservation. !··'----'-~---~
The KCL applied to junction O i;____ Fig. 2 ._,,6,,_2
• __
1/m yields
I 1 +I 6 +I 3 = I 2 +I 4
'-----,-----' ~
R. Outgoingcurrent Incomingcurrent
Fig. ~.60 The Kirchhoffs law (KVL) sates that the algebraic sum of
the potential difference around any closed loop of an electric
coiui.ected in parallel each row has 'n' cells connected in circuit is zero, The KVL is ·a statement of conservation of
series. Applying Kirchhoffs law-in any of the loops, we get, energy, The KVL reflects that electric force is conservative,
I=~ the work done by a conservative force on a charge taken
't nr
·
R+- around a closed pat!, is zero, .
m .-we can move clockwise or anticlockwise, it will make
. E
no difference because the overall sum of the potential
I". =~ I =.I' difference is zero .
. . . R+ nr.
. ·m. .-we can start from any point on the loop, we just have
Therefore z.4 • = nE; to finish at the same point.
n· .-An ideal batrery is modelled by an , independent
req =-r voltage source of enif E and an internal resistance r as
11! , shown in Fig. 2.6!1. A real battery always absorbs
For current to be maidn:ium; power when there .is a current through it, thereby
dI .
-=0 . offering resista_nc~ .to. flow of current.
dn Applying KVL arqupd, the single loop .in .anticlockwise
d ( mnE )·~ 0 direction, starting from point A, we have
+IR + Ir E =0

d: [~Enr
dn Nr
i· ..
=O. Hence,
in tire · · ·
opposite direction
to current
In the opposite
direction to
current
I=--
S
From positive
to negative
terminal.
· ·.
~ .

-+nr . R+r
n . , Fig. 2.63 shows variation of potential in ,j closed loop.
. ' ' '
R=nr ,-
'. . ,' 0---1-~'v--v'---H-+-<>-1-l.
, '! 'z ---:-:-:-:-: ;:--
m-
I =nE=mE 1
1
,· E1 ~ , ',.,. !·
max 2ft, 2r
',
!
....
''
Powe~ Transferre'd to the Load ,·
2

. P=[·: -~~] R
R+-.
m
.. For P to be maximum, ·
nr · •' • J,
R =- (by maximum power transfer theorem)
m
n 2 S2 '
m 2 E2R'
pmax=.-2R. . . •42·
r
,.
.J. )'' , ,Ve ;-_,
Hence in mixed iombi~~ti~n of cells, the current in the ! v, .......... ,..IR...,.t
~_,,__, .
circuit and power dissipated in the load resistance are .. ,.,._l!2..,. , _. iri,--;-, ,. , ,•
maximum for the same conditjon. Therefore it is preferred
to series or parallel combinatioQ, • . (~)
L-~------'-'Flg.2.~~3-------~
www.puucho.com
Anurag Mishra Electricity and Magnetism with www.puucho.com

ELECTRIC CURRENT 239 I


.-Maximum power transfer theorem where Vb - Va is the terminal potential difference across
The power absorbed by load resistor R is the battery.
---- b---- - Cl
r---, ---: •
I
We can traverse the circuit along path 2 to obtain
• r 1
I Real~ R' Vb - Va, we must: get the same potential difference because
' .: II
: Battery potential difference is independent ;f jath followed.
i
I :& :
·----·-· '
I Vb-Va=+IR=(-- R

l___,,f,::~:J
--.--, R+r
path2
.-Toe first step in analyzing a circuit is to simplify it to
simplest equivalent configuration. In Fig. 2.67, each
of the circuits has the- same equivalent resistance
between A andB, in each version,_Ri,R 2 andR 3 are in,
P =I 2 R =(__£_). R;
2
parallel. ·
R+r
For maximum power transfer we take the derivative of P
R, I
w.r.t. R, set it equal to zero and solve the equation for R. R R
dP -,
-=0
dR R R
2
dP = 82 (R + r~ . :-~_[2(R + r)J = O
dR (R,tr) 2 ,
Solving for R, we have
A 8 A 8 A ~ -: . '
(a) (b) (c)
(R + r) 2 -R(2)(R+ r) =·0• · .'
R3 R
(R+r)-2R=-0-,'
R ='r' ',,',' ·,
.. - !'
R2
For a given real battery the load 'resistance maximizes . I
the power if- 'it is equal to the internal' resistance of the .
battery. · -' - ..R, -
,-;- -:~-'-c---1 A
'8 A 8
.-;

I
[
P (R) =('R+
_§_)r R !I 2
(d) (e) '.
____________________ Fig. 2.67 ____ ._, , _____ ,
I I
R R
.-All the resistors in Fig. 2.6_8 (a) are· in parallel
I ·',\" arrangement.
Io r • All the resistors in Fig. 2.68 (b) are in series
I
I J · I''-- --,-•Fig. 2.66
:
1
- - - - - - --- J
arrangement.
-----
___
- - - - -·-- - - - - - - - · - - ~ -·---- ,_ ' - - - - - - - - - - - - 1

The maximum power transfer theorem is general, it


holds for any rea!'voltage source. The resistance R may be a ·!·,
single resistor or R may be the equivalent resistance of a = !

collection of resistors. _____ _


.-What value of R maximizes the power absorbed by all
the resistors?
The total equivalent resistance connected to battery
must be equal.to the internal resistance of the real battery.
a
.-we assign- potential Va to th'e·pciint a and traverse to (a) • -.· '(b)

point b. ,1.. _ _ _ _ _ -
Fig. 2.68
____ -.- 1. _ _ _ , ,_ _ . ---.--·- I
Va +S-Ir=Vb .-Fig. 2.69 shows several circuits iri which the circuit
When we tiaclt the junction b, we ~ust be at potential elements are neither in series nor in parallel.
- . .' ' . ' '

Thus we have · ,, '


,.I

•.· ' '.,

www.puucho.com
Anurag Mishra Electricity and Magnetism with www.puucho.com

1240
L I- - - - - - - - - - - - - - ---- - - -

Follows Fig. 2.72 (a), (b), (c) and (d) to arrive at


equivalent circuit. The equivalent resistance is 3 kn.

s·~-
'........... .. ···F:
+ -
+
'··rkfl··········-· lSeries
..:,:_

,•. ;· -,
' '
(a) (b) (c) 1kn: ! !1 kn 1 kn :1 kn
:' :' 2kn 2kn
: : 1 kn
!o .· .... .-.-.-.-.-.-:
.......... 1 kn ..
~l'--+-vv"'-<~.--'=---"G
·------ . -------------·
Series (a) (b)

(d) (e)
.•.•.. /P~_rallel E
Fig. 2.69

,.. If the same current passes through every resistor in a


given branch [Fig. 2.70], irrespective of the presence 2kn
3kn 12V
of sources in that branch, the resistors are in series 2kn: 2kn C
even though they are not directly connected to each 12V
other. Same is true about capacitors. L-----eG
Equivalent circuit
I - ·--- ------·· G
•A"R~
,
--=-,,,.:LJ ~i..=..__,.,:LJ
1 V1
- R - ~"R~ -
2 V2 3 V3 -
1/ A
~ V1-V2+V3
~ • • "':._____.._I 1-+--<l B '
...,--------.,-vv.....--------,: -
R 1 +R 2 +R3 +R
(c)
Fig. 2.72
(d)

4
' R,
' +
(b) Since V =IR,q.
B
(a) (bl 1=~=4mA
Fig. 2,70
R,q.
(c) Start at point G, assign it a potential V0 ,
.-In the given circuit [Fig. 2.71] we will determine (a)
proceed toward E along any path. When you reach point E
the equivalent resistance between C and G, (b) the
after adding potential drops and gains you get potential of E.
current provided by the source, (c) voltage across
points G and E. Vo+ 12-IR = VE
E F VE -Vo .=12-JR
= 12-(4x 10·3 x 2x 10 3 )
1.0 kn
=4V
A B ,.. In the given figure we
a I R 1 =sn

1.0 kn
1.0kn

1.0 kn 1.0 kn 1.0 kn'


wish to determine (a) the
current! in the circuit, (b)
the potential at each of
the labelled points r1:= 10
+ -
- -
R2 =sn
b
+

a, b, c, d, e, f assuming that -fg: e2 =4V_


1.0 kn + :
the potential at f is zero, :e1 = 12V
D .• ___: r = 10:

-
C (c) power input and 2

1.0 kn 12V?:<- output in the circuit. t


f ....---'W\.-+--"-41
H G (a) First we assume R3 =4n
that current is clockwise. Now
~ig. 2.71 Fig. 2.73
we apply KVL in the assumed
(a) Imagine the wires to be flexible and lift up the direction of current.
inside square with the resistors and source attached.

www.puucho.com
Anurag Mishra Electricity and Magnetism with www.puucho.com

r - . -.
I ELECTRIC CURRENT
--- - -~

Circuit element Charges in potential Sign Note that battery E 1 is discharging, the current comes
R1 (a---t b) Drop IR 1 minus out of its positive terminal. The terminal voltage across its
terminals
R 2 (b---> c) Drop IR 2 minus
E 2 (c---t d) Drop E2 minus = &1 -Ir1
r2 (d---> e) Drop Ir2 minus = 12-(0.5 x 1) = 11.5V
R 3 (e---> f) Drop/R 3 minus The battery &2 is charging, the current goes in from its
positive terminal. The terminal voltage across it
E, (f---> g) Drop E1 plus
= &2 + Ir2 = 4+ (0.5)(1) = 4.5V. The terminal. voltage is
r, (g---> a) Drop Ir, minus greater than the emf of the battery.
Always remember the signs of emf are independent of the ..-When analyzing a circuit, study it carefully. Try to
current. guess is there anything extraordinary in the circuit? Is
Hence we have there any shortcut? Is the circuit symmetrical in such
a way that some sort of bridge is present so that the
-IR1 -IR 2 -&2 -Jr2 -IR 3 + &1 -Ir1 = 0
elements might be redundant. Take a look at the
I &, -&2 following illustrations.
R1 +R 2 +R 3 +R4 +Rs ..-In the circuit shown in Fig. 2.75 we wish to determine
Note that if & 2 is greater than &1 , we get a negative current and potential
value of I, which shows that assumed direction of current is R1 =6.on R2 =4,on
drop on resistor R1 . ~-Vv<----Wl-~e
wrong. Note that points
a, h,g and f have g~.-,,,-,.,,.-~-="-~d
I _ _1_2_-_4__ = 0.5 A 17 n 21
5+5+4+1+1 same potential, they n
are connected by 2 0
· n
(b) Now we determine the potential at each
labelled point in the circuit. conducting wires =12v
V, =Vr +&1 =0+12=12V without any circuit 6.0V T 6_.0V
Va = V, -Ir1 = 12-(0.5)(1) = 11.5V elements between h H~H C
Vb =Va -IR1 =ll.5-(0.5)(5)=9V
them. Similarly
12V
points b, c,,d and e
V, = Vb -IR 2 = 9-(0.5)(5) = 6.5V .L.---o--1 H ...-----'b
have same potential.
Vd =V, +&2 =6.5-4=2.5V Hence the potential Fig. 2.75
V, = Vd -Ir2 = 2.5-(0.5)(1) = 2.0V drop across branch e
Vr = V, -IR 3 = 2.0 - (0.5)( 4) = 0 and f, and a and bis same. The two resistors (6Q and
Note that we have assigned f to be at zero potential, we 4Q in series) are directly connected across the
can choose any point of the circuit to be at zero potential terminals of 12 V battery.
and then determine the potentials of the other points The complex circuitry in the middle has no effect on the
relative to it. The zero potential point is indicated by. the potential drop across the upper 10 Q branch. If the current
ground symbol .J, at point f. The Earth can be considered to through it is I.
be a very large conductor with infinite capacity. and Potential drop across R1 , V1 = IR 1 ~ '
unlimited supply of charge. Therefore the potential of the Potential drop across R 2 , V2 = IR 2 - V1-----V2-.i ·
Earth remains essentially constant. In practice electric Potential drop across branch, -12V-
circuits are often grounded, e.g., outside metal case of a V = V1 + V2 = I (R1 + R2) Fig. 2.76
washing machine is grounded by connecting it by a wire to a V 12
The current I = - - - = - = l.2
water pipe that is in contact with the Earth. R1 +R2 10
Since all the grounded points are a constant potential, it Hence V1 = (1.2)(6) =7.2V
is customary to assign it a zero potential. V2 = (1.2)(4) = 4.BV
(c) Power delivered by ..-we wish to determine the current through the 8 Q
V
source of emf &1 , 12 resistor in Fig. 2.77. Notice the polarity of batteries in
P51 = &1I = (12)(0.5) = 6 W 10 the lowest horizontal branch and in the right vertical
8
Power dissipated in resistors, 6 branch. The net emf in these branches is zero. So
PR =1 2 (R1 +R2 +R 3 +r1 +r2 ) 4 there is no current anywhere in the circuit.
2
= (0.5)2 (5 + 5 + 4 + 1 + 1) = 4 W
gabcdef
The power consumed by battery
2 in getting charged, Fig, 2.74
Ps2 = &2I = 4(0.5) = 2W

www.puucho.com
Anurag Mishra Electricity and Magnetism with www.puucho.com

/242 ..... ELECTRICITY&J\IAG~ETISM"J


2.0Q 6.0Q From eqns. (1) and (2), we have
R1 = R3
R2 R4
All the four circuit, (a), (b), (c) and (d) represent a
Wheatstone bridge network .
.-Jn Fig. 2.SO(a) shown, if a battery is connected
between points A andB, emf, & = 18V,what is current
12Q 15Q
through it? The resistors at C and D are redundant,
they are open branches, therefore no current through
them. The point E is grounded, but no current will
leak to ground, no return path for the current. Points a
Fig. 2.77 and dare at the same potential, similarly points b and
c are at the same potential, because these points are
What is the voltage drop across and the current through
connected by conducting wires without any circuit
the 6 Q resistor (Fig. 2.78)? Note that when current reaches
element. Notice the 6Q and 3Q resistors, similarly
node A, it "sees" two identical paths and therefore splits into
18Q, 9/ 4Q resistors, these pairs are a parallel
SQ C SQ arrangement.
..
R, R2 SQ SQ D I
,!,. Rs 6Q ~ 1QQ
18Q d 18Q c,.
SQ SQ SQ SQ
D •• I
R3 R4 .6V
,6V A B A 81
'•, 4/SQ 3Q 4/SQ
in I
J
Fig. 2.78.
E
6Q
a b
1/SQ 6Q
equal parts, The voltage drop across· Rr is equal to the
voltage drop across R3 • Hence points C and D are at same C - -
(a) (b)
potential. There is no potential difference across Rs and no Fig. 2.80
current through it. So R 5 has no effect. ·By' symmetry the Thus the equivalent .
voltage at C equals the voltage at D, so VCD =· 0. The resistors
R1 and R 2 are in series as are R3 and R 4 • Thus the equivalent
resistance of the circuit
· -between A and B is
2Q
u~s,.~
~--~-
.

resistance of circuit is S Q. The giveri circuit is called I.SQ. The current ! (Parallel of 3!l and.6Q)
Wheatstone bridge. We assume that R 4 is' unknown. R 2 supplied by battery,
can be varied and it is adjusted till the current in the
I= (18/1.S)A = lOA
mid-branch is zero. In this condition:
- -~- ' ~ ' -~- - - ' ' w
·...-1n the Fig. 2.81 shown we wish to determine current
Q ' ' 'provided by the battery. Just flip the circuit about the
1, p' .. dotted line shown in the figure. Now the circuit is
· simple with equivalent resistance 2Q and current
12 Q s '' I = (10/2)A = SA.
G
s R
s
wires are
:~not in 3Q
(a) . (b) ! - contact

rn
2Q 2Q 1Q
' 1Q i
3Q '
3Q V
A,----11-----s
10V
(a) (b)
(c) Fig. 2,81
(d) .,
Fig. 2.79
orJn the Fig. 2.82 shown we wish to determine the
P.D. across AC = P.D. across AD current through each S Q resistor. Note that both the
Also, P.D. across CB =P.D. across DB SQ resistors are connected to same points A and B,
Therefore I 1R 1 =I 2 R 2 ... (1) across which battery is connected. Therefore each of
and I 1R 2 =I 2 R 4 ... (2)
www.puucho.com
Anurag Mishra Electricity and Magnetism with www.puucho.com

SQ resistors is in parallel arrangement with .battery, equivalent circuit is redrawn, the equivalent
therefore the current through it, I= (2qtS)A = 4A resistance is 2/3R. Note that there is no current in

5!1
Series
• - >. R
R
A a c B

!__r.t!±i~
~
5Q (a) (b)
Fig. 2.85
(a) (b)
Fig. 2.82
branches ,be and ef.
orThe equivalent resistance between A· and B is Another symmetry is visible along line bd. The current
· 1 1 1 1 1 10
--· =-+-+-+-=-, R =2Q flow is not a mirror image in branches ab and be because the
R eq. 20 20 5 5 20 ·:"· flow is in same direction. This is called asymmetric
Thus the current supplied by battery= (20/2)A = lOA condition. The special thing about this asymmetry is that
Power supplied by battery = VI = (20) (10) = 200 W. current incoming at b is equal to outgoing current, similar
.-Jn Fig. 2.83 we wish to situation exists at b and d also. Thus resistor in branches be
determine the current sn and de are ineffective.
through 6n resistor. Both c ......--v"'-"-,__,,B In Fig. 2.86 there is asymmetry along line xy. The
the ends of the bottom current reaching O (I 2 ) is equal to outgoing current that
branch are grounded; so the D means there is no mingling of current from upper branch
net potential difference an 2l1
and lower branch into middle branch.
between E and A is zero. If 4Q

we traverse from E to A
there is a potential drop of E 6!1
12 V across battery, so there
must be a potential gain of Fig. 2.83
12 V in the resistor.
Therefore the current in 6 n resistor is
I= (12/6)A = 2A and to the left. What is the current :R
in the 9 n resistor? It is zero because there ,is no :y
potential difference across E and A. The entire: current (a) (b)
of the battery goes from E into the zero resistance
path back to A via ground. When ground connections
are shown, it is assumed that all such points are wired
to a common line even if
not shown .
.-Jn the Fig. 2.84 shown we
wish to determine current
in one of the 4 n resistors +=12v.-, ~..,;,;;,1v-+~....,_Mr,4 (c) (d)
Fig. 2.86
in the circuit. It is zero -
because the current will The resulting circuit is simple enough, the equivalent
follow the path of least resistance. and capacitance are 4R/ 5 and SC/ 4 respectively.
resistance, in fact a path of ..-we wish to determine equivalent resistance between
zero resistance is _ Fig. 2.84 A and B. In Fig. 2.87 points (1, 2), (3, 4, 5) and ( 6, 7) are
available, see figure . at same potential. Equivalent circuit can be redrawn
.-when a circuit is symmetrical' about a line (By as in Fig. 2.88.
symmetry we mean that two parts are mirror images
of each others), then the potential and current must
also be symmetrical. Therefore, currents in ab and ad
are same (Fig. 2.85). Currents in de and be are same.
Potentials of the points b, e and d are same. The

www.puucho.com
Anurag Mishra Electricity and Magnetism with www.puucho.com

ELECTRICllf&M~GNETIJ~]

R
R

B E
C R G

5 Fig. 2.92
Fig. 2.87 .-Referring to Fig. 2.93 we see that symmetry demands
The equivalent resistance of this series combination is that current only circulates in outer branch. Points A
r .r r r 3r and B are at the same potential because the circuit is
R =-+-+-+-=- symmetrical. Therefore no current can go across the
eq. 2 4 4· 2 2
r 1, f 3, 4, 5 · 6; 7 . ':-r resistors in the branch. Tl)e current through both
batteries is 2 A.
A Bl F 12V A
'
. 2.on s.on 4.on
_, ,,_,_,_ --- Fig. 2;88
L.
C G D

· .-In the Fig. 2.89 shown, the 4.0fl 6.0Q 2.on


resistances specified ar_e in
ohms. We wish to E
B 12V.
determine the equivalent
resistance between points Fig. 2.93
A and D. Points B and C, E :
and F are at the same '
NODAL ANALYSIS
potential, so the circuit can
s=---Nv---="<;; 1. It is based on Kirchhoff's current law eqn. At any
be redrawn as in Fig. 2.90. node in electrical circuit LI = 0 (it will be reffered as node
Thus the equivalent
Fig. 2.89 eqn).
resistance is -1 n. 2. In this technique to solve any numerical problem on
- -- electrical circuit assign potential of every junction of circuit
2
taking potential of any one of the junction of the circuit zero
2 (this will be called as reference node or reference junction).
3. Apply node eqn. to solve for unknown potential
introduced in the circuits. Current in node eqn. will be
2
...__,..,,,,,,,.,-B,C'-,JW-./E,F...__""",,,.,- written using resistor eqn. i.e., (1 = iJ
Fig.2.90

..-In the network shown ,B


~~J2ot>
in Fig. 2. 91 all the 'Find (i) Current through 4!1 resistor,
resistances are equal, (ii) Find patenti<11_difference_bePNeen A and B.
R
we wish to determine F . : 4fl 6V
equivalent resistance R A,------./1/V\,._j t-----::,,,0
between A and E. A,ffc-'M.=;c: 10V +6V
Points B and D have
same potential, 1
similarly F and H have
same potential. The
equivalent circuit is D
2Q +4V 4V B,
shown in Fig. 2. 92. The .f.ig. ~91
equivalent resistance of Fig. 2E.20 (a)
network is ?R/2.

www.puucho.com
Anurag Mishra Electricity and Magnetism with www.puucho.com

ELECTRIC CURRENT - __245


~---· ~1

•. _.:._-.,i

S_olution: We assign zero potential at point B 4Q sn


VB =0 A sv 0
VA -VB =10 20
10-6 y
I4n = - - 2n 6Q
4
by V =IR
or lA
Remark
If any of the junction of circuit is earthed then potential of Fig. 2E.21 (b)
that point in circuit is zero. x-0 + x-20 + x-5-y =O
8 4 5
4Q -4 6V
0 f----~·-10 y -0 y - 20 y - 5 - X 0
- - + - - + "----
6 2 5

10V
1~!Y;'x:;;m:Ei•e. :·22-i --~
I.. . -.-• . .-- . ··-'-• • ,,__ - ~----

Find: (i) I 4 n (ii) VA - VB


A 4Q
2n -6 4V -10 X
Fig. 2E.20 (b)
If we assign zero potential at P then potential at 10V 2Q
different point as shown in Fig. 2E.20.

xlL--..J\,'V\,"-----1 ~--__J,B
2Q 4V
(i)Find the current through SQ resistor (I sn ), and Fig. 2E.22
(ii) Find potential difference between A and B (VA - VB).
4Q x SQ
Solution: Node x
A 0
x-6 x-10 x-4
SQ --+--+--=0
20 2n en 4 2 2
34
YB x=-V
5
I 34-30 =~A
Sx 4 5
20V Remark:
Fig. 2E.21 (a) While writing node eqn. at any junction consider all
resistances connected to the junction taking potential of
Solution: Node A: battery if connected appropriately.
ll =0
x-0 x-y x-20 EQUIVALENT RESISTANCE
--+--+--=0 ... (1)
8 5 4 In order to determine the equivalent resistance, we first
Node B: reduce the network of resistance to a single resistance Re~.
y-x y-0 y-20 Equivalent resistance means a single resistance that will
--+--+---=0 ... (2) have same effect as the network itself.
5 6 2
Remark: Network of
While writing node eqn. at any junction, assume its Resistance
potential to be highest or lowest and write node eqn. for
that junction accordingly.

While writing node eqn. at any only junction in the same V


circuit the fredom of assuming the node potential to be V
highest or lowest is not affected by previous choice. Fig. 2.94 (a)
V
Req = -
I

www.puucho.com
Anurag Mishra Electricity and Magnetism with www.puucho.com

r:. ----- -- .. ·-. .. . - --· ····--~-i


L246 ___ ,. ELECTRICITY 8, MAGl!EUSM ·.1
R = Battery voltage
eq Current drawn by network from the battery
2Q
B
2n so

R"

100\/
Fig. 2.94 (b).
x-0 + x-100 2 = 0 Fig. 2E.24 (b)
4 2 It is obvious that potentials
x=S0V
Vp =V'p
R = 100 sn VR =V'R
AB 100-x 100-x
---+--- VQ =V'Q
2 2
On superimposing P and P' and Q and Q' etc., we get
,E: • a'::; :fe IIr------i-
1t .~,X,_;.-;~ij5:I:".: 23 J~
..... the simplified arrangement of resistors as shown in figure.
Q Q'

0(-) B
Fig. 2E.23 Fig. 2E.24 (c)

Solution: Applying KCL we get 69


Hence R,q = r0
149
--·---·-- ... ,,______,,,....,.,.... -'"r
x-0 2(2x-100)
--+-'----'-
R R
0
I.-~---!;;xp.tj,\,~ll.~ ; 25 t >
"' - ' ».'"'k,-,,,,~-=- -· ·--- ·~-:-----

x=40V
Each branch in the following circuit has a resistance R.
R _ 100 =-SR The equivalent resistance of the circuit .between the points A.
AB - x-(lO0x) x 6
3 and Bis:
R
~--,E---~." ._-._~----~-~-·:-~:' -. _· t7-~
.L,::F.?1f'J,~g1tfr..,1_!~Jt,> B
Find the equivalent resistance of the network shown in the
figure, across the points O and A. The resistance of each:
'branch of the octagon is r0 •
Fig. 2E.25 (a)

(a) R. (b) 2R __(c) .43 (g) B_R


Solution: Due to symmetry about line AB network
can be represente~ as .
A R/2
R/2 R/2 R/2

Fig. 2E.24 (a) R/2 R/2 R/2


' R/2 R/2 R/2
••" ••••'• ••••••••••••• s· ... I

Solution: Afrer connecting A andO to the terminals of A


a battery we find, there is symmetry about QA. Therefore the Fig. 2E.25 (b)
current distribution will the same as shown in the figure.
Equivalent resistance = 2R

www.puucho.com
Anurag Mishra Electricity and Magnetism with www.puucho.com

ELECTRIC CURRENT ---- ____ 2m


Thus 2x-40+ x+ 4x-4[20-x] = 0
-120
or x=-.-
4n a 11
V,=+10V
i------,,, Node B Now we can get current in all the branches.
1A ov
V8 =+6V

10V
Lil;~~E!TI~.L~;[!s L>
1A .-In the given circuit we wish to determine the current through
4A
;resistor_given in box.
Vc=+10V Vb=+4V Node A Conducting wires
t-----1
NodeC 3A 2n b Reference node
4V

Fig. 2E.26 50V R=2n

Solution: Noded
R R
X
Step-1: Assign node A as reference node x+ 50 x+50
Node e Nodec
Step-2: Now ~e can assign potential to points
V0 =+6V R
R R R R
Vb =+4V
V, =+l0V
Step-3: Current through 20 resistor is ov 100V
R y R
v,-v& =10-4= 3A Node a
Node f Node b
2 2
Current through 40 resistor is Conducting
½-V 0 l0-6=lA wire
4 4 100 V
Step-4: Apply KCL at node C to get current through Fig. 2E.28
lOV battery is 4A.
Solution:
lJ~~>§~P?~LifvT> Step-1: Assign voltage at all the nodes as shown in
figure. Here x and• y are unknown voltage at node e and f.
4n b an Step-2: Apply KCL at node f
1, X 12
20V
C
ov y-100 y-(x+50) y-x y-(x+50) y O
2n
a Reference ', ~--+~--~+--+~---+-= ... (1)
20-x node '. 2 2 2 2 2
1,
Step-3: Note that outgoing current at b is equal to
an d 4n incoming current at node a, thus we get

20V 100-~+50) +(100 -y)=-[(O;y)+ 0-(\+50)]- .. (2)


Fig. 2E.27
2
Note that we require first two equations to get x and y.,
Solution: Step-1: Assign c as reference node. y-x
Solve x and y to get current in R as - - .
Node a is at 0V 2
Step-2: Assign node bat x volt now node dis at 20- x -25 125 . 75
volt.
x=
2 , y =
2 and t =
2
A

L~?:S~~~~~ i/291>
Step-3: Note that I 1 entry and exist similarly current I 2
entry and exit potential drop an section ab is equal to that on
section de.
Step-4: Apply KCL at node b to get In the given circuit we wish to determine current through'
x-20 x-0 x-(20-x) ,b_ranch having indicated resistor 2Q '
- - + - - + --'-----'- 0
4 8 2

www.puucho.com
Anurag Mishra Electricity and Magnetism with www.puucho.com

··· .. ELECTR1cirr &MAG~fiis~

Conducting wires

1, 1, 100 V
5V
Node c xv 2n l3 (x+5) V X
Node b Reference.p
Node e 1 X 2l+50+2l
1, node '
,--- ----
4
., R R R
4Q : 2n: 4Q
100 V
'
L ••• ••• ••''
10 V 2Q Fig. 2E.30 (b)
Node a
'Node d 0V +10V ov Reference Step-2: By symmetry 100 - y = y - 0 or y = 50V
node Step-3: Now apply KCL at node c
x-100 x-50 x-0
---+--+--=0
R R R
Conducting wires or x=50V
Fig. 2E.29
Step-4: If R,q is equivalent resistance of this circuit.
Solution: We have
Step-1: Assign potential at each node as shown in 100 X 50 50
--=-+-+-
figure. . R,q R R 12
Step-2: Note that outgoing current at node C is equal 2
or R,q =
to incoming current at node b. Secondly incoming current at 3R
e.
I2 =I3 +I4 I ,EX a,m;.;.,, e. l\r3·-1-1·. ~
~,,"
. _: ~ . g ~ :::::'.p;;g._;_::;~ .,.,,:: .
Thus our equation is at node c and node b
100-x
x-(x+ 5) +(x-0) = -[(x-0) + x+ 5-x + x+ 5-10]
100V
2 4 4
'--v---'
2
'-.---'
2
'----.----'
12 13 l4 A
Now solve for x to get potential all the nodes.
100 V
On solving , we get
x=-2.5V,
75
Current through 2Q resistor is · A. ~ + ~ + 1Q_Q 0V x/R x"""
2 R R R NodeA
Equivalent Resistance by Nodal Analysis
Fig. 2E.31
l~~~~t'Tl};~;1,i··Go7.··'3>
F,---"-' j ,jU l~
""'""= --'2.,M,MSM~

Solution: Here we wish to determine equivalent


:In the givenfig,,re we wish to determine equivalent resistance resistance between points A and B.
betwe~n poinf (I. and B. Step-1: By using concepts of symmetry assign potential
at all nodes in terms of x and y.
Step-2: Apply KCL at node A and B
Node A: x-(100-x) +~ + x-y = 0
R R R
y-x y-x y-(100-y)
Node B: - - + - - + ~ - - - ~ 0
R R R
R s: Step-3: Solve for x and y to get,
250 300
Fig. 2E.30 (a) x=-- and y =--
7 7
Solution: Step-4: Now apply KCL,
100 X X 100
Step-1: Attach a 100 V battery and assign potential at - - =-+-+-to get,
each node. R,q R R R
R =7R
eq 12

www.puucho.com
Anurag Mishra Electricity and Magnetism with www.puucho.com

ELECTRIC CURRENT
---------

Determine equivalentnsistance between_,\ and B. Each resistance is R determine equivalent resistance.


100-x R NodeA
k----~---~---.¥x
A B 0
, 100

10
100-x X

100V 100V
Fig. 2E.34
Fig. 2E.32
R _ 100
Solution: Equivalent resistance can be obtained by. AB-2X y
R __ 100 -+-
AB - 2x SQ ·
R R
-+- For node A
R R x-(100-x) + x-50 + x-y + x-0 =O
~ + x-50 + x(lOO-x) =
0 R R R R
R R R For nodeB
75
X=- y-x +y-50 + y-(100-y) + y-50 +y-x + y =O
R
R R R R R R
On solving for x and y, we get
625
y = 50 and X=-
Determine cu_1T~nt_ !hrough _indicated SQ resist<1r, 13
20: .__-a 20V
y+ 1;;;. y 5Q w h1c . . m
. h on substltutmg . R AB .
gives des1re l 3R
. d resu It --.
. 19
,---- ----,
5Q
1....
X j,,...._-N\J\,..--L...~
sn] L:s0gmp,I,~ f357>
5Q -10 10V ABCD is square (see Fig. 2E.35) where each side is a uniform
wire of resistance H2 A point E lies on CD such that if a
L__ _ _ __,.,, _ _ ___J 0
uniform wire of resistance l fl is connected across AE and
10Q = constant potential difference is applied across A and C then B
Fig. 2E.33 and E are equipotential.
CE (b) CE= 2
Solution: (a) - =1
ED ED
~+ X + 10 + X - y - 20 = Q CE 1
(c) - = - (d) CE = ..J2
10 5 5 ED ..J2 ED
f+Y+lO +y+20-x =O·
5 5

60
5
On solving for x and y we get,
-10
1nAo1n:
X=13' y=l3

Iso
y-(-10)
5
lSJ'
D 1n
Fig. 2E.35 (a)
C 1

-10
13 + lO _ 120 _ 24 Solution: (d) Equivalentresistance between A and E:
-"''------A
5 65 13 x+l
y=--
x+2
For B and E to be equipotential

www.puucho.com
Anurag Mishra Electricity and Magnetism with www.puucho.com

I250 __ --- ----


~-----~·- -: ~.:- ··--·-. ~ ~

~~!lmR"~ j 37 ?
-·-;"':"

1
The circuit. diagram shown in the Fig. 2E.37-(a) Consists of a'
large number of elements (each element has two resistors R 1 ,
.and R 2 ). The resistance of the resistors in each subsequent
element differs by a factor of k =2. from the resistances of the
. 2 . '
resistors in the previous elements. Find the the. equivalent
D X E 1-x C· resistance between A and B shown in Fig. 2E.37.

RAE
RAB
Fig. 2E.35 (b)

= REC
RBc
:=-~;00 R1 kR 1 k2R1 k4 R1

Fig. 2E.37 (a)

x+l 1-x
Solution: When each element of circuit is multiplied
(x+ 2)x 1 1 by a factor k then equivalent resistance also becomes k
Solve to get: x=Fz-10. times.
Now CE= 1-x =Fzn Let the equivalent resistance between A and B be x.

, .. ED X
R1 kR 1 k2R1

:~::::
,:::- . - .. · ..... ~
['cg;,,~~i:im~~ , , ~ ~ '·
'In the Fig. 2E.36 ( a) the resistances are connected as shown.
'Determine the equivalent resistance between po_ints A and D. X
A

kX
Fig. 2E.37 (b)

So the equivalent circuit becomes

B 100 C
TA

Solution:
Fig. 2E.36 (a)
1 B ----',__R_2_ __, kX,

20 . Fig. 2E.37 (c)


PointsBandC, andE (R1 -R,) + ~-R-f_+_R_i__
+6R_1_R_2
and F are at the
same potential, so 2
the circuit can be
redrawn as shown in
figure. Thus, the
equivalent _Fig. 2E.36 (b) Find the resistance RAB of the frame made of a thin wire.
resistance is 10.. There exists parallel axis of symmetry. The 'Assume that the number of successively embedded equilateral
points across the parallel axis of symmetry can be treated as triangles .(with sides decreasing to half) tends to infinity [see
equipotential points. Fig. 2E.38 (a)]

www.puucho.com
Anurag Mishra Electricity and Magnetism with www.puucho.com

AB x cannot be negative ~ - - - ~
-4r ± ~16r 2 + 96r 2 (2.J7 -2)
~---r
X
6 3
a
But r=-p
2
2(.J7 -1) a
x = - - - x - p =0.SSap
c, B' a ·I,'(
3 2 .

i _ Fig: 2E.38_ (a)_ .. I 'I ,,,


l;~~#r~J?j~,J391>
Side AB is equal to a·a!'d the resistance per unit length of wire,
;Two circuit§. [as shown in Fig.2E.39 (a)] are called circuit A;
1is r. I

,1=~-~
,and circuit B. The equivalent resistance of circuit A is x and'
1LetRAB =xis equivalent resistance ofsystem between A and !that of circuit. B-- is y between. 1.. ..
and 2. ..
1B. As the resistance ofa conductor is directly proportional to I
2R 2R 2R 2R 2R
lzength,
' the equivalentresistance
. 1 and B1 will be~-·
- between A.,. -2
Therefore, the ,equivalent circuit becomes as given below· in ,2 - ••
Fig. 2_E.~8 (a) - Circuit A

Solution: Let AB= 2r,


tben A1C = CB1 = M 1 = BB1 = r.
In tbe circuit 2r and~ are in parallel between A1 and Bi,
2
tben tbeir effective resistance is
l,
'
:=·~;00
2R 2R 2R

Circuit B
2R 2R

Fig. 2E.39 (a)


2r~ .
Ri =--2- = 2rx (a) y > X (b) y = (F3 + 1)R
2 r+~ 4r+x (c) .xy=2R2_ (d) X:::Y_"';~_ ..
2
Now R 1 is in series witb M and BBi, therefore, tbeir Solution: (a, b, c, d)
1
effective resistance is
2rx
2R
. 2R

R2 =R1 +2r=--+2r
4r+x R • X R
.y
R 2 is in parallel witb 2r (of AB), so tbe net effective •
resistance across AB is
C
:2 ----.L...-,__......,
Circuit A·
2----.L...--....,......,
Circuit B
(b) (c)
flg. 2E.39
R(2R+x)
x=
'. 3R+x
3Rx+ x 2 = 2R 2 +Rx
x2 + 2Rx-2R2 = 0
-2R±~4R 2 +8R 2
x=
2r
2
Fig. 2E.3B.(b) -2R +2.f3R
x=

r2 x 2r (~ +2r)
4r+x
x=-=---
. R2 + 2r 2rx· ) +2r
---+2r
( 4r+x
=> 3x 2 + 4rx-Br 2 = 0
iFor a cell, a graph is plotted between thepotential difference vj
:across the terminals of the cell and the currentl drawn from_,
-4r+~6r 2 +4x 3x 8r 2 1the cell (see Fig.2E.40) Find the e.m.f and internal resistance.,
or X=_
2x3 !of.the ceq. ___ .... _ __ ___ __ ___ ___ __ __ ____ .. · '.

www.puucho.com
Anurag Mishra Electricity and Magnetism with www.puucho.com

:252 ..... ELECTRICIIT& MAGNETISM.-:


V (volt)
2.0

1.5. What i5 the potential differenq,. between the points M and N


for the.circuits shown in Fig. 2E.42 (a) and (b) for case I and
1.0
case II.
0.5
'-+--+--+-l--''1-.,.1 (amp)
2 3 4 5 ~1~ p~~
Fig. 2E.40
~~
MN
~~
MN
Solution: r = ~ = ~ ;: ; Q4 o · · 1 1' Case-II
I 5 (a) (b)
when I =·0, the potential reading is 2V..Hence e.m.f.= 2V
-
e::~~9'i~tte>Jt~-Jr41 ~ ~ . '
Fig. 2E.42

''Solution: Case I: Current in the circuit


Find the current in each part of the circuit. Apply loop law in: =12-6=~=1. 2 A
Fig.2E.41 (a). . ' ' 3+ 2 5
rnn 3V VA -V8 =12-3xl.2
=12-3.6=8.4V
' ' ••• i t'·'· Ve -Vv = 6+ 2x 1.2= 6+ 2.4= 8.4V
Hence VM -VN = 8.4V
3Q 6n: , , .,. 12 6 18
Case II: + = = 3.6A
. _, l ' 3+2 5
J', ,_,
.·,"';,l, I VA -V8 =12-3.6x3
. : t ;,,;i
= 12-10.8 = 1.2V
V8 -Ve =6-3.6x 2= 6-7.2 = -l.2V
4.5V , , . Ve -Vv =+l.2V
Fig. 2E.41 (a) , , 0
• •
=> VM -VN =l.2V
• '.. • :-- ; .. : !' ·:
1--~E~:~m,.,.I:
l£:li'.l'J:,~:2'.'':':Jl~..
~
f43l~
, Soluti_on:
. 10n. 3V
·In the circuit diagram shown in Fig. 2E. 43 ( a) find the
;currentthroligh the l Q resiBtor.

l-11 ·,
.:>05:
fill·· .
2Q 10 V 2n
3n

1ovr1n
2n
,· ,

5V 2n
4.5V
Fig. 2E.41 (b) Fig. 2E.43 (a)

(1) - 3[ - 61 1 + 4.5 =0 Solution: CV-10)-10 + v-o + V-5 =0


or 1+2I1 =1.5 ... (i) 2 2 1
(2)10 (I - l 1) + 3 - 61 1 " 0 ·
or 101 -'IOI{ = -3 ... (ii)
Solving equations (i) and (ii), we get"
1
l=-A
2
1 1 ·
and I -1 1 =--- =0
2 2

www.puucho.com
Anurag Mishra Electricity and Magnetism with www.puucho.com

' ELECTRIC CURRENT 253


V-20+~+V-5=0
2 2
V-20+ V + 2(V-5)=0
~ 4V-20-10=0 ,. ~ ,._·' s,-
+
s, -
+

V = 30 = 15
4 2
V _ 5 = 15 _ 5 = 15-10 = ~ R3 R4
2 2 2 Fig. 2E.45 (a)
i=5/2=~A
1 2 Solution: Let currents drawn from batteries &1 and
- ----- -- i-----. &2 be I I and I 2 respectively. Then current through various
I .l:;xgm,i;>lc~--L44 !_> resistors will be as shown in Fig. 2E.45(b)
. 1, . R1 R,
In the given circuit of Fig. 2E.44 (a) all batteries have ,;• ',,I

e. m.f 10 V and internal resistance negligible. All resistors are 1, (l,+l,J ,,


in ohms. t;:alculate the current in the rightmost 20 resistor. + +
s, - Rs - s,
,, ,,
1~~~1ITT 2n
2n'--~----~

10V-~
1 2
n

10V
·-

R3
. , Fig. 2E.45 (b)
R4

'-----~N2wn'--l10~ . Applying Kirchhoffs voltage Jaw on left mesh,


I 1R 1 + CI1 +I 2 )R 5 +I 1R 3 -&1 = 0 ... (1)
Fig. 2E.44 (a) Now applyi[lg ~ on right mesh,
-I 2R2 + &2 -I 2R4 -(Ii+ I2)R5 = 0 ... (2)
Solution: Let the potential of point P be O. Potential at From equations (1) and (2), •
other points are shown in Fig. 2E.44 (bl Apply Kirchhoffs
current Jaw at Xx· _ I1 = 2A
10 V 4 X (Assume)
and --- J 2 = lA ·

10~(("~
2
1 ' Current through resistance,
R 5 =l 1 +1 2 =3A
r-J1~ V 1!o V: ~L:-~---~-~--i:i-"'"nili-,.~--~--~i:~
p 10 2 10~ 2 0
-'------..MV--.
0 (Assume) A resistance coil,, wired.to an external battery, is placed inside.
•a thermally insulatg_d cylinder fitted with ·a frictionless piston,
Fig: 2E.44 (b)_
and containing ari ideal gas. A current I = 2 .40 m A flows.
x-10 x-10 x-20 (x-10)-0 through the coil, whicfhas a resistance R = 4900 . At what
--+--+--+----=0
4 2 4 2 speed v must the -piston of mass m =12 kg move upward in.
x-10+ 2x-20+ x-20+ 2x-20 = 0 order that the temperature of the gas .remains unchanged?:
~ 6x=70 •[g =; 9.8m/s 2 ] _ _ _ __ __ _ __ _ __ •

Solution: As the gas is ideal and its temperature


remains constant, the heat supplied by the resistance must
be equal to the change in potential energy of the piston i.e.,
- ·tv·
m.

In the circuit shown in Fig. 2E.45 (a), R1 = 20, R 2 = 30, _


R 3 = 30, R 4 = 20, R5 = 20, &1 = l6V and &2 = llV. .. ...,. ~-~-:::
Calculate current through res_ista_11ce ~ 5 • __
,.
I

Fig. 2E.46

E =Pxt =I 2Rt =mgh

www.puucho.com
Anurag Mishra Electricity and Magnetism with www.puucho.com

ELECTRlciTY' &, MAGNETISM


, ,;i,:·~·~·_ ___,

h I 2R currentJ/3. At pointB the current divides equally to J/6 and J/6


or, v=-=-- in branches BC and BD'.
t mg
(0.24) 2 X 490 .b
So, v = -'----'---- 0.24 m/s
12x 9.8

I~ eQ'.i~ ""'e.-r,vl:,··~.
_ :a,·:::·:rr,,ti:i:,'s.G,: ::· · . J ~ 1/3 1/3
1/6 1/6
12 cell each having the same emf are connected.in series and'
.are kept in a closed box. Some of the cells are wrong(y;
connected. 'This battery is connected in series. with an!
ammeter and two cells identical with series with an ammeter
and two cells identical with the others. The current is 3Awhen' D'
,the cells and the battery aid each other and 2 A when the cells Fig. 2E.48 (a)
and battery oppose each other. .How many cells in the battery
ar.e: wro_ngly connected? __ ·
Similarly current divides equally to J/6 and J/6 in
Solution: Let n be the cells in the battery that are
branches C' D' respectively.
wrongly connected, then '
&B = (12- n)& - n& = (12- 2n)& and rB = 12r Let equivalent resistance between A and A' be R eq. and
the potential drop acro~s it IR,q.
Battery Battery
+ ,+ From given circuit the potential difference between A
and A' can be determined as follows:
J 1 J
VA --R--R--R =VB
~s ~SA 3 6 3
. . + -
I I I
(a) (b) VA -VB =-R+-R+-R
3 3 3
Fig. 2E.47
From equivalent circuit VA - VB =IReq.
So according to the given problem as shown in Fig. 5 5
Thus we have, IReq. = IR =:> =
2E.47 (a), (b)
(12-2n)& + 2& =
6
Req.
6R
3 ... (i) Method 2 : In between A and A', symmetry of the
12r+ 2r
(12-2n)&-2& = circu_it indicates that B, C', D are at equal potential and
and 2 ... (ii) sim\larly B' ,C,D'. So the cube may be redrawn as
12r+ 2r
Dividing equation (i) by equation (ii), . R =~'+'~1+,~
eq. 3 6 3
14-2n 3,.
=- · .where '+' stands for the series
10-2n 2
R R R 5
i.e., n=l =-+-+-=-R
3 6 3 6
This means that in the battery o;,ly one cell is wrongly
connected.

I- exdt:ri..,.,..;; r 4s t··-,,.,,•.
. --,~c',:v··SYJ,.,,-\:;',.
·-~- , ,,-,,,,, ,--~/4.41/h,~ ' _ _.',+1.,=
, , ,,_,,,_";._ ~

'Twelve equal resistors each.RO are connected to Jann the,[


edges of a cube: Find the equivalent

resistance of the network. !'
( a) when current enters at A and leaves at A'.
'(b) when current enters at A and leaves at .B',
Fig. 2E.48 (b)
:(c) when c11rrent enters at A and leaves at_B.
(b) Once again the circuit is symmetrical. Figure
Solution : (a) The potential difference between any
two points is same no matter what path we take to arrive at shows the current distribution at junctions A and B. The
the second beginning from first. incoming current at D must be equal to outgoing current,
The circuit is symmetrical as shown in Fig. 2E.48, the entry similar situation exists at C '. Therefore the current in
point and exit point are identical. Therefore at A a circuit J/3 branches CD and C' D' are zero or we can say that the points
flows in each branch, similarly at exit point A' each branch have C and D are equipotential, similarly C' and D' have same
potential.
www.puucho.com
Anurag Mishra Electricity and Magnetism with www.puucho.com

[ ELECTRIC CURRENT . - 255


The equivalent circuit is reduced to Fig. 2E.48 (a).' (c) Let a current I enter the point A and leave the
Resistors irrelevout from point of view of current: point B. Figure shows distribution of current in the circuit.
Method 2 : In between points 1 and 3, 2 and 4 are at The wires AC' and AD have equal resistance and located
the same potential. Current in the wires 2 - 6 and 4- 8 is symmetrically, so they have same current; similar situation
zero. Potential difference between 2 - 6 and 4 - 8 is zero. exists with DC and.C'D'.
From KCL we have
D B'
I.=I1 +2I2 ... (1)

'LID
B
(c)
D'
A'
For the loop

or
AC' D' BA, ap_£lying KVL, we have
-I2R-I3R-I 2R-+I1R =0
I 1 =2I 2 +I 3
For the loop C ! D' A' B', applying KVL, we have
-I 3R +.(Iz, -I 3 )R + 2(I 2 -I 3 )R + (I 2 -I 3 )R
12- 13
...

=0
(2)

D
R 1, B'

J:,,
A C' I
R 12-13
1, 2(1,-1,)
R -
11 1, 1,
5 7 3
1,- 1,
1, C A'
R

R/2 R/2 Fig. 2E.48 (f)


R R/2 3R/2 or 4I 2 -5I 3 = 0 ... (3)
---->
R/2. R 3 3R/2 On solving eqns. (1), (2) and (3), we get
R/2 R/2 7
I 1 =-I
3R/4
12
~ 5
1 3 I 2 =-I
(d) 24
1
R D R I 3 =-I
11 D 6
A @ Ei'
The potential difference across AB =I I R = I_ IR
R C' R · · 12
R C R The potential difference across equivalent resistance
B R R
=IR,q.
D' B R R 7
Resistors irrelevant from Thus, IR ,q. =-IR
12
D'
point of view of current
7
or R ,q. =-R
12
R R

R
2R

R
®
=D Method 2: From considerations of symmetry alone,
points 3 and 6 must be at the same potential, and so must
points 4 and 5.
If two points in a circuit have the same potential, the
currents in the circuit do not change if they are connected at
these points by a wire.There is no current in the wire
because there is no potential difference between its ends.
R @ @ Points 3 and 6 may therefore be connected by a wire, and
=- ~ - = ~ ~4 5 ~ similarly points 4 and 5 may be connected.

~
Fig, 2E.48 (e)

www.puucho.com
Anurag Mishra Electricity and Magnetism with www.puucho.com

-.. -. -. -. --r •.• . ---•• -


--~---~·-
5
c~----'-~d
a

b
.I
3
(b) (c) (di,
Fig: 21:.49
__. ______ ,,y-,.......,w _ _ _ _, _ _ _ _ j, _ _ _

Rab =Ra, =R,a =Rba =r


R Rae =Rb, =R,t =Raf = r/-.fz
The power dissipated in resistor ab is
v2
Pab = -
r
By applying Ohm's law to the upper part, we .can
determine current through eel,
V
led r;:;
(-.,2+3)r
The power dissipated in the condutor eel,
v2
P2 =!Jar
c-/2 + 3) 2 r
Thus the required ratio is
P, =(-/2+3)2 =11+6'12
I P2
Fig. 21:.48.(g)
'·-· ···- ... J r1e~~1eJ~·,,,:,j15ol~.
.
L'.:'.:,\C".;r-t;:'tf!c ,;k_;.;,;,...;.r::;·~-- ~
Thus the equivalent resistance between a and bis J_ R. r·-· ._ ....... ·--- ····.-- :· ---------- ., - - -
12 'InJqe·c/rcuitshown in the Fig,, 2E.50 (a)all the.1-e§istancesl
1are.Jd.en.· ticaLand eq.ual to R_.• A.· consta_nt. vo. lt.age squ,rce.oferrif1
1Vy9lt is connected across AB, Detennine the current,suppliedj
\py~the source. _______ ...... ______ ... ~ !
_In the. cirq.,.it sh.own in Fig. 2E.49. (a)., abed is a squ.ar.e,,A_.. ll the\'
,wires forming the square and its di{lgonals are homogeneous Solution: From symmetry considerations, the
land. have same cross-section. .Find, ·the. ratio of power 1 potentials at points C and D are same. Thus the equivalent
,dissipated in resistors ab and ed. . circuit can be redrawn as in Fig. 2E.50(b). '
1 ! - \ -,>•,13.-··, ----
\
I
C d '
J, I C CID ;l;_•~es
i,. ~2
I
:.c
A A R•
B
·••• R O ••••••:.•( B

L ···---···· ..
Fig. 21:.49 (a) .
B

D
·:ii. R
R •• ••.• , /
~
R l
i

Solution: The circuit has asymmetry about line · w ~ I


XY, i.e., the current in left and right are not mirror images. I ..........
L . . . --~~-_j·.
__ . ________ Fig,• 2 E.so··'-
Just imagine the central junction of wires in the form of two Resistors in branches AC, AD; BC, BD are in parallel
junctions connected by wire ef as shown in Fig. 2E.49 (b). arrangement, their equivalent is R/2 Resistors in branches
Then it follows ftom symmetry consideration that there is no OB, BC/D are in series, this combination is in parallel with
current in the wire ef, thus we can remove it form the circuit. resistor in branch OC/D. J
The resistors of the wires will be proportional to their
lengths.
www.puucho.com
Anurag Mishra Electricity and Magnetism with www.puucho.com

[lj.!CTRIC CURRENT.
Hence, R = R/2(R/2+R) -~R 2A 2A
4A
eq. R/2+R/2+R - 8
18OV
Next we combine branches OC/D and AC/D that are in
series [see Fig. 2E.50(c)].
2A\.4sn.,'
18OV
... /.:.~'.~!'.~1, .-··········- u
90~··i
: 7/BR =. ~·~9,:
/ 90n ·• •.. / 1A

;7. . . . .~.
A 0
0
' 40
:) n
(c)
Fig. 2E.50 Equivalent Equivalent
resistance of resistance of
portion (1) portion (2)
Hence R' =~+~R=ZR
eq. 2 8 8
(b) (c)
Finally see Fig. 2E.S0(d). Fig. 2E.51
R" = R(7/8)R
eq. R + (7/8)R
Thus, the current I from source
Under what circumstances can the terminal potential:
V 15 V
=-- difference of a battery exceeds its emf?
(7/lS)R 7 R

~~~,~f!,tJ1!?1~.,.L 51 :> Solution: Two batteries AB


and CD emfs & and&'(&'>&) and
internal resistances r and r'
'.In the circuit shown in Fig. 2E.51(a)find the current fl.owing· respectively are connected in series
through the loon resistor connecting points V and S. as shown in the Fig. 2E.52.
r-----.,._P If I is the current in the circuit,
the total potential drop in the
Fig. 2E.52
circuit must be equal to net emf :
I(r+r') =&'-&
&'-&
or I=--
/1oon son\ r+ r'
The potential difference across terminals of battery AB
' L---.,w,._~C......--A,V.,.-~ ' is
\.~_0100~_/s\ .. _son®-~/ VB -VA =Ir+&
------- -.. ·----··
Obviously (VB -VA)> & by Ir, i.e.,
Fig. 2E.51 (a)
(VB -VA)-&= (&'-&)r
Solution: Fig. 2E.5l(b) shows simplified circuit. The r+ r'
battery is directly attached to resistor 90n, hence current in when a current is flowing in any battery opposite to its
it is 2 A, see Fig. 2E.5l(c). The total resistance of second emf, then terminal voltage is given by
branch is also 90n, hence current divides equally. Now V=&+Ir
current through 4sn resistor is 2 A and it is a combination of . . •.•..... r;;;:.;7
two equal 90n resistors. Once again current divides equally. 1_1=x~.i<Q~.i.~ , ~
90n resistor is a series combination of 40n and son, hence
current through them is equal, lA. As son resistor is a Two batteries having the same emf & but different internal:
parallel combination of two equal 100 n resistors, they must resistances r1 and r 2 are connected in series with an external!
have the same current, i.e., 0.5 A. :resistor R. For what value of R does the potential difference
between the terminals of the first_ battery become ze_ro? .

www.puucho.com
Anurag Mishra Electricity and Magnetism with www.puucho.com

Solution: Two batteries are i.e., 4l-2I1 -3[, =(1/10) ... (2)
connected in series. The effective,
emf in the circuit is therefore 2& and -(I -I 1 )100-(I-I 1 - [ 2 )100+ 100! 1 =0
because both push the charge in i.e., 2I-3l1 -J 2 =0 ... (3)
the same direction. Hence emfs are R
Substituting the value of I I from eqn. (1) in (2) and (3),
added.
Fig. 2E.53 we get
Net resistance in the circuit is
(r1 + r2 +R). . -2! + 7I 2 = (1/10) ... (4)
Therefore, current in the ci~cllit · ' . ~ I=2I2 ... (~
2& Again substituting the value of I from eqn. (5) in (4), we
l=----
(r1 + r2 + R) getf 2 = (1/30)A, so thatthePDacross 400ohm resistance as
read by the voltmeter.
The potential difference between the terminals of first
battery is (VA - VB), terminal potential ·difference is given by 1
V=-I2 x 400
(VA -VB)= &-lr1 2
where E is the emf of the battery .and r1 is its internal 1 1
=-X-X 400
resistance. Substituting the value of{ _we get 2 30
2&r1 · 20
VA -VB=& =-volt
r1 +r2 +R 3
=& (R + r 2 -r1 ) Method 2. After i 100n 8 100n °200n 1
(R+r2 +r1 ) combining the resistance of A o 8 C,
voltmeter with 400 Q, the. ,R P R G
For (VA - VB) to be zero, we. must have I'
equivalent circuit is shown in I ; 11 100n
R=(r1 -r2 ) Fig. 2E.54 (c), which is a ! P'-----'1----'Q i
This gives meaningful resul\ only if r1 > r2 . Otherwise, if balanced Wheatstone bridge, · 10V
r2 > r1 , then R = r2 - r1 will produce terminal voltage across because . ':ig: 2E.54_(_cl
second cell to be zero CVBc = 0).
.. , ,·· ,.. -r::-,-~'
l ,§;~Q~~.,'.!?. I 54 i, > ·
=~,.~> •we·~-·", ',.___~ '
P( 100) R( 100)
Q = 200 = S = 200
An electrical circuit is sh.own in Fig. 2E.54 (a). Calculate,tl;ze There is no current through the resistance G. Now 10 V
potential difference across tl;ze resistor of 400 ohm, as wil(be, is applied along series combination of 100 and 200 V, so the
. ,, , ,, 'f ~
measured by the voltmeter V of resistance 400 ohm, either by; voltmeter reading, i.e., potential difference across Q will be
applying Kirchhoffs rules or otherwise. V= 200 x ClOJ = 20v
(100+ 200) 3
(1/2)12 400Q
F V E,
(1/2)12 400Q I L.i~i~~tR~.§~
400Q 12
12)
8 (1-1,-12)0 200Q A part of a circuit in steady sta_te along with .the current,
A C:
!12 1oon 100n 200n (1-11) 200Q 100Q (1-12) i [/"lowing in the branches, with value of each resistance is'.
R I I (shown in. Fig, 2E.55. Calculate the. energy stored .in the
,, 100Q i, 100Q s
I !capacitor C. i
p Q" 1A
10V 10V
3Q
(a) (b) I 3Q A 5Q
Fig. 2E.54
'i ! -;::I D
i,

Solution: Method-I: The distribution of current in 4µF


~ 1Q

the given circuit is shown in Fig. 2E.54 (a). Applying KVL to


loops FECBF, ABDCQPRA and ABDSRA, we have I__, 1Q
8
12

2Q C 4Q
/3

respectively: I 2A 3Q
-(1/2)[ 2 x 400+ 200(! -I 2 ) + 100(! -I 1 -I 2 ) = 0
1A
i.e., 3I -I 1 -5! 2 = 0 ... (1)
l Fig. 2E.55
-(I -I1)lOO-(I -I 1 -J 2 )100-(J-J 2 )200+ 10 = 0 L ·-- __ .,; ~-:-:-~~-=-.,=~-"--~,,.,..,. • N'''f!'.:,"---,~ - -

www.puucho.com
Anurag Mishra Electricity and Magnetism with www.puucho.com

ELECTRIC CURRENT 259.


Solution: From KCL, at junctions A and B So energy stored in the capacitor,
respectively, we have ~-=lcv 2 = l x (5 x 10-6 ) x (2.4) 2
2+1-I1 =0 i.e., I 1 =3A · 2 2
I2 =1 A = 14.4x 10-6 J
and J 2 +1-2-0=0 i.e.,
Let potential at points A and B be VA and V8 respectively.
;· -- .· -- -- •.. ·-r ;i
l ~X9-;~:ij~~-'--~;:.->
.
Now we begin at A and after taking into account the
potential drops and gains reach point B. We have A piece of uniform wire is made up into two squares with a
VA-3x5-3xl-lx2=V8 common side of length 4inch. A current enters the rectangular
i.e., VA - V8 = 20 V system at one of the comers and leaves at the diagonally
opposite comer. Show· that the current in the common side. is
So energy stored in the capacitor,
one-fifth of the entering current. What length of wire
U =lcv 2 = lc4x 10-6 ) x (20) 2 = Sx 10-4 J C(lnnected between input and output terminals would have an
2 2 ·equivalent resistive effect?
56, ·>
tr 8 :x:q~;R:J~::c,_,_,_.......
-,,;.,;;,._.;,._,••w--.••.~ .- ,_._.,,_, .•.
C

In the given circuit,


&1 =3&2 =2&3 =6V 11 A . F
R1 = 2R4 = 6!.1 -"--o,--JS..~-o-~-
-
R3 = 2R2 =4!.1
.and C = 6µF
'Find the current in R3 and energy stored in the capacitor.
B I I D
(a) (b)
R1 = 6Q & 1 =6V Fig. 2E.57
C
5µFm
R2 =2Q
m
R =4Q
Solution: Let each side of the double square have
resistance R, and let the lettering of the diagram and the
(1-11) 11 currents flowing be as shown in Fig. 2E.57 (a). Applying the

f •b• t
Kirchhoff current rule, :EI = 0, to the junctions A, B and E in

&3 =3V
f t'urri (with the convention that current entering a junction is
R4 =3Q ~osi.tive and current leaving a junction is negative), gives
(1-11) I 1 -I 2 -I 3 =0, ... (1)
I 2 - [ 4 -I 5 = 0, ... (2)
Fig. 2E.56
I 3 +I 4 -I 6 =0. ...(3)
Solution: Let current I originates from the battery of Applying the Kirchhoff voltage rule to the loops ABED
emf &1 , distribution of current in the circuit is shown in Fig. aµ_d BCFE gives
2E.56, in accordance with KCL. , . I 2 R+I 4 Rx 2R =0, . .. (4)
Applying Kirchhoffs voltage law to loop a in clockwise I 5 x2R-I 6 R-l 4 R=0 ... (5)
sense, we get '. Eliminating Is and I 6 from eqns. (2), (3) and (5), we
-4I 1 +6=0, qbtain
3 ... (6)
i.e., I 1 =-=l.5A I1-I2-I3=0
2
I2 -2l3 +l4 =0 ... (7)
Now applying KVL, to loop b, we get
-(I -I1)x 3-3-2-([-[ 1)x 2+!1 x 4= 0 2f2 -!3 -4l4 = 0 ... (8)
or, -5[ + 9I 1 = 5,
Eliminating I 2 from these three equations gives
9 I 1 -3!3 +I 4 =0 ... (9)
i.e., I=-I 1 - l and 2l 1 -3f 3 -4! 4 = 0, ... (10)
5
1
or l4 =sl1- ... (11)
or, l=~x(¾)-l=l.7A [asl1 =¾]
Further, the potential drop from A to F by route ADEF is
and applying Kirchhoffs voltage law to mesh c, treating
VAF =[ 3 x 2R+l 0 xR =R(2I 3 +[ 3 +l4)
the capacitor as a seat of emf V in open circuit,
using eqn. (3). By (9) this becomes
VA -2-(l.7-l.5)x 2=V8
i.e., VA -V8 = 2.9V VAF =RCI1 +2I4)

www.puucho.com
Anurag Mishra Electricity and Magnetism with www.puucho.com

i?i:in' .. . ELECTRICIIY &.MAGNETISM l


,260 _ .. - -•" - - M'~- '-~--OM _ _ _ ,_,.1

and using (11), Thus if X be the equivalent resistance between P and Q,


R
VAF =I,,l+¾)=~Rl1
2R
The equivalent effect is therefore obtained if a wire 7/5
times the length of any side of the square is connected R
between A and F, because if produces the same potential
R
drop as the double square between these points [see Fig.
2R
2E.57 (b)J.
p
l. ·§,-x··,;:~~P::,',"
.. ··-,:c,·, le-~1-~, R Q
J 58 r::.--- 2R
2R
(a) A network of 12 resistors' each of value R = 60 are,
2R
interconnected as shown in Fig. 2E.58 (a), being placed along
,the sides of a regular dodecagon. Each of the terminals 2R
1, 2, 3, ... ,12 ,has been connected to each of the 9 terminals,
(other than nearest) directly by insulated wires each of.. (b)· (c)
resistance R, there being 9wires from each terniinat making; Fig.2E,58
108 wire connections totally. [Only one set of 9 wires, from:
1 1 1 1
.terminal 1 have been shown]. Find the equivalent resistance of - = - + - + ... to 10 terms+-
,the network when the current enters from Pat the terminal i: X 2R 2R R
and leaves Q at .terminal 2 · : 10 1 6
=-+-=-
'(bJ If the. above network were generalised.so that there are n! 2R R R
(n = even)resistors each of resistance R placed along.the sides! R
of a regular n-gon and if each terminal point of a resistorWerei Hence X=-
6
connected by (n - 2) insulated wires each of resistance R, \
·directly to the (n - 2) terminals, other than its, nearest, The equivalent resistance= !!en= lQ
6
terminals, find the equivalent resistance. across any two;
(b) Proceeding in the same manner as in case (a)
terminals of the network (i.e., current entering at one ofithe;
we find that the equivalent resistor is given by X where
two terminals an.d leaving by the .9ther.
, 3 R 4 R
..I'; 1 1. 1
- = - + - + ... to (n-2) terms+-
1
X 2R 2R R
n-2 1 n
=--+-=-
2R R 2R
. 1ent resistance
Hence equ1va . = -2R
R
n
7 ' Remark:------------------
I The above symmetry simplification can be done only if n is even.

FOLDING SYMMETRY

R 10 R each segment
has resistance
Fig. 2E.58 (a) R
•.
Solution: (a) Since each of the terminals is
connected by an "insulated wire to each of the remaining 11 conducting
terminals by a resistor R, the symmetry shows all the twelve wire
terminals to be symmetrically equivalent before any voltage B
is applied. Fig. 2.95 (a)
However, asymmetry is introduced just at the point
where current enters and also at the point where the current Figure is symmetrical about diagonal shown. We can
leaves the circuit. All other ten points are symmetry points, fold the circuit about this diagonal.
all at the same potential. Hence the given network reduces
to the following network:

www.puucho.com
Anurag Mishra Electricity and Magnetism with www.puucho.com

ELECTRIC CURRENT

b· •.. A
A

- Junction can
. . . R be removed
each resistance 1s now -.
2

·1 R
B B
X
Fig. 2.95 (b)
Now fold the circuit about diagonal b.
c;.
e
(c) (d)
: ·-. Fig. 2E.59
: ·· .. . . R
each resistance 1s now - .*tEor folding the potentials of points must be same.
4
B A R
Fig. 2.95 (c)
A
2
R R
Next fold tbe circuit about c. 2
sa R
R 2
At)i___!i
R 8 ·2
4 R R R
-
R -8 R
4 B 2 '--'/lv--',V,,::,.-.. B
R R B R R
R
55 2 2 2
Fig. 2.95 (d) Fig. 2E.59 (e)

.Now eqmva
. 1ent 1s
. -9R
4

Find RAB given resistance of each branch is R.


r:':Exa
i -::,
. : -le r·-7 ~.,
~Ere= 'L 60 !:,----
A
Calculate the equivalent resistance of the networks shown in
Fig.2E.60 (a) between the points A and C.
B

2n 3Q

G
C
B s
Fig. 2E.59 (a)
6Q

Solution: D
First fold the circuit about diagonal AB. Fig. 2E.60 (a)
A
Solution: Given networks are 'balanced Wheatstone
bridge' as in tbese (P/Q) = (R/S). Now as in a balanced
Wheatstone bridge no current flows through resistance G,
excluding G, effective resistance of tbe bridge between
. . R
Res1stance 1s now -. points A and C will be given by
2

B
Fig. 2E.59 (b)

www.puucho.com
Anurag Mishra Electricity and Magnetism with www.puucho.com

ELECTRICITY & MAGN_lTISM j


-------·--·--·~-.,~ -
8

A R B R Positive and negative ions are produced in the atmosphere due


to cosmic rays from space and also due to radioactive
elements in the soil. In some region in the atmosphere, the
R electric field strength is 100 V/m in the vertically downward
direction. This field exerts force on the positive and negative
ions in the given region in atmosphere.
s
P=Q=S=G=R As a result, positive ions, having a density 500/cm 3 drift
C
downward while negative ions, having a density 300/cm 3
(b) (c) drift upward. All these ions are singly charged.It is observed
Fig. 2E.60 that the conductivity in the given region is
So in case (a) 4x 10-13 (0- m)-1 . Find the average speed of ions, assuming
Sx 10 10' ' it to be the same for positive and negative ions. Also find the
i.e., Req ---=-D cwTent density.
cs+ 10) ' 3
Solution: Since the ion · are singly charged,
magnitudes charge on each ion is l.6x 10-19 c.
Using the expression for current density, J = nev
Magnitude of current density due to the drift of positive
ions
J 1 = (S00x 106 )(1.6x 10-19 )v = 800x 10- 13 v
Magnitude of current density due to drift of negative
ions
J 2 = (300x 106 )(1.6x 10-19 )v = 480x 10-13 v
· 1 m long metallic wire is broken into two unequal parts P and
Q. P part of the wire is uniformly extended into another wire (number density of ions has been converted to SI
R. Length of R is twice the length of P and the resistance of R is system; speed of positive and negative ions has been taken
equal to that of Q. Find the ratio ofthe resistances of P and R -. as v)
and also the ratio of lengths of P and Q, In a downward electric field, current density due to
positive ions will be downward and that due to negative ions
Solution: Let the length of piece"P be L then of Q will is also downward. So total current density
be (1-L)
L J=J1 +J2
So that, Rp =p- J =1280 X lQ-l 3 V ... (1)
S
but J = aE
and RQ =p(l-L)
J = 4x 10-13 x 100 = 4x 10-11 A/m 2 ... (2)
s
Now when part P is extended into another wire R of From eqs. (1) and (2),
length twice of P, i.e., 2L its resistance will be 1280x l0- 13 v = 4x 10-11

RR =p (~~1) = 4p½ [as SL= 2Lx S'.]


v=0.31m/s

Note: We will come across the following integration very


According to given problem frequently. So, remember the result as such.
RR =Rq
i.e., 4 p.£=pli-L)
s s
i.e. L =0.2m
Rp p(L/S) 1 Suppose that you have a large number of identical batteries
So, - with emf Z and internal resistance r, and a load resistor,
RR 4p(L/S) 4
R. What is the current produced through a resistor R when (a)
Lp L 0.2 1
and -=--= -- m batteries are co,znected in series? (b) n batteries are·
LQ (1-L) (1-0.2) 4 connected in parallel across R? (c) m. batteries are in series.
and n such branches are connected in parallel across R? (d)
What is the maximum power transfer to the load resistance in
case (c) ? ·

www.puucho.com
Anurag Mishra Electricity and Magnetism with www.puucho.com

ELECTRIC CURRENT 263 1


- .... J
mCells

~e~ f---'v'v\,--jf-,Vv\,---·--1:J
m cells in series
w R
Equivalent circuit n Branches : &
' 1,,l
n
, .
&

r
r

&
1,
&
r

I----V'lv--!1-----'-N'v-----l
r
&
,,
&
r

r
.
.
,
:
'
m& mr

• • r · ./\/1/\,- · • r · -f\/\1\,- • - • - • • ·• ,· -1\/V'v- • - • _____.


Fig. 2E.63 (a)
' ' '
ms mr
Solution: (a) If we start from A and traverse the
circuit to B the equivalent emf is &,q. = m&.
R R
Equivalent internal resistance, req. = mr
Total resistance of circuit = mr + R
Hence, current in the load resistance, I = _!!!£_
mr+R
(b) Since all the branches are connected to same
points, they are in parallel. The potential difference across
Fig. 2E-63 (c)
each branch is equal to terminal potential difference across
battery. ~iota1 resistance
· of c1rcmt
· · =-mr + R
n
& Current through load resistance,
I n cells,[n I= n& = &
·--. e r •• •· 't parallel
·-------\ t --- -·"Mr---/
&
.
R
R+(:) (: +~)
(d) The current in the circuit will be maximum
R Equivalent circuit
wlien denominator is minimum when(~+ .:..) is minimum.
· m n
· Fig. 2E.63 (b)
There are two variables in the expression to be
Thus, &,q. =& =:> minimised. Let p be the total number of cells, m x n = p.
r So m = p/n
req. =-n
·' Hence, .!!:..[nR
dn p
+.'...] = O
n
Total resistance = !.. + R
n
Current through load resistance, [ ~+.!....]=o
p n2
I= &,q. = &
= .E!.:_ = mr
r,q. +R (~+R) n2 n
R

External resistance =Internal resistance


(c) Equivalent emf of a single branch* m& This is also the condition for maximum power
All the branches are in parallel arrangement, so transfer.
&,q. =m& I =m&=n&
1 1 1 . max. 2R 2r
- =-+-+ ... n times

;r
req. mr mr 8 2R m2&2 n2&2
P = - - ~ = - - = - -2R
mr (: + 4R 4r
req. =n-

www.puucho.com
Anurag Mishra Electricity and Magnetism with www.puucho.com

---~, "" --
264 · '
~---
AMMETER AND VOLTMETER little current from the main circuit. When connected across a
An ammeter circuit element, the voltage drop across the voltmeter [series
measures current through
circuit elements and a
voltmeter measures
voltage across circuit Permanent
----- Voltmeter (2),

elements. A basic magnet


component of both of Multiplier
these types of meters is a
galvanometer.
galvanometer works on
The

the principles of magnetic


field, to be discussed
B
1,
resistor

(a)
.1W_ (b)
Cylindrical Fig. 2.98
later. Here we treat it as a iron core
circuit element that has A
(a)
combination of multiplier resistor and galvanometer]
an internal resistance (G) experiences a voltage drop of
and whose needle A~B· V=Vg +Vm
deflection is directly (b)
proportional to the Fig. 2.96
current through it .
. Ammeter ,,--__,,"l'v--1 R,, Multiplier
The galvanometer coil ,o-A/V.,._-i R,, resistors
has a small resistance G, •,..._,w.,._..J R,,
because the coil consists of Metre
metallic wire. A Switch terminals
Metre
galvanometer measures terminals
current but due to small coil i-----v--P\
resistance only currents in the
(a) Muftirange Ammeter ,(b) Multirange Voltmeter
niicroampere range can be
measured without burning (a) Fig. 2.99
out the coil. An ammeter that
~A~R1- ~ Larger potential drop is across the multiplier resistor
can be used to measure larger ~ ••~
rather than the galvanometer coil.
currents therefore must have (b)
V = Vg + Vm = I gG + I gR = I g (R + G)
a small shunt resistor, with ~ig. 2.97
resistance S, in parallel with a and I =~
g R+G
galvanometer to take most of the current. The shunt
provides a bypass through which a large current I can bypass The voltage V is also the potential difference across the
the galvanometer. The shunt and the galvanometer circuit element having a resistance R because of parallel
resistance are in parallel, thus to bypass most of the current connection.
the shunt must have the smaller resistance of the two. ' ' ,. ',. " --- r-7"· -
Because the voltage across the galvanometer and the shunt l...E:"~AJ'!,}pJg
"
64 ' ".•
1
-·· "'·'""'~-- .,,, ·~\..---""'"'
resistor are equal, we can write
_A voltmeter of resistance R v and. an ammeter of resistance 'RA !
Vg =V, :are connected in series across a b<1tterj of e.mj :!! and of
IgG=I,S .negligible internal resistance. When a resistance R is~
From KCL, I =lg +I, connected in parallel to volf:17!,eter, reading of ammeter,
Hence, IgG=(I-Ig)S increases to .three times while that of voltmeter reduces to 1
I =_E_ one-third. Calculate RA and Rv in terms_ of R
8
S+G
Solution: Let initially a current I flow through the
This equation allows us to select the proper shunt circuit as shown in Fig. 2E.64 (a).
/ resistance for a given current range and galvanometer.
Applying Kirchhoffs voltage law on the circuit,
Voltmeter IRA +IRv -:!! = 0
A voltmeter is capable of reading voltages higher than or t!!=IRA+IRv ... (1)
microvolt range. A large multiplier resistor is connected in
series with a galvanometer. Due to multiplier resistor the
galvanometer now has a large internal resistance, it draws

www.puucho.com
Anurag Mishra Electricity and Magnetism with www.puucho.com

I ~LECTRIC CURR~tlT
Initial reading of ammeter
is I and that of voltmeter is
+ - s
Ii! Imax. -lg
IRv-
When resistance R is (2.Q X 10-4 ) (50)
connected in parallel with 3.0-2.00x 10-4
voltmeter, reading of ammeter = 3.3x 10-3 fl
increases to three times, it Note that the shunt resistance is very small as compared
means current 31 flows Fig. 2E.64 (a)
to coil resistance G = son. The current that passes through
through ammeter but reading of voltmeter decreases to one the shunt resistor branch is (3.0-2x 10-4) A =29998A.
third, it means current I/3 flows through the voltmeter.
The shunt resistor is made of a material that does not burn
Hence, remaining ( 31 -½ = ~) current passes through R as out as readily as the thin wire of galvanometer. The ammeter
will read currents linearly up to 3 A. i.e., for a current of 1.5
shown in Fig. 2E.64 (b) A flowing into the ammeter there will be a current of 200 µA
First, applying KVL on mesh 2 of Fig. ? in the coil of the galvanometer, which would give a half scale
I 81 reading.
-Rv--R=Oor Rv=BR
3 3
Now applying KVL on mesh 1 of Fig. 2E.64 (b)
+ &_
'What is the required multiplier resistor for a voltmeter with ai
{ull scale reading of 3.0 Vin the galvanometer of the previous l
31 !example? i
Solution: Due to high multiplier resistor the major
potential drop is on it and only a small fraction on the
galvanometer.
lg = 200µA = 2.00x 10-4 A
Fig. 2E.64 (b) G=50Q
I
+Z--Rv -3/RA =0
3 As
1 lg
or Z =-IRv-+' 3/RA ... (2)
3 3.0-(2.00x 10-4 )(50)
From equations (1) and (2),
1 2.00x 10-4
-IRv +3IRA =IRA +IRv = 1.s x 104 n = 1sk.Q
3
or 2IRA =-lRv
2
i:r-·;~-~~m:Pif~.·r;:;11 67 .· ">
.,_,,,,,_ucJLh~i',\J~J:.---
'o;,_,,,,w_ ~"""
3
·To m~asure the resistance R of a. resistor, a voltmeter oJi
:resistance Rv is placed across a resistor and an ammeter is'.
-· ---- - . ··------
' full scale sensitivity of a galvanometer is 200µA .
!The '.placed in series with the combination as shown in Fig.
ifmaximum coil current], and coil resistance SQQ It is to be i2E.67.
I
(a) Find the. resistance R in terms of the measured'
'readings on the ammeter I meru and voltmeter Vmeas. ·
:used in an ammeter designed to read currents up to 3.0A (at,
!Ji,ll scale):. Wlt9:t ~-tfz~ !"quire~ shunt resistance? :Cb) Discuss the.result for Rv » Vmeas/Imeru·
Solution: The galvanometer can carry only a small
current, most of the current has to be shunted through the
shunt resistor. The given parameter are
, lg =200µA=2.00xl0-4A (lmeas-11)

=500 G Fig. 2E.67


!max. = 3.0A
A full scale reading of 3.0 A implies that when a current Solution: (a) Potential difference across
of 3.0 A enters the ammeter, I g should be 200 µA. As R = (I meas - I 1 )R; potential difference across voltmeter
IS =1 1Rv = Vmeas and this is measured potential difference.
I = - - we have
g S+G' The two branches are in parallel.

www.puucho.com
Anurag Mishra Electricity and Magnetism with www.puucho.com

ELECTRICITY
, ""
&MAGNETISriil
-·- ------ _: - '·-~----·--..-
' " "'

'' as S=-Ige...G_
(I-lg)
(l.OOx 10-3 )(20)
=-----'------~----'---
50.0x 10-3 -1.00x 10-3
CI -Vmeas/Rv) meas = 0.408(!
(bl From the expression for R, we get The equivalent resistance of the instrument is
I !meas
-=----
R Vmeas Rv
1
--=-+-=-+--
1 1 1
R,q. G S 20 0.408
1 1

If Ry >> Vmeas R,q. = 0.400Q.


I meas Note that shunt resistance is so small in comparison to
l I meas the galvanometer resistance that the equivalent is very
-<<-- nearly equal to the shunt resistance.
Rv Vmeas•
------- -- r:--i
thus 2_ = I meas
R Vmeas
I..1:::~~m1?:,~? j 70 i,,>

or R=Vmeas/Jmeas ,A voltmeter has a resistance G ohm and range V volt.

l 6a i,, :>
I ~~~m ~l-? ,_.v- 1
_,,--,,-,;:J,blli0AAW,.,N=<•,'~
Calculate the resistance to be used in series with it to extend
its range to nV volt.

:To measure the resistance R of a resistor, an ammeter of


resistance RA is placed in series with the resistor and'• the
voltmeter is placed across the series combination as shown in
!Fig. 2E.68. (a) Find the resistance R in terms of the measured, Fig. 2E.70
'.reading on the ammeter Imeru and voltmeter Vm,as· •,(b)
Discuss the resultfor vm,ru/Imea, >>RA. ',' Solution: The maximum current through the
1-lmeas galvanometer is
----<v V
G
R
-o-c+--<Al-'IIV,,..-'- With a multiplier resistor in series with galvanometer
RA .' ' the potential difference across the entire branch is
Fig. 2E.68 nV=lgG+IgR
Solution: (a) The voltmeter and ammeter branches
are in parallel arrangement. Hence
=(f)G+(~)R
vm,as
=U-Im,,,JRv =Im,asCRA +R) =V+(~)R
Hence, RA+ R = Vmeas
1meas or R = (n -l)G
Usually n is referred to as multiplication factor, e.g., a
R =(;:::-RA) galvanometer with a range Oto 1 mV is to be converted to a
voltmeter of range O to 1 V.
V lV
(b) For meas << RA, R "::::' Vmeas We have n = - - = 10
3

I meas I meas lmV


The multiplier resistor R = (10 3 -l)G = 999G.
So you can guess how large the multiplier resistor is in
What shunt resistance is required to make the comparison to the galvanometer resistance G.
l.00mA, 2.0.0ngalvanometer into an ammeter with a range
of0A to .50.0mA?
[ J:~~~~,eJi .Fi~
Solution: The given parameters are A shunt of resistance l/n the value of galvanometer resistance
I, = l.OOmA =-1.00x 10- A 3 is connected in parallel with a galvanometer. How will the
current sensitivity of the galvanometer change?
G =20Q
!max. = SO.Ox 10-3 A Solution: The deflection per unit current is called
current sensitivity of the galvanometer.

www.puucho.com
Anurag Mishra Electricity and Magnetism with www.puucho.com

r~. nEcrii,, CURRENT -


-·· ------ ----~--
267
1

cs =!!. Solution: The currents in the ammeters are


I proportional to deflections produced, let a 1 and a 2 be the
Reciprocal of current sensitivity, i.e., current per unit proportionally constants.
deflection, is called figure of merit. 11 = a.1n1, I2 = a.2n2
I l The ammeters are in series, hence
FM=-=-
0 cs I1 = I 2
Deflection Deflection a1n1 =a2n2 ... (1)
e e·
-----%--- ~ [!J· In the second arrangement the potential drops across
resistors are equal as they are in parallel arrangement.
Resistances of ammeters have been ignored assuming them
(1-1,) to be ideal.
Fig. 2E.71 Thus we have
I'1 R1 =l'zRx
The deflection (0) of the galvanometer is proportional to Also 1
I'1 = a.1n 1
the current passing through the galvanometer.
and I' 2 = a. 2 n' 2
0' I
- =_!_ So R 1 o: 1 n' 1 = Rx a 2 n' 2 ... (2)
0 J From eqns. (1) and (2), eliminating o: 1 and a 2 , we get
where 0' and 0 are the deflection of a shunted R1 n\ Rxn' 2
galvanometer and simple galvanometer. -- --

CS'=! =_![~0] =~(CS) Therefore


I I I I

For a shunted galvanometer with S = ~- WHEAT STONE BRIDGE


, n
. It is an electrical arrangement with forms the basis of
(I-I )~=I G following instruments:
g n g
1. Slide wire bridge/meter bridge
I =-J- 2. Post office box
g (1 + n)
3. Cary- foster's bridge
CS' lg l
Hence -=-:::--- 4. Potentiometer
cs I (l + n) Wheat stone bridge consist of 4 resistance battery and a
So CS'=_E_ galvano_meter as shown in Fig. 2.100.
l+n X
This shows that current sensitivity of a galvanometer
decreases by shunting.
0

Consider two different ammeters in which the deflections of y


the needle are proportional to current. The first ammeter is
connected to a resistor of resistance R 1 and the second to a
resistor of unknown resistance Rx. AtJirst the ammeters are '---,If---•~~-~
&
K2
connected in series between points A and B [as shown in Fig.
2E.72(a)]. In this case the readings of the ammeters are n 1 Fig. 2.100
and n 2 • Then the ammeters are connected in parallel between Node x:
A and Bas shown in Fig. 2E.72(b) and indicate n\ and n' 2 • Closing K 1 and K 2
Determine the unknown resistance Rx of the second resistor. x-y +x-0+x-& =O
. ~f-1~f.LJ~ G Q P
A~»- &-··· 0J - B
(a) Node y:
R y-0 y-x y-&
--+--+--=0
S G R
A~B In balanced eqn. JG =0 ~ x=y
(b) P R p Q
or -=-
Fig. 2E.72 Q s R S

www.puucho.com
Anurag Mishra Electricity and Magnetism with www.puucho.com
/

Note-1: Wheat stone. This is one of the practical form of Wheatstone's bridge
bridge is used to determine' ) ., ', . used to determine unknown resistance.
unknown resistance provided ' (i) /AB = 1 m
the other there resistances of 'i (ii) Wire is of uniform cross-section
network are known and bridge 1· p X
is in balanced condition
Note-2: Different form of
wheatstone bridge. ·, ,
(a) In this situation, whel} -~-; ,_,,. ,i
key K 1 is closed deflection of & , K, I
galvanometer remains same 1 '· Fig'. 2;101·
when only K2 was closed the~ ; ,· ' · ' ·· · -~ '7 ;. ·' · ·'
bridge is in balanced condition: ", ·, C:' '', ·, :.
~=~ .. ·~·G·=~ ·. '· ,,,1,.,
R G I ·p· 1'~' "'.,· Fig; 2.104

Balanced condition:
/. •:}.!")
', A o--'--AJ,t.,,--iC,-./G,}''-.o::--"JV\-i---.B ' (i) Ia=D
,·o . '.-.,, p X
(ii) -=---
(b) s Rl1 R(Z-11 )
·' · P-R(l-11 )
X=---~
' - - - - - - . - i f - - - - - ~ .. •J-,1 Rl,
' ' ' &, ,, ' ', 1 I. l ~ ,-
P· (1-11 )
X=>
~i!l.'.~.102 (~)_ z,
, !
The Potential Divider
A potential divider is used to supply a
specified voltage from a fixed voltage
"i' ! ,· ·r', ·.,' supply, e,g,, a 5 V device can be powered.
' s ' from a 9 V battery using a potential
I~ • , ,

. divider. Fig. 2,105 shows a potential


C ;

't• 'J - '" divider consisting of two resistors and a


source of voltage. The current in each of
' • I I -'
the resistors is same. Since the current,
_ Fig. 2.102 (bl
&
' ,,,, . ,•, V
'.L
, ' ' '
\'
' I=---
R, +R 2
Balanced conditions:
(i) Ia =0. . ·~.,. '' The RD. across resistor R1 ,
r, _,_.,: - R1
(ii) g=R V1 =IR1 = V
p s '
..., - R1 +R 2
and the RD. across resistor R 2 ,
--
----~---- i r ..
-- -- _R_,2'--V
, \I I
V2 =IR 2
R1 +R 2
Note the ratio,
V R1
-1= -
V2 R 2
s s The means the potential difference across resistors in
(a) (b} series is proportional to resistance.
Fig, 2:103
Meter-Bridge
(i) Ia= 0 Meter-bridge is a sensitive device based on the principle
P R of Wheatstone's bridge, for the determination of the
(ii) -=-
Q s resistance of a conductor (wire). Its sensitivity is much more
Note-3: than that of the post-office box.
Application Meter-bridge is shown in Fig. 2.106AC is a 1-meterwire
(a) Slide wire bridge and meter bridge.
www.puucho.com
Anurag Mishra Electricity and Magnetism with www.puucho.com

'L ELECTRIC CURRENT 269.


of manganin or constantan which is fixed along a scale on p
'Wooden base. The area of cross-section of the wire is same at Q (100-1)
Resistance-Box .-·. S~bstitut;ng this. value ~
. of Q in eq. (i), we get
·l R
(100-1) S
Shunt or S =R( 10 ~_:_z)
(P) (Q)
A C R is the resistance taken in the resistance box and I is the
(100-~ length measured. Hence the value of resistance S can be
determined from the above (ormula.
K A number of observations are taken for different
+I- /WI' • resistances in the resistance box and for each observation
Cell Rheostat Key
the value S is calculated.
Fig. 2.106 Finally, the experiment is repeated by interchanging the
unknown resistance S and the resistance box. The mean of
all places. The ends A and C of the wire are joined to two the values of S is ihen obtained.
L-shaped copper strips carrying binding-screws as shown. In
between these strips, leaving a gap on either side, there is a Errors and their Removal
third copper strip having three binding screws. The middle (i) The resistances of the copper, strips fitted at the ends
screw D is connected to a sliding jockey B through a of the wire and that of the solder have not been taken into
shunted-galvanometer G. The knob of the jockey can be account. These are called 'end-resistances'. These are
made to touch at any point on the wire. determined by another experiment in terms of the length of
the wire,and are added in the lengths of the two pans AB
Determination of resistance and BC of the wire.
The wire whose resistance (SJ is to be detennined is (ii) If the end of the ·wire of the meter-bridge is not
connected across the gap between the points C and D, and a exactly at the zero-point of the meter scale, or the knife edge
resistance box across the gap between the points A and D. of the sliding jockey (which tpuches the wire) and its index
Between A and C are connected a cell, a rheostat and a key. point (by means of which of the scale is read) are not in line,
In the experiment, when the sliding jockey thouches the then the length of the two pans AB and BC read on the scale
wire AC at any point B then the wire is divided into two will be different than the real lengths. To remove this error,
parts. These two parts AB and BC act as resistartces P and Q the experiment is repeated after interchanging the unknown
of the Wheatstone's bridge. resistance S and the resistance-oox. By doing so, error due to
First of all, a resistance R is taken in the resistance box any non-uniformity in the thickness of the bridge-wire is
and the key K is closed. Now the jockey is slided along the also minimised.
wire and a point is determined such that, on pressing the (iii) The resistance R taken 'in the resistance -box is so
jockey on the wire at that points, there is no deflection in the chosen that the null-point is nearly in the middle of the wire.
galvanometerG. In this position the points B andD are at the Then the percentage error will be least. In addition to it, the
same potential. The points Bis called 'null-point. The lengths effect of end-resistances will also be minimum. There is one
of both the pans AB and BC of the wire are read on the scale. more advantage. When the null-point is in the middle, the
Suppose the resistance of the length AB of the wire is P, sensiti".ity of the bridge becomes maximum because then all
and that of the length BC is Q. Then, by the principle of the four resistnances (P, Q, R and SJ become nearly equal.
Wheatstone bridge, we have, (iv) The current should not be allowed to flow in the
p R
-=- wire for a Jong time otherwise the wire will become hot and
Q s its resistance will be changed.
Let the length AB be l cm. Then the length BC will be (v) In the initial adjustments the shunt should be used
(100-1) cm. with the galvanometer. Near the zero-deflection position the
resistance of AB, P =p J_ shunt should be removed.
a (vi) The sliding-jockey should not be rubbed on the wire
. (100-1) otherwise the thickness of the wire will not remain same at
and resistance of BC, Q = p - - -
a all places.
where p (in ohm-cm) is the specific resistance of the Rheostat
material of the wire and a (in cm 2 ) is the area of The rheostat is a device used for varying the strength of
cross-section of the wire. Thus the current in an electrical circuit. It consists of an oxidised
nichrome wire (whose specific resistance is high and
www.puucho.com
Anurag Mishra Electricity and Magnetism with www.puucho.com

1270 ELECTRICllY & MAGNETISM

temperature-coefficient of resist.ance is, low) wound on a If the contact-point X of the sliding-constant S be at a


hollow china-clay cylinder. As the wire is oxidised, 'its turns distance of three-fourth length of the wire AB from the
are insulated from each other. The ends of the wire' are screw A, then the potential difference between A and X will
connected to the binding-screws A and Bfixed at the bas~, A be three-fourth the total
metal rod CD is fixed above the cylinder parallel to it. ·rt . 1 d'ff 3V
. e., -
potent1a 1 erence L .
carries a sliding metallic strip S. The strip can b~-slideci to 4
and for, pressing the coil of the wire: It. is called the 'sliding Hence a potential difference of
contact'. A binding screw C is fixed .at one encl of the rod:. 3 Fig. 2.110
v will be establised across the
Using the binding screws A, B and C the rheostat can be 4
used in two ways: , circuit. Thus, by sliding the
(i) As a current-controller: For this, one wire of the contact-point X from A to B any desired potential difference
circuit is connected to one .0f the binding screws .A· and B from O to V can be establised across the circuit. In the circuit,
fixed to the base and the other wire is connected to the the symbol of potential divider is:
binding screwC fixed to the rod [Fig. 2.107 (a), (b)].·.',• The total resistance of the rheostat-wire and the
.--------~'-----'· - maximum permissible current (which can flow without
overheating) is written on the handle of the sliding-contact.
D
:-' · - - ~ t18 C To circuit
electrical'
Necessity of High-resistance Measuring
. VWVI/INVI/VWWW\ Instrument for the Measurement of E.M.F.
A X 8 If we measure the emf of a cell , -
(a) by a voltmeter, then an accurate
value of emf will not be obtained. .. _____ r --- :
The reason is that when the
'i'
D
I I
s
le To electrical
circuit
-•:
voltmeter is connected across the
ends of the cell, it takes some
current from the cell, that is, the cell
+
+ :'
A X B does not remain on open-circuit. As
' VOitmeter
a result of potential-drop across the
(b) -' , '.
internal resistance of the cell, the Fig. 2.111
Fig. 2.107 ''
',:1.
potential difference between the
- ends of the cell becomes Jess than the emf of the cell. Thus,
Thus, in the first position [Fig. 2.107 (a)] the reslsfa'nte the voltmeter measures the potential difference between the
of the length AX of the rheostat wire is included in the plates of the cell which is less than the emf.
circuit, and in the second position [Fig. 2.107, (b) J the If Fig. 2.111, the ends of a cell emf & and internal
resistance of the length XB of the ' resistance r are connected to a voltmeter. Suppose the
w!r': is included. By sliding the ..... . /:'._ '., •• : resistance of the voltmeter is r. Then the current in the
shdmg-contact on the rod CD, the· -vvv7v~vvvv: circuit is given by
value of resistance included in the Fig. 2 _108· <· ' &
circuit, and hence the current in ' 1=-- ... (i)
R+r
the circuit, can be changed. In the circuit, the rheostat is
Let the potential difference between the plates of the
indicated by the symbol. cell be V, Since the plates of the cell are directly connected to
(ii) As a potential-divider: The cell, or the b~ttery; the voltmeter, the potential difference across the ~oltmeter,
whose potential difference V is to be divided, is connected that is, across the resistance R is also V. Therefore.
across the. binding screws A and B; and the electric cir;cuh' is
V=IR
connected across A and C (or across B and CJ as before
Substituting the value of I from eqn. (i), we get
(Fig. 2.109)
~ - - - - - - - - - - - - - T o ~l~ct'ricafl V = _B_ "'"' (")
... 11
R+r
D s C circuit ! The reading of the voltmeter will give V which is less
than& .
.... i .
• " v '
.
V
-
Suppose the emf of the cell is & = 2 volt and the internal
A X 8 resistance is r = 5 ohm. If the resistance of the voltmeter be
L
R = 100 ohm, then the reading of the voltmeter, from eqn.
(ii), will be
''v 100
Fig.;2.109 V = - - - x 2 =l.90volt
100+5

www.puucho.com
Anurag Mishra Electricity and Magnetism with www.puucho.com

271
If the resistance of the voltmeter be 200 ohm, then ·used with the galvanometer to' prevent damage when far
200 from the balance· point. When the balance point is reached
V = - - - x 2=1.95volt
200+ 5 this resistor is shorted out. In the balance condition no
If the resistance of the voltmeter be 2000 ohm, then ~urrent flows in th~ galvanometer but the driver cell (V)
2000 ~upplies current throughout.
V - - - x 2 = 1.99 volt.
2000+ 5 Applications of Potentiometer
It is thus dear that higher the resistance of the
(i)· Comparison of emfs· V
voltmeter, nearer will be its reading to the emf of the cell (2
volt). Hence, for accurate measurement of the emf of a cell, In the figure shown Z1 is the
the measuring-instrument should be of very high resistance. unknown emf and Z 0 is a standard
cell of known emf. The null point is
Potentiometer

rr~rres
.first ,obtained with the standard cell,
The potentiometer is simply a let it be 10 , and then ·with the
Low resistance
piece of resistance wire, usually a unknown emf, let it be .11 . Then
metre long, fixed between two V
points A and B with a cell of Zo =-lo
output V connected between the
L &,
V
two ends. Ii can be used to and Z1 =-11 Fig. 2.114
A~B L
measure the emf of a source
without withdrawing any current So we have Zo =,!.Q,
from the source, measurement of Z1 11
il)ternal resistance and (ii) Measurement of Internal Resistance
comparison of resistances. In the figure shown a resistor is
Consider a source of emf Z V connected in parallel with the cell
and a galvanometer connected in of emf Z and internal resistance r. J
series as shown in Fig. 2.112. The First the null point is obtained with A . . - - - - - - i - - ' B
potential at C is +Z and at D will +V F 0 the switch open; this measures Z, &
be zero. A potential difference of A since no current flows from cell,
C D
Z volt exists between C and D. there is no potential drop across r.
When C is connected to A, the Therefore the balancing length is
R
potential at junction AC becomes pr,oportional to emf z.
Fig. 2.115
+V and that at D becomes V-Z. Z oc 10
When Dis connected to the wire Fig. 2.112 Next the switch is closed and a second balance point 11 is
AB at F, the galvanometer will show no deflection if the found. This will measure output voltage of the cell,
potential drop down the wire is equal to that across the i.e., V =E-Ir, Vu:. 11
source and meter.
E 10
P.D. across AF = P.D. across CD Since -:::::-
V 11
We assume that P.D. across AB decrease uniformly from
A toB. and E = V +Ir
Let length AB = 1 m and AF = 1 m. therefore r =(::-l)R
Then Z =~
l (iii) Cor:nparison of Resistance
V
If length AB = L m, then z =- l Tp,e figure shows circuit in this case. First the null point
L
is obtained with one resistance (11 ) and then with the other
In practice the source is CZ2)-
connected to the potentiometer
wire by a sliding contact (or
jockey). The jockey is moved
along the wire AB until null -A..,._-~~---J__. 8 ,
point [point of zero deflection in ;
galvanometer] is obtained. R
G
The actual circuit is shown in
Fig. 2.113. Key S1 is used to cut
Fig. 2.1_13
R2 C
out the supply emf to prevent
Fig. 2.116
overheating of potentiometer wire. A protective resistor is '
www.puucho.com
.,
Anurag Mishra Electricity and Magnetism with www.puucho.com

~ ' ---
i 272
'·- --··-.
1 IR 1 R1 (v) Measurement of Current
Then -1 = - = --
12 IR 2 R2 The required circuit is shown in figure. The potential
drop across R can be found by finding the null point. If R is
(iv) Measurement of Small emfs (Thermocouple)
known, the value of current can be found. This principle can
Since the balance length is proportional to the emf, be used to calibrate an ammeter or voltmeter.
therefore it will be small.
Illustration:
For a thermocouple the emf is of R In a potentiometer set up, the balance point for the
the order of a few mV, the null point unknown emf & is found at 70 .4 cm from the left and of the
will be very close to one end of the meter wire. If the driver cell has an emf of 1.5 V, then
wire and therefore very difficult to
determine the emf of the unknown cell.
measure. This situation can be
tackled by putting a very large
Sol. We know that
Thermocouple
resistance in series with the & = · &o (because length of the wires is 100 cm)
potentiometer wire. Therefore there l 100
is a large P.D. across R and only a &1.5
--=-
small P.D. across the wire. 70.4 100
Let the dry cell have emf of 1 V and the potentiometer E = (l. 5J(70.4) 1.056V
wire have, a resistance of 10 n. If R = soon then the voltage 100
drop down the potentiometer wire is Checking of Connections:
5 x l = 0.0098V
510

. Al-------tlB A+.,----~--'B,
J J

IR
G
R
(b)
Fig. 2.118
Suppose we want to calculate the balance point for a FiQ. 2.'119: The galvanometer _'should Show oppp§ite' '
deflections when the jockey is first connected;
thermocouple giving an emf of 5 mV. at and later On connected at A.
·Drop down 100 cm= 0.0098V
Therefore for a drop of 5 mV., Consider a battery with emf &, which is less than the
potential difference VAB across the full length of the wire. If
length= 0.00 5 x lOO 51.02cm the jockey J is connected with the point B, then a current I
· 0.0098
flows into the cell & (against its emf) and the galvanometer
Superiority of Potentiometer over Voltmeter shows deflection towards right as shown in figure. If the
(i) When . we measure the emf of a cell by a jockey J is now connected to the point A, as shown in figure
potentiometer, then in the position of null-point no current then a current j comes out of the cell & (in the direction of
flows in the cell circuit, that is, the cell is on open-circuit. its emf) and the galvanometer shows deflection towards left.
Hence we obtain the actual value of the emf of the cell. If the deflections at A and Bare not opposite, then either the
Thus, a potentiometer is equivalent to an ideal voltmeter of emf of & is greater than the potential difference between A
infinite resistance. and B, or the circuit been connected wrongly.
(ii) For measuring the emf by a voltmeter, the deflection Note that: The most common mistake in connecting
in the voltmeter is to be read. There may be some error in up is not joining both positive terminals at A.
reading the· deflection. On the other hand, the Reading of a Potentiometer:
potentiometer method is a null method. One has to read
The reading of a potentiometer means determining a
null-point position on the wire. In reading the null-point
point on the wire AB at which the galvanometer shows no
position there can be a maximum error of 1 mm. If the wire
deflection. Suppose the jockey J is connected to the point C
is 4 meter long and its ends are directly connected to a cell of
as shown in figure. If potential difference between A and C is
emf 2 volt, then the potential-gradient along the wire per
less than the emf &, then the current flows out of the cell
mm will be 2/40000 volt= 0.0005 volt. Thus there can be a
and the galvanometer deflects toward left.
maximum error of 0.0005 volt in the emf. This can be
If figure, the jockey is connected to a point D which is
further reduced by taking a still longer wire.
nearer to the terminal B. If the potential difference between

www.puucho.com
Anurag Mishra Electricity and Magnetism with www.puucho.com

kELECTRIC CURRENT , ' ' ,.


A and Bis more .than the emf S, the current flows into the that of the Wheatstot:\e bridge shown in figure. Hence, the
cell and the galvanometer shows deflection toward right. value of the unkno: :{~:nJe is given by ·
It implies that the point of zero deflection lies· in
between C and D: By trial and error (but no scraping of the
jockey), we can find a point F such that, when the jockey is
connected to this ,point the galvanometer shows no Note that:
deflection. Thus, the potential difference VAF is equal to the 1. The accuracy of the post office box depends on the
emf S of the cell. choice at ratio· arm R 2 •
R1
THE POST OFFICE BOX
,Jt is ,a compact form of the Wheatstone bridge. It 2. If R 2 :R1 is 1 : l, then the value of the unknown
consists of compact resistance so arranged that _\lifferent res/stance is obtajned within ±ill. ·
3. If the ratio R 2 :R1 is selected as 1 : 10, then the
desired values ,of resistances may be selected in the three
arms of Wheatstone bridge, as shown in Fig. 2.120.
r . .. . X .. •
unknown resistance X = 1
1
10
) is accurately measured upto

±0.ill.
A R1 B R2 C 4. If the ratio R 2 : R1 is adjusted to 1 : lQQ, then the vah,!e
,,(_l_) is obtained
~~:·r~~~
of unknown resistance X = to an
'\100 ,
accuracy of ±0.01 Q.

, ,
,o
1\,2sow
\
2 5
'R
2020 E
mustration:
The vaJue of an unknown resistance is obtained by using
500,0, 20002000100~ 500 200 20010cf1
a post office box. Two consecutive readings of R are
observed at which the galvanometer deflects in the opposite
o O .D'!;][l[lI1IJI1U _directions for three different value of R1 . These two values
' ' , , K1 '
: ~-~B' are recorded ,under .the column-I and II in the following
: f \.I !. __ ~
l observation table. .
G
(a)
B

,1 10 10 -16 il.7 j
2 100 10 163 •164
3 1000 10 1638 163,9 I
'
Determine .the-valµe of,the m;tkQOWt:\,resistance.
-Sol.·The observation.table.may b!' completes as follows:

_ F.i!J· 2.1~0 S.N:\ ~(n)Lt~e)~ ~l'.es]n-br~t.tQ ~ X=R(R2~8:l{I


·Each of the arms AB and BC contains three resistances of , , · _' i' \ IJ<l) · \'}Jl1(m · 1(0) lHQJj
•10, 10 2 and :10 3 0, respectively. These are <:ailed the ratiQ ..

arms. -Using . R
. . these resistances the ratio
2
R, can be made to
' 1
'2
10
100
10
10
16,
163
17
164
16:0
16.3
17,0
16,4 i
l

.ha,ve any of the following 11alues: ·100 : .1, 10 : 1, 1 : 1, 1 : 10 3 ,1000 10 1638 1639 16.38 16.39 '
or.I: 100. · ·
The arm AD .is a complete resist_ance box containing The value of the unknown resistance lies in-between
resistance from 1 to 5000 Q. The tap keys K 2 and K 2 are also 16:38,O,and 16.39 n.
-provided in the post office box. The key K1 is ·internally The unknown value may.be the average of the two
C!)tµtected to the point A and the key K 2 to the point B (as i.e., X = 16.38 + 16.39
shown by dotted line in the figure). The unknown resistance 2
Xis connected between C and D, the battery between C and or X=l6.385Q
the k~y.I<: 1 and the galvanometer between D and the key K 2 .
Tqe circuit $qown in Fjg. 2.120 (a) is exactly the same as

www.puucho.com

Anurag Mishra Electricity and Magnetism with www.puucho.com

___ ,; . ELECTRl(ITY &MAGNETiSM :j


I,\, the shunt is n - l of the current I in the rest of the circuit.
n
,. - -- ---· --·· -·--------~-"""! Hence,
,:\n ammeter is-connected to measure the current intensity inaj
1circuit w.ith·a.. resistanceR.. What relative erro. rwill be m.ad.e.if'
·connection of the ammeter does not, change. the current
'intensity· in the circuit? The voltage· across the ends· df the
;cir5Cuit is kept c9nstant. . . •. . . . '. . . : , ....1

Solution: Before the ammeter is connected, I O = ~,


R
and after it is connected, I = -.-·- , where R0 is the
, V , IA gal;a;ci~eter (coil r,:s;;~n;~: 99Q)-;;--;~~~~r~ed in;ci:ci~i
R,+R 0 : 'lam,meter. zis,inga.shunt oflQ and_ con. nected as. shown in. F~._J
resistance of the ammeter. The error is · 2E.76 (a). The ammeter reads 3,A Tl!e same galvariometer zs!
I -I '· 1 1converted into a voltmeter by coririecting a resistance ofl0lQ,
s =O- - = - - -
Io . 1~~
in
1
series, ,This voltmeter is ,cq,nnected as shown in Fig.;
2E.76 (b):'Its reading is found.to be 4/5 of the:jull scale,
Ro reading. Find: · '·
When R 0 << R, the error may be neglected. (i) internal resistance of the cell (r),
·,=· .... ,,,.,,,.,, ' ·1741·-...., ' (ii) range of the ammeter and -voltmeter. , · , 1'
r=·,,.,.•,•s:c,,'
~ ·
Xa!t':11\~-'""'
·, ,i:e:
• , ~- 4i
. Zk
e ,r,:c,,.~,, 74 · ·
r:J;,;w/,,;;;·_,.,__ • 'l

-
11..
~
.b-.
''-""'

- .. -
_. -·--~-- ----- ·····-- (iii) fu]l scale cwfiection current offae galvanopetet:,,,.-.J
[Determine the voltage across a resistance Rus'ing a,volt1]l~ter' Solution: For ammeter
iconnected. to its-ends. What relative. error will be made ifth'ei 991g =(I-[. )1
ireadings of the voltmeter are taken the voltage app/iejd as or I =1001• ... (1)
[~efore it was switched on? The Cl!T"rerzt intensity in the circuit I g is the full scale deflection current of the
,is constant. _________ --· ____ ~ . -.- ·---·- .--- .,·. --'·'·-· · ,..__. galvanometer and I is the range of ammeter.
. . · . V0 -V For the circuit in Fig. 2E.76 (a),
SO I ut1on: The error sought is s =---,where V0 is
, Vo . 12V ~ 3A
the voltage across the resistance k before the voltmeter is 99x 1
2 +r+--
switched on and V the voltage after it is switched on. 99+ 1
According to Ohm's law, r =L0lQ.
For voltmeter range,
V0 =IR and V =I RRo
R+R 0
... (2)
where R 0 'is Y1~ resistanceRof the
.
voltmeter. Hence,
. .' -,:-12v
!,' j---V-.'v--,
Ro
1+~
Ro
E=---
f
I 2n
2Q
~,-----1.VJ--~
is determined only by the ratio between the resistances
(a) {b).
of the section of the circuit and the voltmeter. When
Fig. 2E.76
R 0 >> R, the error may be neglected.
P5~"'""' ·I'
rtc·~,~~lmll,~:.1ir=i-~.
I- -
75
·•• '• -
v
- ~ - - - - - ~ - _ , , _ _ _ _ _ ,_,,_, __ , " ' " - -
Also resistance of the voltmeter= 99+ 101.= 2000.
In Fig. 2E.76 (b), resistance across the terminals of the
battery, '.
!What resistance r should be used to shunt a galvanometer!
R =T+200x2=2.99Q
!:Z:::i::~~~~e:~; ~:~~nee R ~ 10,000 ohm to reduce itj 202
6 I
Current drawn from the battery.
Solution: A reduction in. the sensitivity n times 12
11 =--=4.0lA
means that the galvanometer carries a current I I which is n 2.99
times smaller than the current in the rest of the circuit Voltmeter reading,
before the branching off. Therefore, the current I 2 through 4
-V =12-I 1 r=12-4.0lxl.01
5

www.puucho.com
Anurag Mishra Electricity and Magnetism with www.puucho.com

r·------
eiii:TRIC CURRENT .
- -- -- - "

r----~,------ . --~···:-- - -- -----.


lg 99Q 101Q J:;xq~p:t~
l. .•.. 7s \ ---,.
L'
~
. . " .. ', ,.w. ·"""'"'·' - - - ~
: :
The galvanometer shown in Fig. 2E. 78 (a) has resistance son
and current requiredfor full scale deflection is l mA. Find the
:...----.,. V--..: resistance R,,R 2 and R 3 required to convert it into ammeter.
(c) (d) having ranges as indicated.
Fig. 2E.76 G

5
V = 7.96x - = 9.95V
4 R
9 95
Using eqn. (2), I = · = 0.0SA
200 g
Using eqn. (1), range of the ammeter A .X Y Z
10A 1A 0.1A
I =lOOI, =SA.
I... .. .. · .. . c· f------, · Fig. 2E.78 (a)
1:,1~~qm,~b"Ec:1 77 L>
Solution: For the range 0.1 A, R1,R 2 and R3 are in
A galvanometer has an internal resistance of son and.current series combination, equating potential difference across
required for full scale deflection is l mA. Find the series galvanometer and series combination of R1 ,R 2 and R 3, we
resistances required (as shown in Fig. 2E.77) to use it as a get
voltmeter with different ranges, as indicated in Fig. 2E. 77.
G R1 R2 R, Hence
I,G
... (1)
(I-I,)
1V 10V 100V For range 1 A, (R1 + R 2) is in parallel to (G + R 3).
So, I,(G+R 3)=(I-I,)(R1 +R 2)
Fig. 2E.77
or I,(G+R1 +R 2 +R 3 ) =l(R1 +R 2)
Solution: For range 1 volt, galvanometer and R, are
in series Sub,stituting ~~r::~G:J :::[~R;:i:+=R:),::hav~ ( )
I = V 1 2
g (G+R1) 1 99

or 10-3 =--1__
lg G 1, G
(50+R1) fl fl
or 50 + R1 = 1000
R1 = 1000 - 50 = 950n R1 R R3 R1 R2 R,
For range 10 volt, galvanometer and R 2 , R 3 are in series,
(1-1,) (1-1,)
10-3 = 10
A. z A y
(G +R1 +R 2) (b) (c)
10 3 Fig. 2E.78
G +R1 +R 2 = - - = l0x 10
10-3
For range 10 A, R1 is in parallel to (G + R 2 + R 3 ).
or R 2 = 10000-(50 + 950) = 9000 = 9kn
and for range 100 V, galvanometer, R1,R 2 and R 3 ar_e in So, I,(G+R 2 +R 3 )=(I-I,)R1
series.
I,(G+R 1 +R2 +R3)
R1=~-------
_3 = _ _ _l_0_0_ __ I
10
(G+R1 +R2 +R3) 10-3( so+ 50)
100 3 5 1
G + R1 + R 2 + R 3 = - - = 100 x 10 - ~ - -9-9~ = - -0hm = - -0hm ... (3)
10-3 10 990 198
or R3 =100x10 3 -(G+R1 +R2)
=100x10 3 -10x10 3
= 90x 10 3 = 901<.Q

www.puucho.com
Anurag Mishra Electricity and Magnetism with www.puucho.com

-· _______ , ~ ~_: ___!LECTRICITY.& 'M~[ETIS'[j


(b) The high resistance R keeps the current · drawn
from the standard cell within permissible limit and grevents
a large current to flow through the galvanometer when far
away from the balance point ..
'I (c) At null point no current flows through R, hence
:I (I ~1 9)
I A X
no effect on null point.
(d) The null point depends on the terminal voltage
_F'!J· 2E.~8, (d)._ ,_,.
of C and the emf of C1 only, hence no effect.
5 5 1 (e) The method would not work if the potential
So, R2 =---=-Ohm
99 990 22 difference across AB due to the driver cell C became less
50 than the emf of cell Ci, because there would be no null point
and R3 =--(R 1 +R 2 )
99 on the wire. (i) In this case the .potential difference across AB
_ 50 _ 5 _ 45 _ 15 Ohm due to cell C of emf 2.0 V will be Jess than 1.0 V. Since the
99 99 99 33-· ,• emf of the standard cell is greater than this value, there will
1 . · 1 . be no null point on the wire. (ii) Similarly, then method
Hence, R1 =-Ohm,R 2 =-Ohm
198 22 would not work if the emf ofC were 1.0 V instead of 2.0 V.
15 (f) Suppose the emf S to be measured is say, 1 mV
and R3 =-Ohm = 1 x 10-3 V, the potential difference across AB due to C is 2.0
33
V, the length AB of the wire is 100 cm and its resistance per
,,~¾d'~i~1, e:?"9r~
,.~""'-
-~1?t12E~11:i'3~-1E,sts:~ centimeter is .k. Then the balance length will be
f ' ' ';' ' ' ' ._ ' _, --~ '' ' ..: '·.': ~-·',_ ''". 1 1 x 10-3 y = P.D. across Al due to C
!Fig; 2E.79(a)shows a·potentionieter.using.a cell'. C ofemf2.0, = current in the main circuit x resistance of Al
IV
I - _ . ·· ' .L '__ . ' · - ".
and in:e1"11.ahresistance 0.400 pqnnected to ~ resfstor,"!irej
. '
= -2.0 .kAJ = 2.0AJ V
,AB. Astanilard cell of constant elJ.lf of,1.02 V gives a, bµlance 100k 100
ipoint at (517,3 cm length of the _w/re, A. very high r,~tanceJ or J!,J =0.05cm
1R = 600kn is, put in.-series witf!; .. the standard,ceU.(Thisl

!resistance isc.sliorted by inserting switch S when close to''the


Thus the null point will
1balance p9i~t The standard cell ~ .then ,:eplaced by; a c;ll of, be very close to A and there
,unknown e_mf S and. the null pou;zt tu~ out to be. ~:p cm1
ilength dfthewire. . ,.. , :,,. is an extremely large
' ~ - - - ....;.---~ ""-A--•-•, percentage error in its
·! C measurement. To have a
large balance length, the
circuit shown in Fig. 2E. 179
(b) is modified by putting a "
suitable resistor R '. The ,,-, ____F_ig_.2_E,__,_79 (b) , , ......J
balance length will then be
R measurable and the percentage error will be much smaller.
'i
s·~ ~--- - --- - - - ~
Fig. 2E.79 (a) !
-~··••o· ~ - - - - - - ~ - - '
i:i~"
~" .:'~:"'.9',,.,
""'' ~
,~"''~~"'"'"J
·: ,le··->«r~:::,'Cc:,-
.,,,Ll\&k SU i«,
e,;.;...:r..
(a) What ,i's the value of S ? (p) What is the purpose ofi. -·-·~-•6:,'"'~--F•:••• ),,-•,,:>,, ---,--,-~.---~--: .-~·-7
using the high resistance..R? (C) 1s!•the nullpointajject;ed byl fl. th/n ;uniform: wire AB of length l m, <lTlc unknown'
this high resistance? (d) Is the'n1tJ1 point a./fecte~ by tliel resistarice.X and a resistance 0/120 are connected. by thick
internal resistance ofthe cell C?'(e). Would this metlwd.'work conducting strips, tis shown ir, ihe figure. A battery .and 'a
if: . ; ' ,; .. . . . ! galvanometer (with a sliding j9ckey connected to it) are al.iii
(i) the intematresistance ~f'cell 'c were. highe; thdh tliel available.':Connections are to'. be: made to measure the
1
resistance Qf'Wire AB and . . , l unknown resistan~e. X using the principle of Wh~atstorie
(ii) the emf of cellC were 1.0 Vin.stead of 2.0 V? (fJ Would. the bridge. Answer thefollowing questi9ns: ., ··
circuit work'well for determining e.xtremely small errifof the
order of a:tewritillivolts? What modification do jou suggest.in
the circui.t.. -----------------·.,·-------- .....,. I'
• '° 'r
Solution: (a) The value.of Z is given by (Z1 = 1.02V
is the emf of the standard cell C1 )
A B C D ', . ~
S = l.02x 82.3 =l.Z 5 V
617.3 ~---~~ l. Fig. 2E.80 (a) · .;
• , __•-::c,c;.,=--=....=:--~-=--·--i,~~-N
.

www.puucho.com
Anurag Mishra Electricity and Magnetism with www.puucho.com

:- ELECTRIC CURRENT
I.,,,,,"-·~----
. ·.. iii_l
( a) Are there positive and negative terminals on the:
E-V
r=--
galvanometer? I
(b) Copy the figure in your answer book and show the battery where Vis the terminal voltage of C' and I is the current
and the galvanometer (with jockey) connected at appropriate in the circuit involving C' and R. Also I = V/R. Hence
points.
(c) After appropriate connections are made, it is found that
r=(i-1)R
no deflection takes place in the galvanometer when the sliding But E/V = l/l'. Hence

(1-1'
1,-)
jockey touches the wire at a distance of60cmfromA. Obtain
the value of the resistance X. r=R -

Solution: (a) There are no positive and negative = 4 _0 x (76.3x 60.0) = l.lO
terminals on the galvanometer because it detects current, 60.0
and only zero deflection is required.
(b) L1=~ctrn,~i'~ fii2L>
,------@- Fig. 2E.82 (a) shows a metre bridge consisting of two·
X 120
resistances X and Y together in parallel with a metre-long
constantan wire AC.of uniform cross-section. D is a movable
A J B C D contact that can slide along the wire AC.
. ,.
' 60cm 40cm -The resistors X, Y and resistances of segments AD and DC of
' the wire constitute the four .arms of the bridge. The length of
Fig. 2E.B0 (b) wire AC is 100cm Xis a standard 4.000 resistor and Y is a
coil of wire. With Y immersed in melting ice the null point is
found to be at a distance of 40.0mfrom point A. When the
(c) From the figure, AJ = 60 cm, BJ = 40 cm. :coil Y is heated to 100°C, a 1000 resistor has to be connected
The given circuit is a Wheatstone bridge, in no in parallel with Y in order to keep the bridge balanced at the
deflection mode it is balanced. same point. Calculate the temperature coefficient of resistance
X RBJ of the coil.
-=--
12 RAJ
Solution: The resistances of the two segments of the
X 40 2 wire AD and DC are in the ratio of their lengths. If R 0 is the
or -;:::;-=-
12 60 3 resistance of Yin melting ice (0°C), the balance condition of
or X =80 Wheatstone bridge gives
···•··· • ·--~-1 X kl
i~---~-'--=~·.,,
:~_xctmRt~ ! 81__i,....,...
·'> -=
R0 k(l00-l) 100-1
Fig. 2E.81 shows a potentiometer circuit for determining the where k is the resistance per centimetre of wire AC.
internal resistance of a cell. When switch S is open, the Now, l = 40.0 cm and X = 4.000. Substituting these values,
balance point is found to be at 76.3 cm of the wire. When we get R 0 = 6.000. Let R, be the resistance of Y when
switch S is closed and the value ofR is 4.00, the balance point heated to a temperature t =100°C. When it is connected in
shifts to 60.0 cm. Find the internal resistance of cell C '. ' parallel with 100 0 resistor as shown in Fig. 2E.82(b), the
net resistance becomes
Solution: Let & be C 1000
the emf of the cell C ' and r X=4.000 y X=4.000
its internal resistance. Let
B B
l = AJ be the balance
length when switch S is 1' G
open. When a resistance R
is introduced by closing "A f - - - - . . 1 - - - - - - - l C A ~---.f.'--------lC
the switch a current begins I=4,.- 60.0 cm
40.0,cm
(100-n = 40.0 cm~60.0 cm
to flow through the cell C' R s
and resistance R. The Fig. 2E.81 (a) (b)
potential difference Fig. 2E.82 .
between the terminals of the cell falls and the balance length
R'= lOOR,
decreases to I'= AJ'. The terminal resistance of the cell is
R, + 100
given by
Since the null point remains uncharged, we have

www.puucho.com
Anurag Mishra Electricity and Magnetism with www.puucho.com

40.0
X => 1--1-=_1_ _ _1_
-=--
60.0
R' l+x1 l+x1 l+l
=> R'= 6.000 1
x 1 =-m=33.33cm.
Thus 6.00 = lOOR, 3

=>
R, + 100
R, = 6.380
l;=?f:~~iJ~, j 841>
Temperature coefficient of resistance of the coil Y 'is Jn the simple potentiometer circuit, where the length AB of the
rr = R, -Ro ,potentiometer wire is lm, the resistors X and Y have values
R 0t SQ and 2Q respectively. When X is shunted by a wire, the.
6.38-6.00 'balance. point is found to be 0,625 m from A. What.'is the
6.00x 100 resistance. of the shunt? If the shunt wire is 0. 75 m long and
= 6.3 x 10-4 K- 1 0.25 mm in diameter, what is the resistivity of the mate.rial of
the wire? ·
L:S?£:-9:~Rf~J 83_l> X y

5n 2n
The wire AB ofa meter bridge changes linearly from radius r.
to 2r from left end to right end. Where should the free end of G
the galvanometer be connected on AB so that the deflection in
the galy_anom,eter is zero_? AI-----....L-----~B
4n 4n

A
Fig. 2E.84
B
Solution: LetR be the resistance of the shunted wire,
the effective resistance of Rand SQ in parallel= 5 x R/(5 + R)
At balance point,
Fig. 2E.83 5R/(5 + R) 0.625
2 1-0.625
Solution: Let the galvanometer be connected at a 0.625 5
=-
point x = x 1 from end A where x = 0. 0.0375 3
Let R1 = resistance of left part, i.e., AX1 and On solving we get,
R2 = resistance of right part, i.e., X 1B R=lOO
Length= 100 cm = 1 m. Ra Rrrr 2
Now, p=-=--
Consider an element of thickness dx at a distance x from 1 l
end A and of radius rx. lOx (22,17)x (0.125x 10-3 ) 2
TJ,us, rx =(r+ ix)= r(l+ x) 0.75
= 6.54x 10-7 Om
Resistance of this element will be,
dR =pdx . l=E'xa'\,im;~1teri85 [~>½
L i.~::::C:i:> ::_;'i;J;:;f,.,,L~iL~::::t:L< I ~ _ _ . ~
X 1tT'.2
X
You are given two resistors X and Y whose.resistances are to
J' pdx 1 ]
R, = o rr(l+x)2r2 = rrr2 1- l+x,
p [ :be determined using an ammeter of resistance 0.5 Q and a
voltmeter of resistance 20 k Q. It is known that X is! in the
R -J
2 - , (l+x)2r2
pdx
,range of a few .ohms, while Y is in the range oj'several
,thousand .ohms. In each ca.se, which of the following two
:connections .(Fig. 2E.85) would you choose Jo, resistance
mea.sur~ment? Justify your answer quantitatively._
= rr~ 2 [ 1 +\, - 1: 1]
Solution: For each connection, determine the error in
For null point of zero deflection,
R1 4 resistance measurement. The connection that corresponds
to a smaller error (for a given range of resistance) is to be
R2 4 preferred.

www.puucho.com
Anurag Mishra Electricity and Magnetism with www.puucho.com

L~ECTR!C CURRENT .. --- "


---279.1
--,-'1VV\.,.____ _-fiA'\....-- ------./V\¥-------,rlA --_---~-----------: r:=i--
L~~g,r,m]f~:~ J 87 i.>
Potentiometer wire PQ of 1 m length is connected to a
standard cell &1. Another cell, &2 of e.m.f. 1.02 Vis connected
V V with a resistance r ancj a switch S as shown in the circuit:
(a) (b) diagram. With switch S open null position is obtained at a:
Fig. 2E.85 .distance of 51 cm from P. Calculate:
oi
0

For X, use (b); for Yuse (b).


liJ potential Pradient th e potentiometer wire.
(ii) e.m.f. of cell &1.
Voltage drop across resistance and ammeter will be in
the ratio of their resistances. , (iii) when switch S is closed, will null point move towards P or.
.towards Q? Give reason f~r your answer?
Arrangement (a) is preferred for Y, whose resistance is
large as compared to ammeter resistance.
s,·.
. Arrangement (bl is preferred for X, whose resistance is
not too large as compared to ammeter resistance. Voltage
drop across the ammeter will be an appreciable fraction of
that across the resistance and must be excluded.
~-
[I
~
·_···_._
:f;~am,P/1~ ·\ 86 ,,IV--
,, ,,.,"•.. - .. -- -"~~, ..,,_,_ ,~. t >
: A cell of emf. 3.4 Vand internal resistance 3 n is connected to
;an ammeter having resistance 2 Q and to an external;
-resistance of 100 Q resistance, the ammeter reading is·0.04 A.' Fig. 2E.87
'Find the voltage read by the voltmeter and its resistance. Had;
the voltmeter been an ideal one, what would have been its' Solution:· (i) Potential gradient,
'reading. _
k = ~ = l.0 = 0.02V/m
2
Solution: Let Rv be resistance of the voltmeter. The 1 51
equivalent resistance of voltmeter and lO0V resistance (ii) The e.m.f. of cell &1 ,;,kx 100 = 0.02x 100 = 2 V
R' lOORv (iii) When switch S is closed, there is no shift in the
lO0+Rv position of null point as the position of null point depends
_upon .the potential gradient along the potentiometer wire
Net resistance of circuit, Rnet = R' =RA + r
(which depends upon the e.m.f. of battery &1 and resistance
=R'+ 3 + 2 = lOORv + 5 of potentiometer wire circuit and length of potentiometer)
lO+Rv and e.m.f. of the cell &2 which does not change when switch
-E 3.4 S is closed.
Current in circuit, I=-
Rnet l00Rv + 5
lOO+Rv Lll5>f~m~I.~.Jasl>
But I= 0.04A,
0.04= 3 .4
' In Fig. 2E.88, AB is a 1 m long uniform wire of 10
'resistance. Other data are shown in the diagram. Calculate:,
n:
lOORv + 5 (a) potential gradient along AB, .(b) length AO when:
'lOO+Rv ,galvanometer shows no deflection.
Solving we get, Rv =400Q 2V 15Q
Reading of voltmeter, V =IR'= 0.04x lOORv
lOO+Rv
0
= 0.04x lOOx 4 oo = 0.04x 80 = 3.2 V ' A 8
' 100+400
Ideal voltmeter has infinite resistance. In that case net
1.2 V
resistance of circuit,
R~et =100+ 3+ 2 = 105Q 1.5V
Current I' = E
--= 3.4A
- o.an
Rnet 105
3 Fig. 2E.88
New voltmeter reading V' = I'xR = .4 x 100 = 3.24 V
105

www.puucho.com
Anurag Mishra Electricity and Magnetism with www.puucho.com

[.2so
Solution: (a) Potential gradient along
2
) to = 0.008 v cm -r
AB = ( • "~"",.'-.;"'·-·. -- I
15+10 100 '1n the given circuit, a meter bridge is .shown in a baia~~ed:
5 ·state The bridge wire has a resistance of 1 Q/cm, Finilthe;
(b) Current through 0.3 Q = 1. =l A
' 1.2+0.3 value of Hie unknown resistance X ·and the' current::drown!
Potential difference across 0.3 0•= 1 x 0.3 = 0.3 V 'from the battery of negligible intemal·resistance, ·'
>'"~"" ,.,,, ••
' " • • • • ·- ,• ·•-

Let l be the length AO•then 0.3 = 0.008 x 1


X 6Q
or l = ~ = 37.5 cm
0.008 A.__ _ ___,..,._ _ _ _ ___.B
J 1,
-- -·- --~-· . -- " - . - - .. --. '

i Cells. A and,B and a galvanometer'G are connected to a slide I '' ~----1~1----~~


/wire OS by two sliding contacts .C anq D as shown in. Eig. i 6V
:2E.89. ,The slide wire is 100 cm
long.and .has a resistance of, Fig. 2E.90
1 12:n. WithOD = 75 cm, the galva11ometer gives no deflectjon 1
!when0Cis
I -
50cm. IfD is
·-
moved.to
. '
touch
:'·
the end ofwireS,.,
the,' Solution: For the balanced bridge,the ratio of the two
1value.of0Cforwh1ch the galvanometer shows no deflection is: resistances is equal to the ratio of the lengths. of the two
!62.5 cm. The,e.m.f. of cel1 Bis 1.0,V. · ; parts Al and jB of the wire i. e.,
.Calculate:· · . , •: X 40cm
i• . • ' _·, -=--orX=40
,(i) The potential difference acn~ss O and·D whenD is at- ?5 cin, 60 60cm ·
:markfromO.. ·. · · ·. · · . No current flows through the galvanometer G, th_e
:(it) The potentiql difference across QS when D touches S. resistance of the parts Al and JB are 40 0 and 60· 0,
)(iii) Intemal/isista11ce'of cell A: ·· · ' · respectively. If R be the equivalent resistance between the
{iv)The
' e.m.f.
., of cell A points A and B, then we have
A
-~ = 1 + __1_ _
R (X + 6)0 (40+ 60)0

'
;o 7 D
s1
or R = 100 0
11 '
. I=~=
R (100/ 11)0
5V = 0.66A

'' l,''E
-

I
»
,., "
,
l
. :-;:~.~M'~~-g J 91
~ ,_
.">'-.'"'
-~~'' M-,7+1TT:ER1L. ,, ",,, ,.
.
F~·-
· ,,,, ·

. • " ••
Fig. 2E.89·
. ·'"'"'"~"""""""""~ " ___ ,..,. _ .. _,.,..,,.,,,- . .--..-,,-... - ; A voltmeter reads 5.0 Vat full scale deflection and is'graded,
!according to its resistance per volt·atfull scale deflection, as,
!5000 WV. !fow will you convert iti11to a voltmeter that reads
12
Solution: Resistance of wire OD= x 75 = 90. Let ;20 V at full scale deflection? Will it still be graded. tis '5(/00,
100 p,IV? Will you prefer this voltmeter to one that is grade&.cis,
S and r be the e.m.f.. and internal resistance of cell E. '20000/V? · . . ,' ·
• --· ~ ,f ' • -· - • • • • • •,,_ ••• ,-,,·-----~--,.-- .. )
(i) Potential gradient of wire = 1/50 V/cm. Therefore
voltage drop across the wire OD of length 75 cm Solution: Resistance per volt at full scale deflection =
= (1/50) x 75 = 1.5 V 5000 0 v- 1
(ii) Potential'gradient of wire= 1/62.5 V/cm. Therefore Reading of voltmeter at full scale deflection = 5 V
voltage drop across the wire OS of length 100 cm = (1/62.5) Resistance of voltmeter G = 5000 x 5 = 250000:
x 100 = 1.6 V Also current for maximum deflection,
(iii) (_§_)x 9+r
9 =1.5 ... (i) = lV =0.0002A
• 50000
I

12+r (_§_)x
12 = 1.6 ... (ii)
Range of voltmeter to be changed to V = 20 V
NOW,
20
(iv) On solving eqn. (i) and (ii), R=~-G·= 25000
we get r =30 and S =2V r. 0.0002
= 100000-25000 = 750000

www.puucho.com
Anurag Mishra Electricity and Magnetism with www.puucho.com

g•,~··~w-•-• •

281

Thus, 75000Q' resistor is to be connected ih series.


Resistance of 20 V voltmeter
= 75000 + 25000 = 1,00,0000
. 100000
Its gradmg becomes = - - - 50ooov-1 which is
20 ,An experiment with a post office box, .the ratio of arms are
same as in,the earlier case. A voltmeter with grading 2000 0 ,1000 : 10. If the value of the third resistance is 999 n, find
v-1 will have less resistance and is therefore not preferred. ,the unknown rrsistanse.
[t1Ji~~m~.·1~,s 927'>
(§?~~~=-- ~ ~ ~ " ' ' ' ' ' ~ ~
. - .
Solution: In the given case, the ratio arms are 1000 :
10
•A battery of e.mj. 1.4 V and internal resistance 2 0 is, ~ = 1000 =100
:connected to a resistor of 100 0 resistance through an 10 Q
,ammeter. The resistance of the ammeter is 4/3 n. A voltmeter,
Third resistance, R = 999V
;has also.been connected to find the potential difference across•
Let Xbe the unknown resistance. Then,
the resistor. ·
(a)·Drq"I the circuit diagram. ~=R or 9.xR
Q X P
(b) The. ammeter reads 0.02 A. What is th,e resistance of the,
1 1
voltmeter? · , =--x 999 = 9,990
/c) The voltmeter reads 1.1 V. What is the error in the 100
:reaqirJg? _____ ..... _____ 1
L"Exn:r,m,h'l~~r94
_--=·--~-
. ,1,,,"""""""'"""'',:s;'.~%C,'•
.,~,_.,;:;.;P&n triF"''"'11'
t·-.,.' -,,.
......... -
'~
Solution: (a) The circuit diagram "is shown in Fig. ! • e,w • • •

2E.92 'Apotentiometer wire has a length of 10 cm and resistance 4


!n_ An accumulator of e.m.f 2 V and a resistance box are
1.4 V 2Q •connected in series with it. Calculate the resistance to be
,introduced in the box so as to get a potential gradient of (a)
,0,1 Vim and (b) 0._1 mV/m.
B 100n C
Solut!on: Current in_the potentiometer wire is given
by
I= &
L..--..-{V1----' R+lxp
.... J:ig. 2E.9:; where pis the resistance per unit length of the wire and 1
is the length of the wire.
Cb) Let resistance of the voltmeter be R ohm. The
Now, potential gradient
equivalent resistance of voltmeter (R ohm) and 100 0 in &p
parallel is V=lp
lO0xR lO0R R+ lx p
lO0+R lO0+R Here, l =lOm, p = 40/ m
Resistance of the ammeter = ~ 0.
(a) For V = 0:l V/m, we.have
3 0.l = 2x 4 = - 8 -
Total resistance of the circuit = lOOR + ~ + 20
R+ lOx 4 R+40
lO0+R 3 or R =_±_ =400
Current in the circuit as read by the ammeter = 0.02 A 0.1
Cb) For V = 0.1 mV/m = 0.1 x 10-3 V/m,
Now, 0.002 l.4 (·.- I = _RV)
lOOR +~+ 2 We have
IO0+R 3 0.lx 10-3 = 2x 4
or R =2000 R + lOx 4
Resistance of the voltmeter = 200 0 or 10-4 = - 8 -
(c) Effective resistance between Band C R+40
= lO0x 200 = 200 0 or R =799600.
100+ 200 3 Note: There is no current through the cell and galvanometer,
The potential drop across this resistance battery E, internal resistance rand potentiometer wire AB
200
= circuit current x 200 = 0.02x = ~V = 1.333V are in series.
3 3 3
Reading of the voltmeter = 1.1 V
www.puucho.com
,..,J
Anurag Mishra Electricity and Magnetism with www.puucho.com

1282 . -ELECT~ICllY & MAGNETISiD


------·----- -

t ~~\~mt:11t7 .r-~ And as here, G = 99!1


and I a = (10/100)[ = 0.ll
Fig. 2E.95 shows a 2.0 V potentiometer usert for the S = O.ll x 99 = 0.1 x 99 =11!1
.determination of internal resistance of a 1.5 V cell. The· (I -0.ll) 0.9
. :balance point of the cell in open circuit is 76.3 cm, When a
resistor of 9.5 n is used in the external circuit of the cell, the
balance point shifts to 64.8 cm length of the potentiometer
,wire. Determine the internal resistance of the cell. A galvanometer has a resistance of 30 n and a current of 2
mA is needed for a given full scale deflection. What is the
resistance and. how is it to be connected to convert the
2.0 V galvanometer: (i) into an ammeter of 0.3 A range, (ii) into a'
voltmeter of0.2 V range?
Solution: As here galvanometer resistance G = 30!1

''
:'
'
1.5 V

9.50 ,
G
and full scale deflection current I g = 2mA, so,
(i) To convert
galvanometer into an ammeter
the

of range 0.3 A, a resistance of


value S is connected in parallel
..-,,~ s

_

. _
t (/-/.) ,
~
·:J
1---------.../\1\/'1/'----- .. with it such that j j G •
Fig. 2E,95 (I -I g )5 = I ,G 9
Ammeter
(0.3 - 0.002)S = 0.002 x 3, Fig. 2E.98 (a)
Solution: Internal resistance of a cell using or, S
= 0.002x 30
potentiometer is given by, 0.298
r =Rx 1, -12 = 0.2013 n
'12 (ii) To convert the galvanometer
Here R = 9.5!1, 11 = 76.3 cm, 12 = 64.8 cm
Hence r--95 . x76,3-64.8_95

I-····s~,A;~;1;>if;;;_
- - - - - . x11.5_17"
64.8
f'"%] ··.>
- - - , ,,
64.8
into a voltmeter of range 0.2 V, a
resistance R is connected in series with
it such that

i.e.,
V =I, (R +G),
0.2 = 2x 10- (30+ R)
3
c:J I
g Voltmeter
Fig. 2E:98 (b)
..., ··'-·· . . ·L__, ~
i.e., R = 100 -30 = 700
A galvanometer has a resistance of 50 n and its Juil scale
,deflection current is 50 µ A What resistance should be added
_to it so that it_ can have a range of 0 - 5 V?
L~~,~~P:l~ 19~
Solution: Here, the maximum value of I a =10µ A. The scale of a galvanometer is divided into 150 e1jual
The upper limit gives the maximum voltage to be measured ·divisions. The galvanometer has current sensitivity of 10'
which is V = 5 V. The galvanometer resistance, G = son. divisions per mA and a voltage sensitivity of 2 divisions per
mV. How can the galvanometer be designed to read ·
From the above relation, Rh = lO0kn
(i) 6 A, per division and_ (ii) 1 v; per division?
If we work out, we would understand that higher the
range of voltmeter, higher is the value of shunt resistance. Solution: As per the resistance of galvanometer,
., '
·Example ,!i 97
.---,,. G = Full scale voltage
h.'£,.,,:i.:c"coJ"'''':.::""-. _J~
1 :-..
Full scale current
3
What is the value of shunt which passes 10% of the main; = 75x 10- = SQ
cu_rrent through a galvanometer of 99 ·!1? lSx 10-3
For conversion into ammeter of range I A.
Solution. A shunt is a small . s -
resistance, S,in parallel with a

.CJP
galvanometer (ofresistance G) as V---~
shown in Fig. 2E.97
(I-Ia)S =la xG,
S = IaG R
z.. e.' Fig. 2E.97 (a) (b)
(I-Ia)
Fig, 2E.99

www.puucho.com
Anurag Mishra Electricity and Magnetism with www.puucho.com

(I-I )S=I'G And as this current produces a deflection of 50 divisions


g G 1 in the galvanometer,
S=-g- .. . 8 S0div 1 div
I -Ig Current sens1t1V1ty = - = - - - = - - .
I lO0µA 2µA
lSx 10-3 x 5
(lS0x 6-lSx 10-3 )
..· - .• . . . ·. r--·1· .
I Ex:am,e·Le •I 102 . ~
Lt¾!.,!::¥.itZ·:~.Ef..SE'il-~,::::?'·<~;;_irt;~.... --J~
lSx 10-3 x 5
= 150x 6 '.Consider the potentiometer circuit arranged as in the Fig.
2E.102 (a). The potentiometer wire is 600 cm long.·
= 8.3x 10-sn r
For conversion into voltmeter of range V volt,
I~ (G +R) =V A._______,,._
15 r _ _.

fuJr~~~~~ij:J:~ .. ···-·· - . '


The deflection is a moving coil galvanometer falls from 50;
,divisions to 10 divisions when a shunt of 12 ohm is applied.·
E/2
What is the• resistance
'* ~- """
of the galvanometer?
w•'·-'',-·••-
Fig. 2E,102 (a)
Solution: In case of a galvanometer, I cc 8 ,(i) At what distance from point A should the jockey touch the
[G 10 1 wire to get zero deflection in the galvanometer?
So,
r=so=s' :(ii) If the jockey touches the wire at a distance of 560 cm from
. 1
i.e.' IG =- I .!>,. wh_at will be the current in ?hf_galv_a_nometer?
5
Now in case of a shunted Solution: (i) When the jockey is not connected.
(1-IG)
galvanometer as shown in Fig. [=.E...

,J:::i.
2E.100, 16r
(I -IG)S =IGG Resistance per unit length;
1 1 15r
i.e., (I--I)xl2=-IG 1'.=-nlcm
5 5 i 600
Fig. 2E.100
Let l be the length when we get zero deflection.
S)=(1'.l)S =.E...x 15r xi
( 2 2 16r 600
·A galvanometer ofresistance 95 Q, shunted by a resistanc.e ofj l =320cm
5 n gives a deflection of 50 divisions when joined in seriesl S X-E
X r-o-11--+--N\N-~
'with a resistance of 20,n and a 2 V accumulator. What is thej
current sensitivityof the galvanometer (in div/µA)? ,
a
X I--./V',/\l'--..-N\N--4
@
E/2
20 kn x· L-H-+--N\N--'
,,____-1.,Gi)--L---./1/V'./'-----, x-E/2
95Q Fig. 2E.102 (bl'
iI
., '
(ii) Let potential at A is zero
2V
Then apply Kirchhoffs first law
j -0 x-s
.
.. -~-.. ~~----·Fig.2E.101
--~---·~--. • ••- ••••oM> j _x_-_o + -~2~_ + _(x_-_S_-_O_) =0
Solution: In accordance with the given problem, the 14r r· 2r
situation is depicted by the circuit diagram in the Fig. 14S
X=--
2E.101. As 20 k n is much greater than the resistance of 22
shunted galvanometer ( < SQ), the current in the circuit will
be I
x-s
=--2 =
(14z)_s
22 2 =-3S_
2 r
I 10-4 A=l00µA g r 22r
20x 103

www.puucho.com
Anurag Mishra Electricity and Magnetism with www.puucho.com

1284

'One of the circuits for the measurement of resistance bj 'For the arrangement of the potentiometer shown in the Fig.
potentiometer ,is shown. The galvanometer ls connected ·at, '2E.104, the balance point is obtained 'at a distance 75 cm
point A and zero deflection is observed at length PJ = 30 cm., ifrom Awhen the key k is open..
In second case the secondary,cell is changeci\. >·'. ·\ : '.The second balance point is obtained at 60 cm from A when
Take S,=.10V and r=illinJ:st reading·.-,· · '_. ''.,· ., the key k is closed. Find .the internal resistance (in n) of the
and S/=5Vand r=2.Qin2 nd reading: '·•. ·.· '-i ,battery e,.
In second case, the zero deflection is observed at length F'J ,;,
'10 cm. What is th_e resistance ~- (in ohm) is?
E 0 =2V
Ep
l--{•)---'111/\,---- ... ,,
• : < '
.,;
. -~· 'i •.:
. ~ ,,,

R
6Q
' ',.' Fig. 2E.104 (a)

E,
' t ~~ :t Solution: Let"- is resistance per unit length of wire
Fig. 2~. 103,
-- ''
'·:_-;,."'f
- ' - -,-
-"-,.-·.;,·;:" .. -·- AB. When k is opened
' ,
V I R I R D·
Solution: - =1- =2- E 0=2V
C
1 30 10
(1) Ss, =1 1 (r1 +R) , ..
=> 1O=I1 (1+R) => I 1 = ~ ' ,
l+R,t
(2) Ss, =I 2 (r2 +R)
=> 5=1 2 (2+R)' 6Q
5 . ,, .,_., Fig. 2_E.104 (b)
=> 12 = - -
2+R
[(AX,)=&, ... (1)
lOR SR
k is closed
0l+R)x30 1Ox(2+R) IAX 2 =&1 -Ir ,. , (2)
=> 4+2R=3+3R
=> R=lQ I=_§_ ... (3)
' ' R+r
0 75
=> r=(~-l)R=( · -1) 6
X2 0.60
· r=LSn

www.puucho.com
Anurag Mishra Electricity and Magnetism with www.puucho.com

"
- - - ---···-··-_ _ _ __,,,2=85~

·,

•--- - -----~~ -·=~- •· ' - ·- - ',_.,_,,w~'li.;:·"'-,,·_•. ' - · - · - - - ' • - - • ---'"--'·-'•~'·"

Pevel
1. In the given network, the · - · · 40· · '\ (a) 5.85 Vacross each coil and 9.2 Vacross the battery
equivalent resistance -_ B -..-----'W-tv--.,..,! (b) 5.85 V across each coil and 12 V across the battery

·am
between A and B is: .,1 (c) 3.5 V across each coil and 8 V across the battery
(a) 6fl ! i (d) 3.5 V across each coil and 12 V across the battery
(bJ 16 n ! A., .. L...~vv,'l'v-...::::::J I 6. Maximum power developed across >76 v - - ----
(c) 7fl I 3Q vana. ble resistance
. R'm the crrcu1t
.. " ',
CdJ sn shown in figure is: · • 1 R •J
2. In the circuit shown in the ;· · v • -m-- --.---77 (a) SO watt ~ i
figure, power developed I ---N=-~ :,1 (b) 75 watt ' ;
across 1 n, 2 n and 3 n , 1 3
n ii (c) 25 watt
.
resistance . the ratio:
are 1n . ;' '..., ... "j
"c1 (d) 100 watt 1Q..V. __ 1.i:1.. ;

~~-!:~:; 7 I_ --~-2n _____ , _ J 7. Two wires of same -dimension but


resistivities p 1 and p 2. are connected in series. The
(c) 6: 4: 9 equivalent resistivity of the combination is:
1

r1~· .
(d) 2: 1 : 27 (a)p 1 +pz Cb)-(p,+pz)
3. A galvanometer of resistance 20 fl gives full scale 2
deflection when a current of 0.04 A is passed through (c) ~P1P2 , , (d)2(p,+pz)
it. It is desired to convert it into an ammeter reading
upto 20A. The only shunt available is 0.05 fl s. ""'b_,., """"'same""'·£
but having different 'internat ·• S
1
s:]·_.
2
,, .- .i
resistance. The resistance that must be connected in resistances r1 and ·r2 ( < r1) at" 1• '' ' : ,
series with the coil of the galvanometer is: connected in series to an external · r· • · · · . .· ,
(a) 4.95 n (b) 5.94 n resistance R as shown in figure. !___ ~_· R .•• - .
(cJ 9.45 n Cd) 12.62 n For· this ·situation mark out the
0

4. The length of the potentiometer wire is L. A cell of emf c~rrect stat~ment(s).


Eis balanced at a length!:. from the positive end of the (a) ()nly qn_e value of R exist for which potential
3 _cjiffererice across battery having internal
wire. If the length of the wire is increased !:.. At what resist~hce r1 is zero.
2 (b) Only one va!ue of R exist for which potential
distance wiil the same cell. give a balance point: difference across battery having internal
(a) 2L (b) !:. resistance r2 is zero.
3 2 (c) No value of R exist for which potential difference
(c) !:_ (d) 4L across any of the battery is zero.
6 3· (d) For all value of R potential difference across both
5. The terminals of a battery of emf 12 V and negligible the batteries would be zero.
internal resistance are connected to two coils, each of
resistance son connected in series. A voltmeter of
resistance l000fl is connected first across the coils and
then across the terminals of the battery. The reading.in
three cases are:

www.puucho.com
Anurag Mishra Electricity and Magnetism with www.puucho.com

i-- -· ............
L286 __ _
9. Potential difference across the 10V r=10 15. The potential difference across the terminal of the
terminals of the battery shown battery is 10 V when there is a current of 3 A in the
in the figure is: battery from negative to positive terminal. When the
(a) 8 V current is 2 A in the reverse direction, the potential
(b) 10 V difference becomes 15 V. The internal resistance of the
(c) 6 V battery is:
(d) zero 40 (a) 2.5 o (bl 5 o
10. As the switch S is closed in the (c) 2.83 o Cdl 1 o
circuit as shown in the figure, current passed through 16. In the circuit shown in the figure, if a wire is connected
it is: between A and B. How much current will flow through
(a) 4.5 A the wire:
(b) 6 A (a) 5 A
(c) 3 A (b) 10 A
(d) zero , A
3
11. In the circuit shown in the figure: (c) 20 A
(a) Current passing through 2 0 3 120 B 60
resistance is zero (d) ~A
(b) Current passing through 4 0 3 . a_ov
resistance .is 5 A · 17. n identical cells are joined in series with two cells A
(c) Current passing through 5 O 40 20\( 40' and B with reversed polarities. EMF of each cell is &
resistance is 4 A '
I
and internal resistance is r. Potential difference across
(d) All of these , cell A or B is (n > 4) :
12. A tetrahedral is consisting of 6 A · ··• · ;
identical wires as shown in figu're. i~ - I . (a) 2&
n
Each wire is having a resistance of 1 · I
20. When an ideal cell of emf 2 V is c (c) 4&
1
connected across AB, as shown then · · n
0 :
the current through CD is: B I 18. The resistor in which
(a) 1 A · (b) ~A maximum heat will be
19 produced is:
(c) 4A (d) zero (a) 60
13. A cell of internal resistance 1 O is connected across a (b) 20
resistor. A voltmeter having variable resistance· G is (c) 5 o
used to measure p.d. across resistor. The' pfot of (d) 40
voltmeter reading V against G is shown. What'is value 19. In the shown i A· 4g··gy· 10 3y· 4a· .. s
l
circuit, potential ; ~~~ ;

20Vtz=
of external resistor R? · ·
difference ·. . l......vM,-J I
---<VJ---,
between points A '
20 . . .....•. '
·vf10v _ . i and B is 16 V. The current passing through 2 0 resistor
1----./\/\/\I'--,.-,_ j
5V R will be:
(a) 2.5 A (b) 3.5 A
G(O)-
24V (c) 4 A (d) zero
(a) 5 o Cbl 4 o 20. In the shown circuit, the ; ·· · · · JR s; • · 1
(c) 3 o (d) I 0 reading of the voltmeter · - · ~ , R -~. -!
14. A circuit is arranged as shown. Then, the current from is V1 when only S1 is 1 6R S :

AtoBis: closed, reading of I v :


,. - ·---- ,- fl,: voltmeter is V2 when · · I
only S 2 is closed and its : . .1 .
100 ~-- reading is V3 when both ; · · - &.. .. .. . . .J
'

+ S1 and S 2 are closed. Then:


•10V 100 . J.:-sv'
f • (a) V3 > V2 > V1 (b) V2 > V1 > V3
(c) V3 > V1 > V2 (d) V1 > V2 > V3
B
(a) +500 mA. (b) +250 mA.
(c) -250 mA. (d) -500 mA.

www.puucho.com
Anurag Mishra Electricity and Magnetism with www.puucho.com

·-·7
ELECTRIC CURRENT 2.BJJ
21. For what value of R in the circuit 26. A galvanometer of resistance Ra is to be converted
as shown, current through 4
will be zero:
(a) 1 n
Cb) 2n
n
2Qrnn R
.
,-l';
6V
into a ammeter, with the help of a shunt of resistance
R. If the ratio of heat dissipated through galvanometer
and shunt is 3 : 4, then:
3
(a) R =-Ra
4
(b) R =-Ra
(c) 3 n 10V 4V 4 3
Cd) 4 n 9R (d) Ra = 16R
(c) Ra=-
22. In the circuit shown in the figure, 16 9
when switch S 1 is dosed and S 2 is C: 27. A circuit is comprised of
open, the ideal voltmeter shows a e "' eight identical batteries and
reading of 18 V. When switch S 2 is r s, s, a resistor R = 0.8!1. Each
dosed and S1 is open, the reading battery has an emf of 1.0 V
of the voltmeter is 24 V. When both and internal resistance of
S1 and S 2 are dosed, the reading of the voltmeter will 0.2!1. The voltage difference
be: across any of the battery is:
(a) 14.4 V (b) 20.6 V (a) 0.5 V (b) 1.0 V
(c) 24.2 V (d) 10.8 V (c) 0 V (d) 2 V
23. A circuit consists of a battery, a resistor Rand two light 28. In order to determine the e.m.f. of a storage battery it
bulbs A and B as shown: was connected in series with a standard cell in certain
R circuit and a current I 1 was obtained. When the
battery is connected to the same circuit opposite to the
standard cell a current I 2 flow in the external circuit
A B
from the positive pole of the storage battery was
obtained. What is the e.m.f. of the storage battery?
The e.m.f., of the standard cell is &2 •
(a) &1 = I, +I2 &, (b) &1 = I, +I2 &,
If the filament in lightbulb A burns out, then the I, -!2 I2 -Ii
following is true for light bulb B: 11 -! 2 1 2 -1 1
(a) it is turned off (c) & 1 = - - &2 (d) & 1 = - - &2
(b) its brightness does not change ! 1 +! 2 ! 1 +! 2
(c) it gets dimmer 29. A wire of cross-section area A, length L1 , resistivity cr 1
(d) it gets brighter and temperature coefficient of resistivity a 1 is
24. The resistance betweenP and Qin the shown circuit is: connected to a second wire of length L2 , resistivity cr 2 ,
R · te,mperature coefficient of resistivity a 2 and the same
R R
area A, so that wire carries same current. Total
resistance r is independent of temperature for small
p Q
temperature charge if (Thermal expansion effect is
R negligible)
R (b) 2R (ii) a, = -a 2
(a) -
2 5 (b) cr1 L1 a 1 + cr 2 L2 a 2 =0
(c) 3R (d) ~ (c)L1a1+L
(d) None
. 2a 2 =0
5 3
30. The battery in the diagram is to
25. Current in 3 n resistance is :
(a) 1 A
(b) _!A
7
5
(c) -A
7
be charged by the generator G.
The generator has a terminal
voltage of 120 volts when the
charging current is 10 amperes.
The battery has an emf of 100
volts and a internal resistance of
100v. m
L:]
1 ohm. In order to charge the battery at 10 amperes
(d) 15 A
charging current, the resistance R should be set at:
7
(a) o.in (b) o.sn
(c) 1.on (d) 5.on

www.puucho.com
Anurag Mishra Electricity and Magnetism with www.puucho.com

r.:;:- ····=·
t~BB. < ----~---···-··. ____ :· ;• ·: ELECTRICITY·&llm:GNEJISM I
31. ABCD is a square where each side -,·,10~ ··•· (a) 10 V/m (b) 1 V/m
is a uniform wire of resistance ill. A ~ B , (c) 0.1 V/m (d) none
A point E lies on CD such. that if a .· I 37. An ammeter A of finite resistance, r:···,· ci··:··: ·,
uniform wire of resistance ill is 11~ . 1n E . ~ 1
connected across AE and constant I and a resistor R are joined in series
to an ideal cell C. A potentiometer P i
r~---
·_. R ~
l
, D C
potential difference is applied ___ 1!l ...... ,. ~ is joined in parallel to R. The i A ,.
across A and C then B and E are ammeter reading is I O and the :. · ·· . P
equipotential: potentiometer reading is V0 • P is .'...c.. . .:-.. •... ···•••
(a) CE= 1 (b) CE= 2 now replaced by a voltmeter' of finite resistance.Tlie
ED ED ammeter reading now is J and the voltmeter reading
CE 1 (d) CE =../2
(c) - = - is V:
ED ../z ED (a) I >1 0 ,V <V0 (b) I >1 0 ,V=V0 ·
32. Power generated across a uniform wire connected (c) I =1 0 ,V <V0 (d) I < I 0 ,V = Vo
across a supply is H. If.the wire is cut into n equal parts 38. In the arrangement
and all the parts are connected in parallel across· the shown in figure when i ,,
same supply, the total power generated in the wire is: the switch S2 is open, ;
2 the galvanometer,shows ·
(a)-~ (b) n H
n no deflection for
(c) nH (d) H l =LI 2 When the
n . switch S2 is closed,the
33. When electric bulbs of same power, but different galvanometer shows no
marked voltage are connected in ·series across the deflection for 1 = SL/ 12.
power line, ·their brightness will be: The internal -,:esistance
(a) proportional to their marked voltage. (r) of 6 V cell, ru;i\i the
(b) inversely proportional to their marked voltage. emf Z of the oth~r battery ·.are respectively:
(c) proportional to the square of their marked (a) 3n, s v (bl _2n, 12 v
voltage. (c) 2Q, ;!4 V (d~ 3,f.!, 12 V
(d} the same for all of them. ·39. A wire has a non-uniform , - ....,,,·-···---.
34. A galvanometer coil.has a resistance 90Q and full scale cross-section as shown in fi~re. A ; · , · ·· ,. , ·i · j
deflection current 1Q mA. A 91 on resistance is steady current flows through it..,-.-_ ·. ;p:,c;~v- ··: --:
connected in series with the galvanometer to make a The drift speed of .e\ectrons at IL. i.. ' ~'.' . 1
voltmeter. If the least count of the voltmeter is 0.1 V, pointsP,andQisvp ;mdvQ. ......,...,-.•·--·1
the number of divisions on the scale is: (a) Vp = VQ (b) aVp < VQ
(a) 90 · (b) 91
(c) v 1 > vQ (d) Pata inswficieqt
(c} 100 · (d) none
40. If X, Y and Z in figure are igen.tical
35. In the figure shown for gives values of R1 and R2 the
balance point for Jockey is at 40 cm from A. When-R2
is shunted by a resistance of 10n, balance shifts to 50
lamps, ,which of the following
changes .to the brightnesses -of the
lamps occur when switch S is
· .. . -~,.,_~-_.·]
(AB =1. m)
.
cm. R1 And R2 are: closed? · ~ •. :..:. ' . y.
R1 R;·. (a) X stays .the same, Y d.ecreases
(b) X increases, Y decreases
(c) X increases, Y stay~ ·the same
(d) X decreases, Y increases
I A B 41. A battery consists of a variable number n ..of identical
.__ ___, .__ ___, !' .cells having internal r_esistance connected in series.
-W-M•--'•~- _,
The terminal of the battery are short circuited ,1md the

~v"""'"·,-r9r
O, - ···--·

1 current I measured. Whi~h one of the graph below,


(a) °n, sn Cb) 20n, 30n
3
15
Cc) 10n, 15n Cdl sn' n
36. A potentiometer wire has length 10 m and resistance
2
(a) b~
I .
l
I
Cb) i~ . · . 1
l .• '
lOQ. It is connected to a battery of EMF 11 volt and (?,~. _ --·-- ._n__, ;.a ._. ~.,:~:., n
internal resistance ill, then the potential gradient in
the wire is:

www.puucho.com )
Anurag Mishra Electricity and Magnetism with www.puucho.com

. . ..... ··:·7
ELECTRIC CURRENT ---- ··--- 289:1

(c) ~t / :
OLL.__n;
(d) :~t-----
-OL_n
length 2L. The temperature of the wire is raised by the
same amount t,T in the same time t. The value of N is:
(a) 4
(c) 8 (d) 9
(b) 6

(e) :~tr
'.QlL._n
: 46. The equivalent resistance
between A and B is :
(a) 32.5 Q
(b) 22.5 Q
Sn
15n

10n 1
I

(c) 2.5 Q 20!1 100:


42. In previous problem, if the cell hand been connected in (d) 42.5 Q i
parallel (instead of in series) which of the above 47. Two current elements P 30n 40n
graphs would have shown the relationship between and Q have current voltage
total current I and n: characteristics as shown below:
. ·,
(b) ~t /
cru ~ " · oL-..n : 1

(c) : )
1. 0LL._n
/ :~t--- .
(d)
10 P.O. (volt) 10
. --
P.O .. (volt) '
• !

.
OL_n Which of the graph given below represents current
voltage characteristics when P and Q are in series.

(e) :~fr
,9lL._n (a) - " . .
·f16L---------
: '
' 0 20
P.O. (vol!)
43. Three identical resistors are
connected across a voltage

'Le-·
source V so that one of them is . . .
in parallel with two others 1
2 ' .•
'"ii' :
which are connected in series ' ,1-v_ ___, 1.E1 : l
as shown. The power dissipated
through the first one, compared to the power
(c) J1'---------:
I ~
. .10 ~ 20
(d) ;:~
, !
:
0
.
20 ,
P.D.(voll) P.D.(volt)
dissipated by each of the other two, is approximately:
(A) the same
(b) half as much
(c) twice as much
(d) four times as much (e)
i_J-·-~
=ILL· .
10 21)
44. In the diagram shown, all the P,D,;{Volt)
wires have resistance R. The
equivalent resistance between 48. In the given circuit, the quantity of
the upper and lower dots shown charge that flows to ground long
in the diagram is: time after the switch is closed is:
(a) RI 8 (a) 12 µC
(b) R (b) 9 µ C
(c) 2R/ 5 (c) 13 µC
(d) 3R/ 8 (d) zero
45. A wire of length L and 3 49. For what value of R the thermal power developed in it
identical cells of negligible internal resistance are is maximum:
connected in series. Due to the current, the (a) R1R2
temperature of the wire is raised by /J.T in time t. N R1 +R 2
number of similar cells is now connected in series with (b) ~R1R2
a wire of the same material and cross section but of (c) R1 +R 2
(d) R1 -R 2

www.puucho.com
Anurag Mishra Electricity and Magnetism with www.puucho.com

r-·« -

·-· ..
1290
50. The circuit diagram shown consists of a-large·number (c) always goes from the negative terminal to positive
of element (each element has two resistors R 1 .andR 2 ). terminal
The resistance of the resistors ion each subsequent (d) does not move
element differs by a factor of K = ½ from the 55. 1\vo bulbs rated (25 W - 220 VJ and (100 W - 220 VJ
resistance of the resistors in the previous elements. are connected is series to a 440 V line. Which one is
The equivalent resistance_ between A and B .shown in likely to fuse?

E
Fig. is: (a) 25 W bulb (b) 100 W bulb
2 3 4' '
56.
(c) both bulbs (d) none
In the circuit shown the cells are _.· ··
A

B
~-00.
~

. .
ideal and of equal emfs, the
capacitance of the capacitor is C and .
the resistance of the resistor is R. Xis .
7
_J z
y;

first joined to Y and then to Z. After a ' ,


(a) R1 -R2 long time, the total heat produced in B - • . ;
2 the resistor will be:
(b) (R1 - R 2 ) + ~ (a) equal to the energy finally stored in the capacitor
2 (b) half of the energy-finally stored in the capacitor
(c) twice the energy finally stored in the capacitor
(R 1 -R 2 ) +JR~+ Ri + 6R 1R 2 (d) 4 times the energy finally stored in the capacitor
(c) - - - - - - - - - -
2 57. 1\vo non-ideal batteries are connected in parallel.
( d) None of these Consider the following statements :
51. In the circuit shown in figure reading of voltmeter is V1 (A) The equivalent emf is smaller than either of the
when only S 1 is closed, reading of voltmeter is_· .V2 two emfs
when only S 2 is closed. The reading of voltmeter is V3 (BJ The equivalent internal resistance is smaller than
when both S1 and S 2 are dosed then: either of the two internal resistances
3R (a) Both (A) and (BJ are correct
R s, (b) (A) is correct and (BJ is wrong
(c) (BJ is correct but (A) is wrong
S2 (d) Both (A) and (BJ are wrong
V 58. In the circuit shown the ·R•
A
resistance of voltmeter is
& 10,000 ohm and that of
ammeter is 20 ohm. The
(a) V2 > vi
> V3 (b) V3 > V2 > V1
ammeter reading is 0.10 amp
(c) V3 > ½ > V2 (d) ½ > V2 > V3
and voltmeter reading is 12 volt. Then R is equal to:
52. A metallic resistor is connected across a battery. If the
(a) 1220 (b) 1400
number of collisions of free electrons with the lattice is
(c) 1160 (d) 1000
somehow decreased in the resistor (for example
cooling it), the current will : 59. By error, a student places
(a) Increase (b) Decrease moving-coil voltmeter (V) nearly R=12V,r=2Q
(c) remains same (d) become zero ideal in series with the resistance
in a circuit in order to read the 4Q
53. A constant voltage is applied between the two ends of
·current, as shown. The voltmeter
a uniform metallic wire. Some heat is' developed in it.
reading will be:
The heat developed is doubled if:
(a) 0 (b) 4 V
(a) both the length and the radius of the wire are
(c) 6 V (d) 12 V
halved.
(b) both the length and the radius of the wire are 60. In a balanced wheat stone bridge, current in the
halved. galvanometer is zero. It remains zero when:
(c) the radius of the wire is doubled [1] battery emf is increased ·
(d) the length of the wire is doubled [2] all resistance are increased by 10 ohms
[3] all resistance are made five time
54, In an electrical circuit containing a battery, the charge
[4] the battery and the galvanometer are interchanged
(assume positive) inside the battery :
(a) only [1] is correct
(a) always goes from positive terminal to the negative
(b) [1], [2] and [3] are correct
terminal
(c) [1], [3] and [4] are correct
(b) may go from positive terminal to negative
(d) [l] and [3] are correct
terminal

www.puucho.com
Anurag Mishra Electricity and Magnetism with www.puucho.com

- -- -- - - -· --1
., _____ ,_ 291,
---~..J

61. The circuit below is made up using identical light 66., In , the figure, the
bulbs. The light bulbs of maximum brightness of the potentiometer wire AB of
following will be : length L and resistance 9r is
joined to the cell D of emf E A 1------'i---'B
C
and internal resistance r. The C
+ -
cell C's emf is EI 2 and its
internal resistance is 2r. The ~.2r
D 2
E
galvanometer G will show no
deflectioµ ;when the length AJ is:
(a) 4L (b) SL
(a) A (b) C 9 9
(c) D (d) E (c) 7L (d) llL
62. A 6 V battery of negligible 6V 18 18
internal resistance is 67. 24 identical cells, each of internal resistance 0.5 0, are
connected across a uniform arranged in a parallel combination of n rows, each row
wire of length 1 m. The containing m cells in series. The combination is
positive terminal of another AL-----"---_J B' connected across a resistor of 3 o. In order to send
battery of emf 4 V and maximum current through the resistor, we should
internal resistance ill is have:
joined to the point A as 4V, 1n
shown in figure. The (a) m = 12, n = 2 (b) m=8,n =3
ammeter shows zero deflection when the jockey (c) m = 2,n =12 (d) m = 3,n = 8
touches the wire at the point C. The AC is equal to : 68. In the given potentiometer circuit length of the wire
(a) 2/3 m (b) 1/3 m AB is 3 m and resistance is R = 4.50. The length AC
(c) 3/5 m (d) 1/2 m
for no deflection in galvanometer is:
63.

~i~;~:~:~i~:~· ·~-
·;:;;;
the jockey J in the
,, '
E=SV r=O .. SQ

A-----~---- B
R=4.5Q

position of balance. If R C
is now made 80, through what distance will J have to
be moved to obtain balance?
(a) 10 cm (b) 20 cm
(c) 30 cm (d) 40 cm (a) 2 m (b) 1.8 m
64. n resistances each of
resistance R are joined
.th . f
w1 capacitors o capacity ,
HR 3J
-
. '& ,R C_ R ... C
.. · .
R•
(c) dependent on r1

connected across a 4 m long


(d) none of these
.69. A battery of emf E 0 = 12V is e, R = BQ 1

C ( each) and a battery of uniform wire having


emf E as shown in the figure. In steady state condition resistance 40 / m. The cells -A B
ratio of charge stored in the first and the last capacitor of small emfs E1 = 2V and
is : E 2 = 4V having internal
(a) n: 1 (b) (n -1) : R resistance 20 and 60
(c) n2 +l respectively, are connected
(d) 1 : 1 as shown in the figure. If galvanometer shows no
n 2 -1
reflecrion at the point N, the distance of point N from
a
65. The length of potentiometer wire is 1. A cell of emf E
the point A is equal to :
is balanced at a length l/3 from the positive and of the
wire. If the length of the wire is increased by l/2. At (a) .! m (b) .! m
6 3
what distance will the same cell give a balance point: (c) 25 cm (d) 50 cm
(a) 21 (b) J.. 70. When a battery is getting charged :
3 2 (a) the voltage drop across its internal resistance is
41
(c) !.. (d) zero
6 3 (b) the terminal potential difference is less than its
emf

www.puucho.com
Anurag Mishra Electricity and Magnetism with www.puucho.com -
. - ··'.··- ... "---·"··' - ~,
I 292
' ,

.ELECTRICITY &MAGNETISM j
(c) the terminal potential difference is more than its the width of the bar. The electric resistance of the bar
emf · across its rectangular ends is:
(d) its terminal potential difference is zero (a) £!_g_ (b) Pio
71. Which of the following wiring diagrams could be used
to experimentally determine R using ohm's law? aro ar0 m[l+ ~]

,.,:EJ E:J:
Assume an ideal voltmeter and an ideal ammeter:
(d) None of these

(b)
77. Fig. shows a part of complete circuit. The current in

FI J_ ,,,e::l
various branches in steady state are shown in figure.
R R The energy stored in capacitor is:
1A
(c) !__I --J 2A
3Q
4V c-4µF 1U
72. Current density! in(~ cyl)indrical wir~ of radius R is
J 0 --1 for o,;;x,;;-
given as J = R
2 . ·The current
x R R
J 0 -for-,;;x,;;- 1A
R 2 2
flowing in the wire is: (a) 200µJ (b) 400µJ
(a) 2_ rr.J oR 2 (b) ~ rr.J oR 2 (c) 600µJ (d) B00µJ
24 6 78. A cell develops the same power across two resistance
7 2
(d) 2-rr.1 R 2 R1 and R 2 separately. The internal resistance of the cell
(c) -rr.f0R
12 12 o is:
73. What is the equivalent 2Q 4U
(b) R, +Rz
capacitance between A and 2
B in the circuit shown:
(a) 6 µ F
(b) 1.5 µ F '.
J:F (c) ~R 1R 2 (d) ~R1R2
. 2
79. The region between two concentric spheres of radii ra
(c) zero ' 6µF 3µF
and rb is filled with a conducting material. The inner
(d) 2 µ F sphere is maintained at a potential Va and outer
74.
:e:afnre~:=~:r i~fin::;;~db:~ ; a ·!
X. ···.;'..
sphere is at lower potential Vb, as a result of which,
current is there in radia!Iy outward direction. The
series with circuit it reads lA. ·, variation of electric field intensity as a function of
When the voltmeter of very large '. · ' distance r from centre of sphere is given by:
resistance is connected across X it . 12V_ •. J E-·.. .E
reads lV. When the point A and B
are shorted by a conducting wire, the voltmeter i\
measures 10 V across the battery. The internal
resistance of the battery is equal to:
(a) zero (b) 0.50
(a) i:: ::.j., . (b) Ea •·

(c) 0.20 (d) 0.10 '. _____ ,:a rb


75. The ratios of lengths, masses, densities, and E' E
resistivities of two wires are in ratio 1 : 2, 1 : l, 1 : 2 'I ;

and 4 : 1 respectively. The ratio of their resistances is :


•E .. ..
(a) 1 : 1 (b) 2 : 1 (c) ; a (d) Ea
(c) 4 : 1 (d) 1 : 2 E,' Eb . . ..
76. A curved rectangular bar forms a resistor. The curved I
·-·· 'a ,, .. ,. _r. 'a ,, r·
sides are concentric circular arcs. If p is the resistivity
of the material of bar, 10 is the length of inner arc of
radius r0 , (r0 + b) is the radius of the outer arc, and a is

www.puucho.com
Anurag Mishra Electricity and Magnetism with www.puucho.com

! ELECTRIC CURRENT
80. Current passing through 1 Q
resistance is zero. Then the emf
& is:
(a) 8 V 6V 20 E
(b) 6 V
(c) 4 V
(d) 12 V
81. The two batteries of emf &1 and &2 having internal Temperature coefficient of resistance of the coil is :
resistances r1 and r2 respectively are connected in (a) 6.3x 10-4 K-1 (b) 4.3x 10-4 K-:__
series to an external resistor R. Both the batteries are 4
(c) 8.3x 10- K- 1 (d) 23x 10-4 K-1 -
-._____;
getting discharged. The above described combination
85. A wire has linear resistance p (in A
of these two batteries has to produce a weaker current
then when any one of the battery is connected to same Ohm/m). Find the resistance R
resistor. For this requirement to be fulfilled: between points A and B if the
side of the "big" square is d:
(a) &2 must not lie between -----2__ and r2 + R
&1 r1 +R r1 (a)~ .
..[i
(b) &2 must lie between -----2__ and r 2 + R (b) .fipd _B
&1 r1+R r1
(c) 2pd
(d) None of these
(c) &2 must lie between -----2__ ;nd _rl_ 86. The wattage rating of a light bulb indicates the power
&1 r1 +R r2 +R dissipated by the bulb if it is connected across 110 V
(d) &2 must not lie between -----2__ and _rl_ DC potential difference. If a SOW and lOOW bulb are
&1 r1 +R r2 +R connected in series to a 110 V DC source, how much
82. An ammeter has resistance R0 and range I. What power wilt' be dissipated in the SOW bulb :
(a) SOW _- (b) 100W
resistance should be connected in parallel with it to
(c) 22W (d) 11 W
increase its range to nJ?
87. The same mass of copper is drawn into two wires A
(a) Ro (b) R 0 (n+l) and B or radii r and 3r respectively. They are
n
connected in series, and electric current is passed. The
(c) __&,_ (d) __&,_ ratio of the heat produced in A and B is :
n +l n-l
(a) 1 : 9 (b) 1 : 81
83. Each of the three resistors connected in a circuit as (c) 81 : 1 (d) 9 : 1
shown below has a resistance of 2Q and can dissipate a 88. If in the above question A and B are connected in
maximum of 18W without becoming excessively parallel between the terminals of a source of emf, the
heated. The maximum power that the circuit can ratio of heat produced in A and B is :
·dissipate is : (a) 1 : 9 (b) 1 : 81
(c) 81 : 1 (d) 9 : 1
89. Seven resistors are connected as shown in the
diagram.
..30Q
. 40. B
.
A .
(a) S4W (b) 36W 6Q . 80
·,
(c) 18W (d) 27W
84. Figure shows a meter bridge, wire AC has uniform
1QQ
. 8Q .
• '100
·'
cross-section. The length 'of wire AC is 100 cm.Xis a
standard resistor of 40 and Y -is a coil. When Y is The equivale.nt resistance in ohms of this network
between A and B is :
immersed in melting ice the null point is at 40 cm from (a) 6 (b) 8
point A. When the coil Y is heated to 100°C, a 100 Q (c) 12 (d) 20
resistor has to be connected in parallel with Yin order 90. In the circuit shown, the rea_ding of the ammeter is
to keep the bridge balanced at the same point: doubled after the switch is closed. Each resistor has a
resistance ~10 and the ideal cell has an emf.~lOV.
Then, the ammeter has a coil resistance equal to:

www.puucho.com
Anurag Mishra Electricity and Magnetism with www.puucho.com

[294 ·>·~-~1icrRICITY&~jnsMJ
95. In the diagrams, all light bulbs are identical, all cells
are ideal and identical. In which circuit will the bulbs
be dimmest question :

(a) .
(a) 2Q (bl lf.l
(c) 2.SQ (d) None
91. Two lamps, each with a ~---1 ~ 1----,
resistance of son, are
connected in series. The lamps (bl
will fuse if a power of more
than 200 W is dissipated in it.
What is the maximum voltage
that can be applied to the
circuit? (cl
(a) 100 V (b) 140 V
(c) 200 V (d) None
92. A beam of fast moving electrons having cross-sectional
area A falls normally on flat surface. The electrons are
absorbed by the surface and the average pressure
exerted by the electrons on this surface is found to be R (d) 'j
If the electrons are moving with a speed, v, then the
effective current through any cross-section of the
electron beam is: 96. A battery, or batteries, connected to two parallel plates
(a) APe/(mv) (b) APe/(mv 2 ) produce the equipotential lines between the plates as
shown. Which of the following configurations is most
(c) APe I (me) (d) APm I (eV)
likely to produce these equipotential lines?
93. What should be the value of R so that the electric
power consumed by it is maximum:
• R 20 40 .

5 12

10
-2V -1V OV 1V 2V,

(al 12n
(cl 6Q
(bl 24f.l
(d) none of these
94. The diagram shows a bimetallic strip used as a
thermostat in a circuit. The copper expands more than
,. :r:J: ~,:CJ:
~ . · ,2V.• __ 2V, ... · 2V. ., '

,,,fr~ ,,,:r:J:
the invar for the same temperature rise.
" y signaf lamp ,w

inVar Electric bell

copper
motor 2.V • 2V _ .. 21/_ 2V ...
97. In the multi-loop circuit shown in the Fig. which of the
What will be switched on when the bimetallic strip following set of equations is correct?
becomes hot question
(a) bell only
(b) lamp and bell only
(c) motor and bell only
(d) lamp, bell and motor

www.puucho.com
Anurag Mishra Electricity and Magnetism with www.puucho.com

LELECTRIC CURRENT
&,

1,f R2
-1,

&, R1

R3
- 13 (a) O.SV (b) 1 V
12 +1 3 =1 1
(a) +I 2R2 -&1 +I 1R1 -&2 =0
&2 -I 1R1 -I 3 R3 =0
(c) 0 V
100. A uniform wire of resistance R
stretched uniformly to n times
(d) 2 V
<[> 8
I, +!3 =I1 and then cut to form five wires of A
(b) -I 2R2 -&1 -I 3 R3 =O equal length. These wire are
&2 -I 1R1 -I 3 R3 =0 arranged as shown in the Fig. The effective resistance
I1 +I, =!3 between points A and B is:
(c) -J 2R2 -&1 +I1R1 -&2 =0 (a) nR I 5 (b) RI (Sn)
&2 -I 1R 1 -I3R 3 =0 (c)n 2 R/5 (d) R/(Sn 2 )
I 1 +I 2 =I 3 101. A battery of emf 2V is connected across a long uniform
(d) I2R2 +&1 +!3 R3 =0 wire AB of length lm and resistance per unit length
&2 +I 1R 1 +I 3R 3 =0 2Qm- 1 • Two cells of emf & 1 =1 V and & 2 =2V are
98. A metal rod of radius connected as shown in the fig. If the galvanometer
a is concentric with a
shows no deflection at point P, the distance of point P
metal cylindrical shell
from point A is equal to :
of radius b and length
2V
1. The space between 2Q
rod and cylinder is
tightly packed with a
high resistance
material of resistively
I
i I r
r---
+
~v
p
A l==2v=2n==·=r==::1 8

p. A battery having a
terminal voltage V is
connected across the
combination as
l (a) 0
1V 1Q

(b) 50 cm
(c) 100 cm (d) 25 cm
shown. Neglect
resistance of rod and 102. A frame made of thin
cylinder. If I is the total current in the circuit then: homogeneous wire is
shown in Fig. Assume
2nlV that the number of
(a) I )V (b) I successively
p p (lnb-lna)
embedded equilateral
(c) I 4-n:lV lV
(d) I triangle with sides
p (lnb-lna) 4rrp (In b - In a) decreasing by half
99. A circuit is comprised of eight identical batteries and a tends to infinity. The
resistor R =0.8Q. Each battery has an emf of 1 V and side AB has a A " - - - - - " - - - - - - - - . B
internal resistance of 0.2Q. The voltage difference resistance R0 • The
across any of the battery is: equivalent resistance between A and B is x,.
(a) x is infinite (b) xis zero
(c) X =2Ro (d) -1)
x= (--fi3 - Ro

www.puucho.com
Anurag Mishra Electricity and Magnetism with www.puucho.com

I 296
103. When an ammeter of negligible A B
resistance is inserted in series y •10V A
X
with circuit it reads 1 A. When =-
the voltµieter of very large =- o·
B
resistance is connected across X C
it reads 1 V. When the points A 1:1V
and B are shorted by a (a) A lb) il
conducting wire, the voltmeter measures_ IO V across (c) C (dJ. D
the battery. The internal resistance of the ·battery.is: 106. A galvanometer with an internal resistance of 100.Q
(a) zero (b) 0.2.Q will show a full scale deflection with a current of
(c) 0.5.Q (d) 0.1.Q lOmA. Which of the following circuits would tum this
104. A galvanometer is a sensitive instrument that gives a galvanometer into an ammeter which will read lOA at
reaqing proportional to the current that flows through full scale?
it. If such an instrument has a built-in (internal)
resistance of 200 .Q and requires a current of 5.0 mA
for full-scale reading, what resistance should be
connected in parallel with this galvanometer to make
-~~-
'0
(a) ~ G ,
it function as an ammeter that reads 10 A when the
reading is full-scale? I
(a) 400 .Q (b) 0.2 .Q . '
(c) 0.1 .Q (d) 4x 10 5 .Q
105. In the circuit diagram, all the bulbs are identical. /
Which bulb will be the brightest?

' . f.
AN8WER8

1. (d) 2. (bl 3. (al 4. (b) 5. (bl 6. ( l 7. (b) 8. (al ,i


(d) (a) 16. '
9. (dl 10. (al 11. (dl 12. 13. 14. (bl 15. (dl {bl
17. (dl 18. (dl 19. (bl 20. (bl 21. (al 22. (a) 23. (dl 24 (bl
• I
25. (c) 26. (al 27. (cl 28. (al 29. (bl 30. (cl 31. (dl 32. (bl
33. (cl 34. (cl 35. (al 36. (bl 37. (al 38. (bl 39. (cl 40. (b)
41. (dl 42. (al 43. (dl 44. (dl 45. (bl 46. (bl 47. (c) 48. (d)
49. (a) 50. (c) 51. C•l 52. (a) 53. (bl 54. (bl 55. (al 56. (dl
57. (cl 58. . (d) 59. (dl 60. (cl ' 61. (bl 62. (a) 63. (bl 64. (dl
65. Cbl 66. (b) 67. (al 68. (d) 69. (cl 70. (c) 71. Cbl 72. (dl
73. (dl 74. (bl 75. (dl 76. (b) 77. (dl 78. (cl 79. (cl 80. (bl
:
81. (al 82. (dl 83. (dl 84. (a) 85. (a) 86. (cl ' 87. (cl 88. (b)
89. (a) 90. (a) 91. (c) 92. (al 93. (a) 94. (bl 95. (cl 96. (b)
I

97. (dl 98. (bl 99. (cl 100. (cl 101. (al 102. (dl 103. ·, (bl 104. (cl
(c). ',
105. 106. (bl

www.puucho.com
Anurag Mishra Electricity and Magnetism with www.puucho.com

l;ElECTRlf CURRENT,
-· -- -- -· r------------ ·---":·1
I'

L [!11 Redrawing the circuit


6Q

net resistance between A and B is 5 n.


J1 2 &= lOxl+lOxl =l0V
2. [b] (, -I, x 1 =(I-I,) x 2 =:, - =-
)' --~ I 3
l+l
., lx 1 1
'I r=--=-
i 1+1 2
3Q
For max. power
i----1 I I
R=r=_!
"
2

P=I 2R
=:> maximum power = 100 x .! = 50 watt.
2
[2 [2
P1 =4-xl, P2 =-x2, 7. [b] Let length of each wire be i
9 9 ,... · - •. 112· ..._ -· ,,22 - - ·
P3 =l 2 X 3 ~~~",N\. ~~----
F1 :P2 :P3 = 4: 2 :. 27 ..•• P2 .•. _. ____ :
3. [a] Since the shunt is in parallel to (20 + R)Q P11
=:, 0.04x (20+R) = 0.05x (20-0:04) R1 =_L
=:, R=4.95Q A
P21
R2 =_i_
A
R =R1 +R 2
l
pxl CP1 + P2)-
2
A A
5. [bl When the voltmeter is ; · · - -50 n · 50 n l
connected across any of ---'\l\l\,---"N\,"-'--;,
the coil as shown in ,_.r.n_ ! 2&
8. [a] Current in the circuit is,. I = - - - -
· figure. Then by solving I r1 + r2 + R
the circuit we will get
the potential difference
'------1
f-----'
'
I Potential difference across the battery having
12V internal resistance r1 is,
across voltmeter as 5.85 - - - ~ ----- --·· •i ,, _ "'-I _ &(r2 +R-r1 )
V which is nothing but equal to voltmeter reading. vi - c;;;, r1 -
r1 +r2 +R
When voltmeter is connected across terminals of
the battery then the reading of voltmeter is 12 V. Potential difference across the battery having
internal resistance r2 is, ·
,, _.,. _&(r1 +R-r2)
v2 - ~ - 1r2 - - - - - -
r1 +r2 +R

www.puucho.com
Anurag Mishra Electricity and Magnetism with www.puucho.com

1298
If V1 = 0, then R = r1 -r2 > 0 as r1 > r2 15. [d] In first case
If V2 = 0, then R = r 2 - r1 < 0 which is not possible.
1A------<f-----"11"----B
I
9. [d] 4 Q resistor is short circuited r
. 10V 1n A-----; t---"W • •B
A~--<'I . B 1=3
VA-&+3r=VB
VA - VB = 10 = /!; - 3r ... (i)
.. In second case similarly
4Q VA-&-2r=VB
10 => 15 = & + 2r ... (ii)
I =1- =l0A
. From eqns. (i) and (ii),
r =l Q.
VA-lO+lOxl=VB
16. [b] Due to symmetry of circuit. Current distribution in
VA -VB =0
10. [a] Using Kirchhoff's law circuit will be as shown
I = ~ =l0A
i
:· 20V
A~•_,,_.______ ,N_..,,_}~;
2n I 1-1, 4n 5V !
2-,i--',-·y,.•..,-,•--·-s ,
Now, VA =VB
4+4

=> 6(I-I1)=l2I1
=> 6(10-I1 )=12xl 1
60=18!1
=c 60 10
I,=-=-
20-2!-2!1 =0 ... (i), i 18 3
20-2!-4(!-! 1 ) =5 I_
211
= 10-2x 10 10 A
15-6+4I1 =0 ... (ii) 3 3
From eqns. (i) and (ii),
Ii =4.5A
11. [d] 2 Q resistance will be short circuited as potential
drop across it is O also 4 Q and 5 Q are in parallel
17. [d]. r~_·:,_-_-_~_.,__, ·____
E r

i ,_
---"Nv--
__ _1_, :
---_--_-

2Q
I=(n-2)&-2& =(n-4)&
1-1, n nr nr
4n 20V V=&+Ir
I
=& + (n-:rex r 2e(1-;)
1 4+ 5 9 18. [d] 2, 3, 6 Q resistors are in parallel
-=--=-
R 4x 5 20 Among these resistors, maximum heat will be

=>
I=
20
20
x 9= 9A
I 1 x4=(9-I 1 )x5
produced in 2 Q; [ H =:
2
t]
I 1 = SA,I -! 1 = 4A 2Q 4Q

:&?:'.3'
12. (d) From the symmetry of figure it is clear thatO ·and,D
are at same potential so no current flows from CD
branch.
Extra (1) Find Req across AB and across BC.
(2) Find the current through CD if battery is
connected across BD. Similarly in 4, 5 Q max heat will be produced in 4
13. [a] When galvanometer resistance tends to infinity n.
G --,oo, P.d. across R is 20 V 20 29 9V
Requivalent ;::::;-+ 1 = - ~ I = -
20= 24-Ir= 24-Jxl 9 9 29
or I= 4A 9
VAB =l x v (1 is equivalent resistance of 6 n, 3 n,
also 20=4xR 29
or R =SQ 2Q)

www.puucho.com
Anurag Mishra Electricity and Magnetism with www.puucho.com

fnmR1c cuRRENr ·. 299


"v BC 20 x -9V ; -20V (20
; -
. eqmva
- 1s . lent resistance
. Also VA +4+2xR ;Ve ... (iii)
9 29 29 9 From eqn. (iii) and (i),
of4Q, SQ) VA +4+2R;VA +6(VB ;Ve)

(9V) 1 2
2R;2
Heat generated in 2Q; - -t x
R;lQ
29 2
2 22. [a] In first case
. m
Heat generation . 4Q ; (20V)
- 1
x -t &
18;--x 6 ... (i)
29 4 6+r
Heat generated in 4Q is max. In second case
19. [b] VA -4[ -9-(I -I,)+ 3-4[; VB &
2 4 ; - - x 12 ... (ii)
l6-9I +I 1 -6; 0 12+ r
M.
0

4!2 1Q N ° 4Q From eqn. (i) and (ii) r ; 6 Q


A "'\Pv .:·1~1,j~~B Now let in the case S1 and S 2 both are closed
(Resultant of 12 and 6 Q will be 4)
2!2 4
V ; _§_ X 4; & X ••• (""")
111
lQ;9J-Ji ... (i) 4+ r 10
VA -4I -2I 1 + 3-4I; VB (as r ; 6 Q)
l6-BI-2I 1 +3;0 Eqn. (i) divided by eqn. (iii)
19; BI+ 2I 1 ... (ii) 18 10 6
-=-X-
From eqn. (i) and (ii), V 4 12

,._~,r~ . ~?l
I 1 ;3.SA
V; 72 ;14.4V
5
23. [d] Rt I ,j. => VB t
Alternative :-
R R I'

e . l
2
When S1 is closed Br E
I ---;

(3+ l)R 4R
& • 3&
V1 ; - x 3 R ; -

When S2 is closed
4R 4 P, ;(i:)'r ;I:r P2 ;J'2 r
2
& 6&
J ; - =>
7R
When both are closed equivalent of 6R and 3R is
V2 ; -
7
P1 ;[_£_] ;(_§_)\
R+!:_
!:_ P2
4 R+r
2R. 2
I ;_E_
3R P,
(2R&2r
+r) 2
P2
P1
;(2RR +r+r) >l ; P2 > P1
E 2E
V3 ;-X2R;-
3R 3 So ball B will become brighter.
=> -V2 >V1 >V3 24. [b] Redrawing the circuit
21. [a] For current in 4 Q ; 0 R
. .. C
R R
1,=0 p Q
2Q 4!2
R R
B -
'
~
E E
R
. JOV A 41( ... p Q
Ve ;VB
VA +6;VB ... (i)
VA +10-2[; Ve; VB ... (ii)
From eqn. (i) and (ii), E
1Q-2f;6 => [;2A

www.puucho.com
Anurag Mishra Electricity and Magnetism with www.puucho.com

R :· A c
2Rx- •B . 2!1 15!1
2 ~R
Requivalent = R
5
I
10n
2R+- 46. [bl! SQ
D /C'. D
2 I
,2QQ 10!1
25. [cl 0+ 10-21 -3(1 -1 1)-3(1 -1 1) = 0 C
10=81-61, .. ,(i) '30J.L . 40Q
I I 2Q 3Q Redrawing the circuit
' 10\/
1, . 1~1, - ---- ·- 20Q 30!1
2Q
J_
.,,,
2Q
A"""'"""".,._-,( )t---\a.,_n_ 8
---- .. D
Also 40Q 10n
0+10-21-21 1 =·o
21+21, =10, ... (ii) 15Q "'10!1
From eqn. (i) and (ii), · . I
15 20 2!1 25/2Q 8Q I
11 =-A l =-A
7 ' 7 A~•--"''""•"----'w.~.- - - \ ~ B !
c ____ ....... g
. . SA
=> !-t,=7 RAB = 22.5 Q
48. [dl Sum of charges on plates of capacitor was zero
26. [al Let
A• . ·c:3·:··
VA -VB =V
B,
49.
initially and will be zero finally.
No charge is flown to ground.
. [al .This is 'parallel
' ,
combination
- .. - -.
bf batteries
-- '
.......... ,.R.
v2 R

RG 3 3 R,
Given, V2 =4' R =4RG
R,
R
&1R2 + &2R1
43. [dl Hint: P1 =V 2 / R; equivalent e.m.f.
R1 +R 2
(VI 2) 2 R;R 2
Resistance
.! R1 +R 2
The given circuit is
V
& = &,R2 + &2R1 '
44. [d] Points 1,2,3, ....... are R1 +R 2 :e
equipotential .... -·· ,. ' and we know that power R
' developed in 'R' in such circuits
is max. when R = internal
resistance
R = R1R2
R1 +R 2
52. [al Due to decrease in number of collisions resistance
will decrease
=> current will increase
54, [bl During charging positive charge flow from positive
terminal to negative terminal.
57, [cl Equivalent e.m.f.= &,r, + &212
r1 + r2

and 11, 2', 3;, ....... are also equipotential.


Equivalent resistance = ri r2
r1 + r2
Equivalent resistance will be less than either of the
two resistance

www.puucho.com
Anurag Mishra Electricity and Magnetism with www.puucho.com

r·~----
ELEaR1c cuRRENr
---- - --- - -
301\
73. [d] Consider the shown circuit.
2n .. 4Q

:
A
X1F
_6µF __ 3µF _
si

I
... J
Current through resistors
. &1r1 + &1r2
=> eqmvalent e.m.f. > ~~-~~ = &1 I=&
r1 + r2 6
& &
·
Also eqmvalent &2r1 +&2r2
e.m.f. < ~~-~~
r1. + rz
VA -Ve =2x-=-
6 3
... (i)

& ... (ii)


Equivalent e.m.f. < &2 VA -VB=-
i.e. equivalent e.m.f. will be in between & 1 and & 2 • 3
From eqn. (i) and (ii)
61. [b] Let potential at Pis O & that at Q is V
V V . . V/3 V/3' => Ve =VB
if 6 µF is connected through B and C it will be short
C
circuited.
. 6x 3 E
So net capacitance = -- = 2 µ .
D 6+3
E
11 _ 1 m, - l P1 _ 4
B
Q ----+----<>---..
75, [d]
z;-z'm 2
- 'j;";-1
0 0 0 0 d, 1
-=-
Potential difference across bulb C is maximum. d2 2
64. [d] Each resistor and capacitor are in parallel i.e., there pl p x 12 p1 2 p x 12
R----------
is no difference in first and last capacitors.
A Axl V (:)
equal charge will be stored on both capacitors.
Ill Cells 2
,--, i---W'---i 1-W,, ....... . R, =£.i..x(.!!.) x.:!!.x m 2

H i---W'---i 1-W,,- ....... . n rows I R2 P2 12 d2 m,


!
67, [a]
'------li---W'---i......,,,,,.········ I =1x(½r x½x½=½
76. (b) Let us consider an
elemental portion of
. . . _3n ___ ··-·····-'·· the resistor. The
For max. current or developing maximum power
element consider is a
mE mr
r = o.sn circular arc of radius r
L

- rnE
'
'
mr
and thickness dr. The
resistance of this
'
... -- --' -- --- -- -- -- -. - - element would be
dR=pxr0_
adr
i.
where lo = roe
3n dR= prx10
So,
3 = requivalent (due to all cells) 3 Q ar0 x dr
3= mr = .".:. If we divide the entire resistor in these elemental
n 2n portions, then these elemental resistors are joined
On cross checking options m = 12, in parallel, equivalent resistance of which is given
n = 2 satisfies. by
70. [c] VA -&-lr=VB 1 -f - 1--f'1J+bar dr
VA - VB = & + Ir > &
-- -0-
R dR 'o p1 0 r
= aro x In ro + b
plo ro

www.puucho.com
Anurag Mishra Electricity and Magnetism with www.puucho.com

.J. - - _ ELECTRl(ITY.& M4~~!Tl~M]

plo 81. (a) When two cells are connected in series,

{ .rob]... .
R
I = &, + & 2 • When resistor is connected across
ar0 1 1 +- r1 +r2 +R
77, (d) Using KCL and KVL we can find the potential battery of emf &1 , I 1 =~
difference across capacitor which comes out to be r1 +R
20 V. When resistor is connected across battery of emf
So energy stored in capacitor is, ~ I . &2
c;.-2, 2 = - -
U = 4x 10-6 x 202 = 800µJ r2 +R

,._,,,(OD B.1 ______ _


2 For required condition to be fulfilled.

and
~
&2< -
-
&1 r1 +R
I < 11
I <1 2
r2 - a · nd -&2>r2-+-
&1 r1
R

82. [d] To increase the range upto n times, the resistance S

P, =
z2 R,
(shunt) should be such that
X 18 =I In
(R1 + r) 2 s· (I I n)R 0 R 0
&2 I n-l
=P2 2 X R2 I--
(R 2 + r) n
~ (R 2 +
R1 + r
r)
R1
2
= R2
83. [d] P =I 2R =1 2 2 =18

.I . 1/R , I
On solving r = JR1R 2 I
79. (c) Consider a shell of radius r . ---~,...~ ,w,>: ,
--- ,-,F _ _ _ , .• • d !1- i. I
and thickness dr as shown in .- . vv
I
figure. Resistance of this shell 1/R I
is .... .i
dR = p dr where p is I =3A
4nr 2 2 2
resistivity of conducting l_ -
Pnet =[(iJR]2+[I R]=I!+I R
· material.
18
R =fdR =f ~ pdr = +18=27
ra 41tr2 2
X l
~:"[:.-~]. 84, [a] -=--
Ro 100-1
Since null point remains unchanged
I=v.-vb X 40
R -=-
R' 60
The electric field intensity at the location of shell
R'=6Q
. E =-
1S,
pl- l00R,
2
4nr And 6
R, +100
E R, =6.38Q
a R, -Ro 6.3x 10-4 K-1
R0 t
As current is zero for r < r0 and r > r6 and hence E. 85. [a] The circuit is equivalent to
80. [bl Current through 1 n = 0 IA -- ........,.• ·- ......... -
· Current through . I B _ 1. - • r r
6 r
2Q=-=3A r/2
2 6V 2n r r/2 r
VA+& =VB
VA+ 2x 3 = VB -1
&=6V
r st
t ---~--~
Let each half side has resistance r (=pd/ 2)

www.puucho.com
Anurag Mishra Electricity and Magnetism with www.puucho.com

ELECTRIC CURRENT
2r R (5+4)x(10+8)
Therefore 60
9+18
90. [a] Ohm's law V =IR =} V ocI (for ammeter)
Vz=Iz=2
A 2r 8
Vi Ii
R
=} 10=2x10x--
r/2 ' -
2+R
=} R =2Q
2
R =~[2r+ C r)C:)]=r.f2 (on solving) 92. [a] Let n be the average no of electrons per unit
2 (2+ 2)r volume, present in the beam of electrons, then
R =pd/ ./2 impulse-momentum theorem gives
vz (nAv)(mv)=F, P =FI A
86. [c] R=- =} • n =P/(mv 2 )
p
ll0x 110 ll0x 110 Alls
l=vAvs=--
R 1 - - - R2 µv
50 100
93. [a]

110
110 V
110xl10

10
I= 3R = 3x ll0x 110 = 33 amp.
Here R,q =12
For maximum P;R =R,q =12
L
R

-~.
Pi =I2 xRi =10 x 10 x llOxllO 97. [d] Junction rule
33 33 50 I 3 =I~ +I 1
= 200 a,22W loop DCEFD
'' -Z2 -I1R1 -I3R3 =0
9
87. [c] Let RA and Rn are resistances 1oopABEFA-Z1 -I 3 R3 -I 2 R 2 =0
IA 99. [c] For loop CDAC, 4(1.0)-4(0.2)I -0.8I1 =0
RA =p- .' '
AA =} 5 =I -I1 .•• (i)
' l For loopABCA, 4(1)-4(0.2)(J-I 1)+0.8I1 =0
and Rn =p_!,__
AB 5=1-2I1 ... (ii)
RA =1_ x Aa =(AaJ2 'A
R8 18 AA AA

[ :.lAAA =lnAn =}
An= -
- IA]
AA ln ,j>~

(1tx 9r 2 ) 2
(itr2)2

In series HA:H 8 =RA:R8 =81:1


1 1
88. [b] In parallel HA,HB =Rn :RA =-:-=1:81
81
1
.L:f-<,, H,

C
81 1 Using eqn. (i) and (ii), I 1 =0 and I =5
89. [a] The given circuit is equivalent to V = Voltage across any battery-
=& -Ir =l.0-5(0.2)=0V

r:~!
A 511 411 B

;-t,•,w r,oo: 100. [c] R =Pli


A1
12 =nl1
[p=resistivityofthewire]

A 1 11 =A 2 l2
As lOx 4=5x 8 this is balanced Wheatstone A1
A 2 =--=A1 /n
network 12 I 11

www.puucho.com
Anurag Mishra Electricity and Magnetism with www.puucho.com

r =0.20
New resistance of the wire = R' = ~: = n 2R
105. [cl As all bulbs are identical, the power (related to
Resistance of each wire =n 2R / 5 brightness) is most useful as .P =l 2 R, so .the
Since the wires are arranged to form balanced . resistor with the most current will be brightest.
Wheatstone's bridge As bulb C has the same current as that through
(n 2R/ S)+(n 2R/ 5) n 2R the battery, while the branches with bulbs A, B
RAB and D are in parallel and have the total current
2 5.
101, [al I =2/ (2+2)=(1/2)A split between the branches, the current .through
bulb C is greatest.
'A
-<;n =Bl=S/2 ... (i) 106. [bl _ _ I = lOA
·-. R;=foon
·®'--7·.
l mA;Cj
Also, 2-21 1 +I -1 1 =0
=> 11 =lA t ,; .... ,
VA -Vp =I, -1=1-1=0
using eqn. (i) and (ii), x =0.
... (ii) (= 10A 19 =
Point A and P are at same potential. l____ \~, ..,
103. [bl I =lA
R =internal resistance of the body I ,R, "' IR,
12 =(X + Y + r) (1) ... (1), I,R,
R =~-
Also l=(X)(l) => X =10 ' I
And 10 = voltage across battery = voltage across X. l0x 10-3 x 100
0.10
(when A and B are shorted) 10

10 =(_E__)x => 10 = ~ => l+r=~


X+r l+r 5

www.puucho.com
Anurag Mishra Electricity and Magnetism with www.puucho.com

-305)

cPeve I
1. In the given circuit A3 (c) E1:E2 ;4: 1 (d) E1:E2; 1: 4
6R
ammeters are ideal then, 5. A cuboids has longest dimension double of the shortest
which of the following 2R A2 R dimension. Then the ratio of maximum to minimum
statements are true? resistance across its parallel faces is :
(a) Reading of A 3 will be (a) 2 : 1 (bl 4 : 1
& R
half as shown by A2 A1 (c) 8 : 1 (d) 3 : 2
6. If temperature for a metallic conductor increases then
(b) Reading of A1 will be thrice as shown by A2 which of the following quantity decreases :
(cl Reading of A3 will be lowest . (a) µ, (bl p
(d) Reading of A1 will be thrice as shown by A3 (cl cr Cd) vd
2. In the given circuit if JI and J2 be the current in 7. A voltmeter and an ammeter are joined in series to an
resistors R1 and R 2 respectively then : ideal cell, giving readings V and A respectively. If a
. -- 3A -
resistance equal to resistance of ammeter is now
f----e
joined parallel to ammeter then :

3A (a) V will decrease slightly


(b) V will increase slightly
2A
(cl A will become half of its initial value
(a) JI ;3A, J2 ;2A (d) A will- become slightly more than half of its initial
(b) J1 ;Q, J2 ;2A value
(cl J 1 ;2A, J 2 ;2A 8. The figure shows a &
,( d) J1 ; 2A and J2 can't· be determined with given potentiometer ----1
data arrangement & is e.m.f. C J
3. In. the given circuit of driving cell. &' is to A 1--------.--..,.J B',.,
R1R 4 ; R 2R 3 . Then choose be determined. Then D
the correct statement : which of the following "----1 1-~'M/1.---"" G i
are essential condition?
?'__ r
{a) If positions of battery
and galvanometer are (a)·& must be greater than&'
interchanged then (bl The positive terminals of C and D must be joined
galvanometer will still & to A only
show zero deflection. (cl Either the +ve terminal of C and D or-ve terminal
.(b) Rate of heat dissipation through R 1 will change· if of both C and D must be joined to A
position of battery and galvanometer are inter ( d) The resistance r must be smaller than total
changed . resistance of wire AB ·
.(c) If e.m.f. & is doubled still no deflection is shown 9. A uniform wire of resistance R is shaped into a regular
by galvanometer. n-sided polygon (n is even). The equivalent resistance
(d) If galvanometer is not ideal it will show deflection between any two corners can have :
if e.m.f. is doubled.
.(a) the maximum value !I:_
(d) Initial rate of heat dissipation in 1 will be less than 4
in 2.
(b) the maximum value R
4. The area of cross section 2 n
of a current carrying
-conductor is A and A: (\) _ 0
(c) the minimum value R( nn~ 1)
0

at section (l) and (2) 2 (d) the minimum value !I:.


1 n
respectively. If v a,, Va ·· ·
and E1 , E 2 be the drift velocity and electric field at
sections 1 and 2 respectively then :
(a) va, :va 2 ;1: 4 (b) va, :va, ;4: 1

www.puucho.com
Anurag Mishra Electricity and Magnetism with www.puucho.com

10. Two cells of unequal emfs, •14. A galvanometer has a resistance of 100 Q and a full
&1 and &2 and internal · scale range of 50 µ A. It can be used as a voltmeter or
resistances r1 and r2 are as a higher range ammeter provided a resistance is
joined as shown. VA and added to it. Select the correct range of resistance
Vn are the potentials at A combination(s) :
and B respectively : (a) 50 V range with 10 kn resistance in series
(a) One cell will continuously supply ene_rgy to the (b) 10 V range with 200 kn resistance in series
other. . · · · (c) 5 mA range with 1 Q resistance in parallel
(b) The potential difference across, both the cells will (d) 10 mA range with 1 n resistance in parallel
be equal. 15. In the circuit shown in the A c· F
figure : .
_[
(c) The potential difference across one cell will be
greater than its emf. (a) &= 6.6 V ,S I 4V'
12V
l
(d) VA -V8
(&1r2 + &2r,) (b) 11 = 1.1 A
. (c) 12 = 0.5A 2n 4n 6n

11. In the network shown,


(d) all of these
B (ovj 1 0.5A
points, A, B and C are at D G
potentials of 70 V, zero
1. 10n 16. Voltmeter reads the potential difference between the
and 10 V respectively. A--'1/1(1,---c<
(a) Point D is at a (70V) D terminals of an old battery as 1.4 V while a
potential of 40 V : potentiometer reads its voltage to be 1.55 V. The
(b) The currents in the C (1?V) voltmeter resistance is 280 n. Then
sections AD, DB, DC (a) the battery of the cell is 1.4 V
are in the ratio 3 : 2 : 1. (b) the Mttery of the cell is 1.55 V
(c) The currents in the sections AD;DB, DC are in the (c) the internal resistance r of the battery is 30 Q
ratio 1 : 2 : 3. · (d) the internal resistance r of the battery is 5 Q
(d) The network draws a total power of 200 W. 17. In the shown circuit :
12. In the circuit shown, the cell ·has emf = 10 V and (a) current passing through 2
internal resistance = 1 n : n resistance is 2A
3n 2n· ,2n (b) current passing through 3
n resistance is 4A
(c) current in wire D to earth
an an 4n' is zero
(d) potential of point A is 10 V
· 18. ·Two concentric metallic shells of radius R and 2R, out
2n 2n ' :.. w. of which the inner shell is having charge Q and outer
(a) The current through the 3· n resistor is 1 A shell is uncharged. If they are connected with a
(b) The current through the 3 Q resistor is 0.5 A · conducting wire, then:
(c) The current through the 4 Q resistor is 0.5 A (a) Q amount of charge flow from inner to outer shell
(d) The current through the 4 n resistor is 0.25 A cb) g_e· number of electrons flow from outer to inner
13. When some potential difference is maintained
between A andB, current! enters the network at A and '. shell where e = 1.6x 10-19 c
2
leaves at B:
(c) KQ amount of heat is produced in the wire
20n C 5fl 4R
2

;A B
(d) KQ amount of heat is produced in the wire
2R
19. Both terminals of a battery of emf
sn D ~0!1
& and internal resistance r are
(a) The equivalentresistance betweenA andB is SQ grounded as shown. Select the
(b) C and D are at the same potential correct alternative(s) :
(c) No current flows between C and D (a) Potential difference across A
31 and Bis zero
(d) Current - flows from D to C
5 (b) Potential difference across A and B is &
(c) Current across AB is zero
(d) Current across AB is &
r

www.puucho.com
Anurag Mishra Electricity and Magnetism with www.puucho.com

20. A current passes through a wire of non-uniform 2R 2R 2R 2R


cross-section. Which of the following quantities are
independent of the cross-section? 1 ~
(a) the charge crossing in a given time interval
(b) drift speed 2 ~
' ' CircuitA
(c) current density
(d) free-electron density ,2R 2R 2R 2R
21. A battety is of emf & is being charged from a charger
such that positive terminal of the battery is connected
to terminal A of charger and negative terminal of the
battery is connected to terminal B of charger. The
~:EEE.-r--i--ro Circuit B
internal resistance of the battery is r.
(a) Potential difference across points A and B must be (a) y > X (b) y = (-,/3 + l)R
more than&. (c) ,y = 2R2 (d)x-y=2R
(b) A must be at higher potential than B 25. In the circuit shows the readings R
(c) In battery, current flows from positive terminal to of ammeter and voltmeter are 4A - ~
the n_egative terminal and 20V respectively. The meters ·
(d) No current flows through battery are non ideal, the R is: V
22. A battery of emf & and v-
internal resistance r is (volt) (a) SQ (b) less than 50
connected across a 10 (c) greater than 5 Q (d) between 40 & 50
resistance R. Resistance R 26. In a potentiometer arrangement. &1 is the cell
can be adjusted to any establishing current in primary circuit. &2 Is the cell to
value greater than or equal ---+-~-=------1 be measured._A Bis the potentiometer wire and G is a
2 (ampere)
to zero. A graph is plotted galvanometer. Which of the following are the essential
between the current (I) conditiol} for balance to be obtained.
passing through the resistance and potential (a) The emf of & 1 must be grater than the emf of & 2 •
difference (\/) across it. Select the correct alternative
(b) Either the positive terminals of both & 1 and & 2 or
(s):
(a) internal resistance of battery is SQ the negative terminals of both & 1 and & 2 must be
(b) emf of the battery is 20 V joined to one end of potentiometer wire.
(c) maximum current which can be taken from the (c) The positive terminals of & 1 and & 2 must be
battery is 4 A joined to one end of potentiometer wire.
(d) V - I graph can never be a straight line as shown in (d) The resistance of G must be less than the
figure. · · · resistance of AB
23. Two identical fuses are rated at l0A. If they are joined: 27. In a potentiometer wire experiment the emf of a
(a) in parallel, the combination acts as a fuse of rating battery in the primary circuit is 20 V and its internal
20A resistance is SQ. There is a resistance box in series with
(b) in parallel, the combination acts as a fuse of rating the battery and the potentiometer wire, whose
SA resistance can be varied from 1200 to 1700.
(c) in series, the combination acts as a fuse of rating Resistance of the potentiometer wire is 75 Q. The
lOA following potential differences can be measured using
(d) in series, the combination acts as a fuse of rating this potentiometer.
20A (a) 5 V (b) 6 V
24. Two circuits (shown below) are called 'Circuit/\ and (c) 7 V (d) 8 V
'Circuit B'. The equivalent resistance of 'Circuit a' is x 28. In the given
and that of 'Circuit B' is y between 1 and 2. potentiometer
.A ~j
circuit, the
G
resistance of the
potentiometer wire AB is R 0 • C is a cell of internal
resistance r. The galvanometer G does not give zero
deflection for any position of the jockey. J which of the
following cannot be a reason for this?

www.puucho.com
Anurag Mishra Electricity and Magnetism with www.puucho.com

[308 ELECTRICITY, MAG~ETISM 1


(a) r >R 0 (a) the intensity of light bulb A increases
(b) R » Ro (b) the intensity of light bulb A decreases
(c) emf of C > emf of D (c) the intensity of light bulb B increases
(d) The negative terminal of C is connected to A (d) the intensity of light bulb B decreases
29. A metallic conductor of --- ·---- · ---- ·-- ·- - , 33. Consider the circuit shown in the Fig.
!rregular cross-~ection .(1)
1s as shown m the . ~ ·
•P ~ . (1)' ,a : . 5Q_A 3Q. '3Q.

figure. · A constant · · · · · · - · · · ·· ··
:2sv 1QQ 1QQ 4Q
potential difference is applied across the ends (1) and I
(2). Then:
(a) the current at the cross-section P equals the
40 2!'.lB3Q
current at the cross-sectiol). Q
(b) th~ electric field intensity at Pis less ihan that at Q (a) the current in the SQ resistor is 2A
(b) the current is the SQ resistor is lA
(c) the rate of heat generated per. ·u~lt ·,ime at Q is
(c) the potential difference VA -V8 is SV
greater than that at P ·
(d) the potential difference VA -V8 is-SV
(d) the number of electrons crossing.per unit area of 34. The ammeter connected in following circuits has zero
cross-section at P is less than that at Q. resistance. The voltmeter in (B) has infinite resistance
30. A conductor is made of an ·· · · and a reading SV. The value of resistance R has ·not
isotropic material and has shape of I ~ - '· i
been specified. Which of the following circuit(s) has
a truncated cone. A battery of : '
same current in the ammeter?
constant emf is connected across it ·. + - ! ··20v· ----
and its left end is earthed as shown ,
! , '
-=, /
I
in figure. If at a section distant x
from left and, electric field intensity, potential and the
R

'h
. _ _,_ _ . . .,
rate of generation of heat per unit length are E, V and (a)
H respectively, which of the following graphs is/are
correct?

ca/El . • ,_ 1

1o~x1
\, ""'- ~--.'
(b)lo. ... ____ ,. _ x__:
(b) .
'Ht• / '
6Q
(c<oLx
' I
20 v·- · ... • t

31. 1\vo bulbs consume same power when operated at 200 .!


V and 300 V respectively. When the two bulbs are 4Q 8Q•
connected in series across a DC source of 500 V then : (c)
(a) ratio of potential difference across them is 3 : 2
(b) ratio of potential difference across them is 4 : 9
(c) ratio of power consume across them is 4 : 9
(d) ratio of power consume across them is 2 : 3 20V 60
32. Two light bulbs shown in the circuit have ratings ,=---,-{A
A(24V,24W) and B(24Vand36W) as shown. When
I 6Q
the switch is closed:
r---~· ·· - - -- --- ---· · ·--- - i (d) •
i . 6Q
;12v A
.... 6Q_ - -- • -- .'
1.,· L____::
'r--
f 12V .
(24V,24W)
B
(24 V, 36 W)
35. In the circuit diagram shown in the Fig. Which of the
following is true:

! -·-- - _;

www.puucho.com
Anurag Mishra Electricity and Magnetism with www.puucho.com

..
1 ELECTRIC CURRENT
i. - - · · , ___ ·-·- •

D sn sn A 3 7. In the circuit diagram each resistor of resistance SD..


The points A and B are connected to the terminals of a
cell of emf 9 volt and internal resistance 2/3 D..
15V
sn sn

C sn
(a) The points A and C are at the same potential
(b) A is at a higher potential than C
(c) Magnitude of p.d. between A and C is 5 volt
(a) The rate at which heat is produced in the cell is 6W
(d) C is at higher potential than A
(b) The current in the resistor connected directly
36. In the potentiometer circuit of given Fig. the between A and B is 1.2A.
galvanometer reveals a current in the direction shown (c) The current in the resistor connected directly
wherever the sliding contact touches the wire. This between A and B is 1.8A.
could be caused by: (d) None of the above
p &,
38, Which of the following does not have the same
dimensions as the henry?
2
,Q.._--v1.M/\/\M/\r--" (a) joule . (b) tesla- m

~
(ampere) 2 (ampere) 2
(c) ohm-second (d) 1
&, - farad - second
(a) &1 being too low (b) r being too high
(c) a break in PQ (d) &2 being too low

=
AN9WER9

ievel~2: ~~ tha~ One_.Al~~;~atives are C~r~;-i:~7:::::-,_


1. (a, c, d) 2. (d) 3. (a, b, c) 4. (a, d) 5. (b) 6. (a, c, d)
7. (b, d) 8. (a, c) 9. (a, c) 10. (a, b, c, d) 11. (a, b, d) 12. '(a, d)
13. (a, b, d) 14. (b, c) 15. (a, b) 16. (b, c) 17. (a, b, c) 18. (a, b, c)

19. (a, d) 20. '(a, d) 21. '(a, b, c) 22. ,(a) 23. (a, c) 24. (a, b, c)
25. (c) 26. (a, b) 27. (a, b, ,) •'. 28. (a) 29. (a, b, c, d) 30. (b, c)
31. (b, cl 32. 'Cb, c) 33. (a, b, c) I
34. '(b,.d) 35. (c, d) 36. (a, b, c)
37. (a, b) 38. (b, d)

www.puucho.com
Anurag Mishra Electricity and Magnetism with www.puucho.com

f,, ---¥··--,-~-.-- - -~ -··


1310 ' '
--·--~··--"'---- -.-
4. [a, d]
=·Level-~ci&.lha~OneftJ~maW~~~C':~~ ·_ 1"~=~ Current does not depend on area
1. [a, c, d] => Current will be same through the two sections
From Kirchhoffs law Ao
neA 0 xva 1 =ne va 2
-6RI 1 -IR+ & = 0 ... (i) 4
-2R(I -I 1 )-(I -I 1 )R-IR + & = 0 Val 1
--=-
6R ~' Va2 4
11 ..---w.---{")'---,
2R A2 R eE-r:
1 Va 1 E
Also Va=-=>--=-
1-1,
. -
me Va2 E2

& R 5. [b] R
max
=R·2a
6-a
I R . = R-a
On solving I1 =-
3 mm 2a,b
2I Rmax 4
I-I 1 = -
3 Rmin 1
=> Reading of A1 =I 7. [b, d]
Reading of A 2 = 3!_ In first case
3 V=&xR &
and A = - -
Reading of A 3 =~ R+r R+r
3 In second case
2. [d] Since 3A is in upper part of circuit. Therefore out of
V= &xR ·e
SA coming in lower part, 3A has to go to the upper
part. Out of which some part will flow through R2
and rest through the unknown resistance.
.
~--'\M,-~I
I •
. 3A
I~

A=[R:~],C . ~ •
r 2

------.L....--s,,....----
R2
.-(unknown)

L-_,,.,_____..
3A

&
R

&
=-->---
+-
2 '

2A R1 2R+r 2(R+r)
2A
Clearly V increases and A becomes slightly more
2A will go through R1 . then half of its initial value.
3. [a, b, c] 8. [a, c]
On interchanging Galvanometer and battery if & is less than &' then no neutral point will be
Since still R1R4 =R2R3 (wheatstone bridge found. Since potential drop across whole wire AB is
condition is satisfied) & which is less than & '.
If different terminals of C andD are joined to either
of A and B. Starting from A if potential increase
along AB then it will decrease in wire containing 'r'
'by&.
No neutral point will be obtained.
9·. [a, 'c]
No deflection The resistance will be maximum if it is divided into
Initially current through R 1 was two equals parts i.e., !!c2 each, and it will be
'
&
I1 = - - - minimum if it is obtained across one of the sides;
R1 +R2
because if two resistances are connected in
Now current through R 1 after interchanging is parallel, the resultant is smaller than the smallest
&
I2 = - - - resistor.
R1 +R3 10. [a, b, c, d]
Since current has changed. Let
Rate of heat dissipation changes.
Since current through galvanometer is zero.
It will always show zero deflection.

www.puucho.com
Anurag Mishra Electricity and Magnetism with www.puucho.com

ELECTRIC CURRENT _31.!J

AC~, E2 r2
or

14. [b, c]
20I 1 -5([ -I 1 ) = Oar I 1 =-

2I 3I
:.I-2I 1 =I--=-=current
5 5
fowmg
I
5
l " f r omDto. C

For converting it to voltmeter 'R' is joined in series


I= &2 -&1 cross checking the options
r1 + rz SOµA 100Q R
VA -&1 -Ir1 =V8 A•-->--...,.,._rv----'W---eB
VA -V8 =&1 +Ir, VAB = 50(100 + R)
=&1 + (&2 -&,)r1 For R = lOx 10 3 0.
r1 + rz VAB = 50(100 + 100 x 100) x 10-6 V
_ &1r2 +&2r2 = 50x lOOx lOOx 10-6
VA - VB -
r1 + rz = SOx 10-2
3
Potential difference across each cell will be For R = 200 x 10 0.
VA-VB. VAB = 50x 10-6 (100+ 200x 10 3 }
i.e., equal also VA -V8 = &1 + Ir1 > &1 = 50x 10-6 x 2x 10 5
Cell of e.m.f. &1 is absorbing energy provided by =lOV
cell of e.m.f. &2 For converting to ammeter low resistance in
11. [a, b, d] parallel
Let V = potential at D
70-D =lOI1
V-0 = 20I 2
V-10=30([1 -I2) R
Solve for I 1 , I 2 and V
12. [a, d]
For R =l, I= Sx 10-3
5x 10-3 x 1 = SOx 10-6 x 100
It is obvious that net resistance = 9 n + internal
5 x 10-3 = 5 x 10-3 i.e. ,it satisfied.
resistance (lQ.) = 10 n
10 15. [a, b]
So, current through cell or 3Q. resis~or = = lA. I1=I 2 +0.5 . .. (i)
10
Current distribution in different resistors is shown Using Kirchhoffs law
A C F
in the following figure.
30 2Q 2h ..I J_ l
&12Q 4V
__ l1A O.SA 0.25A
1ovj_ O.SA 0.25A
an BQ 4n 2n
4Q 6Q
1n 1,

2n 2n 2n BO.SA D 12 G
13. [a, b, d] Vv -4I1 + Ixl2-4-6I 2 =Vv
As C and D are joined, they must be at the same 4I1 +6I 2 =8 ... (ii)
potential, and may be treated as the same point. From eqn. (i) and (ii),
This gives the equivalent resistance as 8 n. U we I 1 =1.lA, I 2 =0.6A
distribute current in the network, using symmetry.· Now
20n 20Q V8 -4(1.1) + 12-&-2x 0.5 = V8
C & =12-1-4.4
A SQ SQ B
D & =6.6V
16. [b, c]
Potentiometer reads the voltage of battery and
20Q i,c SQ 1-1 1 voltmeter reads the potential across the terminals
of battery
B
A-----1 1-21, battery of cell is of 1.55 V
H, VAB =1.4V
SQ D 20Q I= 1.55
280+ r
VA -Vv =VA -Ve

www.puucho.com
Anurag Mishra Electricity and Magnetism with www.puucho.com

- ,_ --

~. . i
1.4V = 280x
1.55
r~ VA -& +Ir= V8
VA -V8 =&-Ir
280+r 0 = &-Ir
r X1.4 = 280 + (1.55 -1.4) &
I=-
rx 1.4= 280x 0.15 r
2.8 31. [b, c]
r =-X 15 =30.!J
1.4 Let resistance of bulbs are R1 and R 2
17. [a, b, cJ (200) 2 = (300) 2 ·
Ve -6-2!1 -2+ 3(J -I 1) = Ve
R1 R2
8=3I-5J 1 ... (i)
R2 9
Ve -3(I-I 1 ) + 2+ 10 = Ve -=- ... (i)
3J-3J1 =12 ... (ii) R1 4
R, R2

~tz1m JJ!'.
11 ~--w----~w"'"'"-~
A B '

1, '
1'-·-----'I '-I- - - - - - ' ·
l
C D 500V .. .

From eqn. (i) and (ii),


1-11 3Q
V, =IR1 =( 500
R1 +R2
R, )x
I 1 = 2A 500 500 2000
I=6A = l + R2 = l + _SI = 13
J-I 1 =4A
R1 4
Also current in wire D to earth is 0.
500
18. [a, b, c] ½ xR 2
Initially the system is. shown in figure. Initial R1 +R2
energy stored +Q)n sys_t!!_m 500 4500
_ __ is =--4 =-3-
1+-
9
V1 2000 4
-=--=-
½ 4500 9
P1 (V,J2R2 =(V, )2 x R2
P2 · R1 (Vz) 2 ½ R1

u, =--=--
Q2
8neoR
KQ2
2R =(iJ2 x!=i
As the two shell are connected, the entire charge Q 32. [b, c]
passes through wire and resides on outer shell, so v2
For a bulb R=-
that both the shells have same potential. W
Final energy stored in system is ~ Rn :::RA
Q2 KQ2 when switch is open I A =I 8
ut =--
PA =RAJ!
8ne0 x 2R 4R
So, amount of heat generated in wire is, PB =R 8 Ii
KQ2
MI =U;-Uf = 4R PB< PA and
VA> 12V and
19. [a, d] VA = 0, VB = 0 (as A andB are earthed)
After closing the switch
VA -VB =0 V =V8 =12V
From Kirchhoff's law
~ PB =36W
PA =24W

www.puucho.com
Anurag Mishra Electricity and Magnetism with www.puucho.com

[ffECTRl~C~-~~E~[ __·_ ....... 313]


- - "'"'"

50 5!1
33. [a, c]

5!1 A 3!1 3!1 50 5!1

28V 10!1 100 40 5!1 5!1 B


V0 =+15
V0 -lOxl=VA
4!1 2!1 B 30
VA =15-lO=SV
5!1 A 3n Ve =15-Sxl=lOV
37. [a, b]

= 28V 10!1 100 40 -

4!1 2!1 B

5!1 A 3!1 50

5 5
28V 100 5!1 = 28V 10!1 100

5/2_~ '
4!1 20 B 40 B
LJ15/8 ·

15/2
A 5 B

35/8

I= Current through SQ resistor


28
5+5+4
2A =,__c:\ '" 5

Ao
I 1 =(__2Q_)I =_!=lA So, eg. circuits diagrams
10+10 2
7/3
VA -VB =5I 1 +3I 1 =8I 1 =SA
34. [b, d] 9
(a) Req = 3!1 I =~=3A
· · · c a) I A =~=SA
For ClfCU!t 20 R,q
4 9V 2/3
20-V 20-8
For circuit (b) I A = - - = - - = 2A Heat produced in cell =I 2 r=9x (2/ 3)=6W
6 6
For circuit (c) I A =0 (Balanced Wheatstone's
Bridge) -~5/8
For circuit (d) 6Q is in parallel with 6+ 6=12Q. (b) ~
. " . . . 6x12 Q
Their euectlve resistance 1s - - =4
6+12
Net resistance of the circuit= 4 + ·6 = lOQ R1 I
Iz
I A = 20/10 = 2A R 1 +R 2
35. [c, d] 3x (35/ 8) =:Z:=l. 2 A
Current 1n . . I =lS
. th e circuit -x 2 35 +5 5
. 15 8
I =2A So, (a) and (b) are correct.

www.puucho.com
Anurag Mishra Electricity and Magnetism with www.puucho.com

L·314. _______ 'iLECTRICITY&·MA~!![~j

·A thin uniform wire AB of unknown resistance is ·connected


between points A and B. AP and QR are thick conducting
stripes. A battery of unknown ·e.m.f. & and• a galvanometer:
(with a sliding jockey connected to it) are. available.;
:connectiol)~ are to be made to find e.m.f. &, resistance of:
1wire AB and length of AB. The battery with galvanometer is'

·connected between points.A and B in two alternative way


·such that jockey when pressed on wire AB at distance 80 cm
.and I20 c.m respectively from end A, galvanometer shows 1. Reading of ammeter when PS
,
=.!..n is:
'.no d_efl_ec\iOJl.

··1~1'_'1~
A B
:-:0] R
(b)

(d)
(1-~)(r+&~)
(1- 1) (r+ :o)
n2
& .

'If one end. of battery with galvanometer is connected to pt. 2. Find the value of PS for which reading of ammeter is
'Rand jockey is pressed on AB at di;tance 180 cm from end' min.:
'4, .tile .<leflec:tion jn gajvanometer is. zero .then fil)d..,: ___ (a) 21 (b) i2
3
1. The length of wire :
(c) 31 (d) 1
(a) 200 cm (b) 280 cm 4
(c) 320 cm (d) ,300 cm . 3. Find the value of PS for which reading of ammeter will
2. The resistance of wire AB is : be max.:
(al
. 3
.!n (bl 3 n (a) 0 (b) i
2
10
(cl n (dl 10 n (c) 1 (d) 21
3 3
3. The e.m.f. of battery is & = :
(a) 2 V

(c) 4 V ,,.----·-•""
(b) 5 V
(d) 10 V
a)
2 "'-'. In the circuit shown, both batteries are ideal. 'EMF &1 -of.
:battery 1 has a fixed value, but etnJ' &2 ofbattery2 can be;
, C.........=..c~C,: ..,~.~~• / I
varied between l V and 10 V" The graph gives the. currents'
,In the figure, ,PQ is a wire of uniform cross,section andi 'through the. two batteries as. a function of &2 , but are nor;
,resistance R 0 • A is an ideal ammeter and the cells are of: m;rked as which plot corresponds to which battery. But for:
!negligible .internal resistance. ·The jockey J can be· freely; ·both plots, ,current is assumed to be negative when thei
:slide ov~rwirePQ making contact on it at S. Length of wire[ direction to the current through the battery is opposite the:
1
,_.p_Q,_is, {'. rn """ - ,. "' " - • ·---· __ '. · direction of that battery's emf. (Direction of from'Jiegative[
to positive)

www.puucho.com
Anurag Mishra Electricity and Magnetism with www.puucho.com

rL-.~-·-----,_
EiEaRJc___ctiRRiNr
~-·~- ____ j1s1
0-4
10Q

~0-2 9Q 3Q 12Q
"E
~ 6Q 2Q
R, 0 15Q
&2
+
R2
"
()
10 &z(V)
+
&,
-0.2

1. The value of emf &1 is:


(a) 8 V (b) 6 V 20Q
A
(c) 4 V (d) 2 V
2. The resistance R1 has value :
(a) lOQ (b) 20 Q 1. The current through 12 Q resistance is :
(a) 0.1 A • (b) 0.75 A
(c) 30 Q (d) 40 Q
(c) 0.5 A (d) 1.25 A
3. The resistance R 2 is equal to :
2. The reading of the voltmeter connected across 20 Q
(a) lOQ (b) 20 Q
resistance
(c) 30 Q (d) 40 Q (a) 15 V (b) 10 V
(c) 5 V (d) 22-5 V
3. The reading of the ammeter is
(a) 0.5 A (b) 2.25 A
A car battery with a 12 V emf and an internal resistance of (c) 1.5 A (d) 0.1 A
0.Q4 Q is being charged with a current of 50 A.
1. The potential diffei;ence V across the terminals of the 6
battery are :
(a) 10 V (b) 12 V ·A network of resistance is constructed with R1 and R 2 as
(c) 14 V (d) 16 V shown in Fig. The potential at the points 1, 2, 3_ .. N are
2. The rate at which energy is being dissipated as heat ·v,, V2 , V3 , ••• Vn, respectively, each having a potential K
inside the battery is : times smaller than the previous one.
(a) 100 W (b) 500 W v, Vz V3 Vn-1 R1 N
(c) 600 W (d) 700 W Vo
R1 R1 R1
3. The rate of energy conversion from electrical to Rz Rz Rz Rz R3
chemical is :
(a) 100 W (b) 500 W
(c) 600 W (d) 700 W
-- . - - - . . - _ <>·,-~ ¼-~------·- 7' 1. The ratio Ri is:

;'.;fl~i!f'lit~ ~'. 5 (a) k2 _I_


R2

(b) k ~l
Fig. shows two ideal voltmeters and an ammeter which are_ k
connected- across the various circuit elements. If the (c) k-_!_ (d) (k-1)2
voltmeter connected across 9 Q resistance. reads 4.5 V, then k2 k
answer the following questions. 2. The ratio R2 is:
Ra
(a) (k-1)2 (b) k 2 _I_
k k
(c) _k_ 1
(d) k - -
k-1 k2

www.puucho.com
Anurag Mishra Electricity and Magnetism with www.puucho.com

I •. ··-
i 316_~ · .. --··--
3. The current that passes through th_e resistance R2 1. What is the equivalent resistance of the system aboutA
nearest to the V0 is: and C?
2 2
(a) (k-1) V0 (b) (k + 1) V0 (a) r (b) !:.
k R3 k R3 2

(c) (k+~)Vo k 2
Rs
(d) (k-~)Vo k 2
Rs
(c) 3r
2
(d) 2r

2. Find the ratio of the power developed in segment AE


,.~ to that in segment HM:
7 /,· (a) 1 (b) 2
(c) 3 (d) 4
"Relation between current in conductor and time is shown in 3. If a potentiometer circuit having potential gradient k is
Fig. then determine connected across the points H and C, find the
' . ,',' .· '
balancing length shown by the potentiometer:
(a) ::. (b) 2v
k 3k
(c) 3v (d) none of these
2k

9
1. Total charge flown through the conductor is All bulbs. consume same power. The resistance of bulb 1 is·
(a) J 0 t 0 /2 (b) I 0 t 0 360 . Answer the following questions
(c) I 0t 0 / 4 (d) 2I 0t 0
2. Write the expression of current in terms of time :
(a) I. =I O - t (b) I =Io ( 1 + -t )
to . . to

(c) I=Io(:
0
-1) (d)I=Io(l-ttJ 4

3. If the resistance of conductqr · is ·k, then total heat ____, HI - - - - - '


dissipated across resistance R is'
2 . 2
(a) 10 Rt 0 ·(b) 10 Rt 0 1. · What is the resistance of bulb 3 ?
2 . 4 (a) 4 W (b) 9 W
(c) 12 W (d) 18 W
(c) 15 Rto (d). I20Rto 2. What is the resistance of bulb 4?
3
(a) 4 W (b) 9W

In Fig. each of the segments (e.g:, AE , GM, etc.) has,


'
8 L -~
(c) 12 W

of each bulb is 4 W?
(d) 18 W
3. What is the voltage output of the battery if the power

(a) 12 V (b) 16 V
resistance r . A battery of e.m.f. S is connected between A:
(c) 24 V (d) none of these
and C. Internal resistance of the battery is negligible.
D G C
r r

r r r

r r
H F
M
r r r

r r
B
E

;I--"'---
& s

www.puucho.com
Anurag Mishra Electricity and Magnetism with www.puucho.com

ELECTRIC CURRENT
-~-'-'------ ".31'7:
·... ~-'MATdlitING TYPE ·P~dillE~'-,,_,,;:"s
C- - _, ~ -···f"··--- - ... "'"·· --·-~"'-w..,-,. . - . --~
4. In the circuit shown, match the following :
~,, 1 ' t1 · - - -...--A "~-a-· ·:

1. Six batteries of increasing emf and increasing internal


:·~:
'-------· '-------·
15V,1Q 10V,1Q
resistance are as shown in figure. Match the following:

.
icF~~-::·
.
.
:~v.::·_a:
:y~~- -
. D
3

(a) Potential difference across battery A (p) A


· ' \ '\.
..
Column-II : :

(b) Poteri.tial difference across batt'eiy B (q) B


Coluii)n-1 Column-II
(c) Power is supplied by battery (r) HV
(a) Potential of point A (p) zero
(d) Power is COQSctmed by battery (s) 9V
(b) Potential of point B (q) 2V
(t) None
(c) Potential of point C (r) 4V
(d) Potential of point D (s) 6.V 5. Current is flowing through a wire of non-uniform
cross-section. Cross-section of w,ire A is less than the
(t) Non·e -I cross-section of wire at B. Then match the following
2. In the potentiometer arrangement ;..
shown in figure, null point is obtained
at length I. Match the following :
:,t5" · -&~- . ·

1
J
·1
Ji

(a)
Column:
. Colim\i!,-1

current at A
:: __"f

(p) is zero
Column--11

I &, __ _ (b) drift velocity of electrons at A


(c)

electric field.in the wire at A
(q) is more than at B
(r) is Jess than at B
'
'\ Coll!l'!l!'.\-f '\ '\4J' Column-II (d) current density at A (s) is equal to that at B
(a) If&, is increased (p) 1should increase
6. A potential differel)ce Vis applied across a copper wire
(b) If R is increase ( q) I should decrease of diameter d and length I. In Column I the information
(c) If &2 is .increased (r) Ishould'remain the··! about the change in one of the physical quantity is
_same to_ again get the 1 given and the effect(s) of this change are mentioned in
null point Column II Match the Column I with Column II
.. ' . ~.
'\ cotymtf,1 '\ '\. ' ' ifoiumn-11 :1
3. In the circuit shown in figure, if a
resistance R is connected in parallel
(a) Voltage Vis•doubled (p) Drift Speed will decrease
with R 2 , then match the following : (b) Length 1is do4bled (q) Drift Speed will be halved
(c) Diamete( d is doubled (r) Drift.speed will be doubled. 'i
... (d) Temperature 0£ the (s) Drift speed will not change I
'\ Coluinri;;I '\ '\ wire is ~ncr~ased,, . 'i
(a) Main current I (p) will increase
7. Three bulbs A,B and Care having rated powers PA,PB
(b) Power across R1 (q) will decrease and Pc respectively, each bulb is designed to operate at
(c) Power across Ri (r) ~u remain same rated voltage V. It is given the PA > PB > Pc, In Column
I the three bulbs are arranged in different
configurations, while in Column II the information
about intensities of bulbs are mentioned. Match the
Column-I with Column-II. Neglect the variation in
resistance due to change in temperature.

www.puucho.com
Anurag Mishra Electricity and Magnetism with www.puucho.com

I 318 w 1,"
~--·_:.~------------ ... .. . .. ~
f., "'
ELECTRICl1Y&MAGNETISf!i_]
1
;,;,;.;;;,_

c:olumn-1 )' <'t\ \. ·9,,,r._m'i:f;f ·,, .1


(a)
·. Column-I Column-JI
(p) A is .having .th~ mini'
"
(a)
Y±,· ' ,
Ctlrtertt-j,assing through 40 (p)
,~,·
'·\'"'

possible if$.= 6 V I

·~ mum intetisity: cf resistance can ,be zero . .


- .

iI
(b) Cµqentpassing through 4h (q)
re:Sj$trinCe Cari'be fi:orr;tJttb C. ·
. .,
possible if Z .> 6 V I

V . '·I
_J ditec_tiort .. I
(q) C,is glowing with';maxi- Chtrent passing through'4fl (r) possible if. S. <:• 6 V I
(b) (c)
. !
.. . .
mum brightri~ss · resistance can be fromG
. to'F
,I

dii'ec~iOil I

iI (d) Ct;tri:ent,passing through 2fl (s) possible for:any·


I
resistance will be from B to A value of$ from I
.1
di~£!cti0ii I z~o to 'infinit · !
(r) B is:glowing withlnihiJ
mum brightness/. , . . :
I
10. Fig. shows an experimental setup for a potentiometer,
'lI point R is null point [no deflection of galvanometer]
i'
~---11---~1 $ 1 is primary source. Match the following:
r¥-- -~,.,-·--~- --,,.-...·--~- .
·/: V j
""'~ ·'.,·s·. ·c-··------·· I E: ,r ,. l
i
(d) (s) A.is glo\l(ing;~th maxi- !.
m1Jm brightness.,: ' .'
I
''!

.Y--------~--:::.... ....
[,
J.,. -------~~--al I
I

'
8. A network consisting of three resistors, three batteries,
·~n i
and a capacitor is shown in Fig.
,,::·(I,+ lz) B. • ·-lz- -~;;-, ~~:0~)1>\t..,... _e.o,__ J
!
sn 10v X an 4ni
~ 5j1F f (a) {f only emf, of.battery},i"is . (p) Point R wtll shift to
BV 12 I ~creaSed, ;, left ,_ ' -
F E _)2.v__ .o.· I (b)· ff:onlyresistance ofrheosl;atic (q) PointRwillshiftto i
is'.inci.'eased · ,,: · right :
Co_lilmn.J (c) If resistance box is co@ectecl . (r) Point R. mil:}' ·shift.to l
~n~ r~sistance RB is-inc,i'.~ased left oc right , I
(a) Curre.ntin.J;,ranch EB.is_ lOµC I ,.
Ifan ideal battery is ~on:;; '
(b) Curi:entin1branch CB is: O.SA.. I (d)
nectecl in parallel tO '$ I I ' . , . ...
. I
(c) Curi:entin brarichEDis 1.SA .I
(d) Char&~ on Capacitor is , (s) · SµC 11. The diagram shows a circuit r·-··-----.. ~7·-·8----
. with two identical resistors. !
I

v ·' .. ·· 1' · 1
The battery has a negligible i .
9. A circuit is shown in Fig. R is a· non-zero variable with
internal resistance. · Hu:u--iu:u-i!
finite resistance. S is some unknown emf with What will the effect on the :
polarities as shown. Match the columns.
1 ·s -·. ~c .· ""ol
amme_ter and voltmeter be if I
.;..--.-.._____,,JH ,
]." ' ! the switch S is closed? ~.......,.,,..~ ..J
l2n 4n R'
.II' - I- 6V
e
I j
, .:
I

l ~---_·;'i~.] I E'

www.puucho.com
Anurag Mishra Electricity and Magnetism with www.puucho.com

; ELECTRIC CURRENT 319


. . ...
\. Colu11;1n,I \ ,, Column-II ! \ ·. 'Column-I \ \ Column-II
.
(a) Ammeter reading (p) (a) (p)

~
Increases 0

(b) Voltmeter reading (q) Decreases


,
(c) Equivalent resistance of cir- (r) does not chang~
cuit o
(d) Power dissipated across R in (s) becomes zero
right branch. (b) ,---l20 V . (q) 2 ampere

12. Column-I has some conductor across which battery is \)

+
( R
connected as shown. Variation of resistivity p is also 30
indicated. Which of the quantities in column-II remain ...
constant throughout the volume of conductor. (c) 20V .· .. (r) 4ampere
Column~ Column-II
o
(a) (p) Magnitude of elec-
tric field
~ ""
120

(d) (s) 5 ampere


(b)

(c)
(q)

(r)
Magnitude of cur-
rent density

Power dissipated
[:g 60
o

per unit volume


14. A circuit i& shown in Fig. R is a non zero variable
variable with finite resistance. & is some unknown emf
/
with polar:itles as shown. Match the columns
B C D

(d) (s) Drift speed of free.: 20 40 R


electron

A
I t 6V
E
F

(t) Electric current· ·


'\ Cplumn-1 \. \. Column-II I
(a) Current passing through 4Q (p) possible if & = 6 V '
13. Column-I has four circuits each having an ammeter. resistance c'an be zero
Column-II has four values of current in the ammeter.
The ammeter has zero resistance. The voltmeter, in (B) (b) Current passing through 4Q (q) possible if & > 6V
has infinite resistance and a reading 8V. The resistance resistance can be from F to C
direction
R has not been specified. Match the circuit with its
correct ammeter reading. (c) Current passing through 4n (r) possible if & < 6 V
resistance can be from C to F
direction
(d) Current passing through 2Q (s) Possible for any
resistance will be from B to A value of & from
direction zero to infinity

www.puucho.com
Anurag Mishra Electricity and Magnetism with www.puucho.com

I 320 .• ELECTRICl~~ifv14~~ffiil

15. In the circuit shown in Fig. battery; ammeter and Column.•!


voltmeter are ideal and the switch S is initially closed (a) 'fe):Jninal potential dif4ferei>ce (p) R;,'r !
as shown. When switch S is· opened, match the across the cell to1 be maxirigim !
parameters of column I with the effects in column II
(b) Power transferred to R idess than (q) R<r
the maximum possible ·
'\ . ,. /•.:. Column-I . J'\ '\' Columr;,1..!J I I
(c) Power dissipated m the. cell is maxi· (r) R'=OO
(a) Equivalent resjstance acro~s
the battezy
(p) remains same
i
I
I)JUJll - -I
(d) Fastest. drift of ions m the electro' (s) R=O
(b) Power dissipated by left.resis- • (q)
tanc.eR .
increases
I
i
,l;,te in the cell will be for :

(c) Voltmeter reading (r) decreas1?5


.. ' 18 . For the circuit shown in Fig., 4 cells are arranged. In
(d) Amllletei: reading ·. (s) becdmes zero 1 ·column I, the cell number is given while.in Column·n,
some statement related to cells are given. Match the
Column I with Colum,,,n"-·~Il::.._ _
16. In a potentiometer experiment : .. •. •Cell I ·----- '1
. 10.Q ,,-·····:
'\ ,/... Column-I '\ '\ ,Colunjn,il>
,-d,_,.,,·,
: I W,A,,o-.0...~I
~------·
(a) Deflection of galvano· .me· (p) Accuracy-in measure-
ter·is.·in Same direction.at ment iri.cteases
I .f~l!!I...~Y,.~9.
-~W,/\,"-c'~
the two ends ofthe wire i i·,
~ •••... :
1V,Hl
~...••• !Celllll
3V,3Q
. '
(b) AJ)fQte_<;tive, resistance (q) Accurat:y in measure:-_ 1' ' l<'.lt&<----'
'
added m series to the gal• ment de~reas~s i ~::,X.: CelllV
vatiometer 1 _; ---2:V,,._2!1 · " ,

(c) A short wire is used as..a (r) e.m.f. of the battery m


potengometer the primary circuit. is Jess
. than the e.m.f. ofthe (a) Celli {p) , _Cherirtcal energy,of<:ell1fd¢~as/Ilg \
cell.to be measured
·'' (b) C~ll.II (q) · Chehiital energy pf cell is}nct~asing j
(d) more length of potenti· (s) u~c~rtamcy·m the;Joca-
:ometer·up to.null:point tion of balance. pomtin° I (c) .CeJI.III (r) Workldone byceJI is +"'.e .: ·
~reases' I (d) Cell IV (s) Th~rrnalenergy ileveli>ped ce!Lis l1
+ye·,··__ "' '·,,;. •
17. In Fig. the resistance R is variable, r is the internal
resistance of battery of e.m.f. & '
! . ~- ---- - :·;, • -·----·:

I
I E 'r

www.puucho.com
Anurag Mishra Electricity and Magnetism with www.puucho.com

,-
! ELECTRIC CURRENT
6. Stat!"ment 1: Two unequal resistances are
=L-~-A~·S~S-E,~R_T=IO_N_A_N_D_R_E~A_S_O_N~~·--'~ connected in series across a cell, then potential drop
across the larger resistance is more.
Direction: In the questions that follows two
Statement 2: The current will be same in both
statements are given. Statement-2 is purported to be the
unequal resistances.
explanation for statement-I. Study both the statements
7. Statement 1: Two unequal resistances are
carefully and then select your answers according to the
connected in parallel across a cell, then current
codes given below:
through the smaller resistor is more.
Select your answer as Statement 2: More current will flow through a
(a) If Statement-I is true, Statement-2 is true; larger resister.
Statement-2 is a correct explanation for 8, Statement 1: If a wire is stretched to increase its
Statement-I. length n times then its resistance also bec;ome n time.
(b) If Statement-I is true, Statement-2 is true, Statement 2: Resistance of the wire is directly
Statement-2 is not a correct explanation for proport;lonal to its length.
Statement-I. 9. Statement 1: When a wire is stretched so that its
(c) If Statement-I is true; Statement-2 is false. diameter is halved then its resist<3snce becom~ 16
(d) If Statement-I is false; times.
Statement-2 is true. Statement 2: Resistance of wire decrease with
1. Statement 1: A steady increase in length.
current is flowing in a 10. Statement 1: The value of temperature coefficient of
conductor having non-uniform • resistance is positive for metals.
cross-section as shown in Statement 2: The temperature coefficient of
figure. The drift speed of the : ~ resistance for insulator is also positive.
electrons in-creases as one 11. Statement 1: When an insulated wire is bent, its
moves from A to B. resistivity increases.
Statement 2: Dn"ft ve1oe1ty. 1s
. given
· by, vd = -eE-t and Statement 2: On bending, the velocity of electron
m decreases.
-> 12. Statement 1: If the radius of c:opper wire carrying a
in above described situation, E is increasing as one
current is doubled, then the drift velocity of the
moves from A to B.
electrons will become one fourth.
2. Statement 1: Potential difference across the
Statement 2: Drift velocity will change according to .
terminals of a battery can be greater than its emf.
Statement 2: When current is taken from battery, the relation, I = neAv d·
13. Statement 1: A wire of resistance R is bent in the
V = & - Ir (Symbols have their usual meaning).
form of a circle. The resistance between two points on
3. Statement 1: If the length of a conductor is doubled, circumference or the wire or at the end of diameter is
the drift velocity will become half of the original value R/4.
Statement 2: The resistance between the two points
(keeping potential difference unchanged).
on circumference of the circle will be the parallel
Statement 2: At constant potential difference drift
combination of two resistances of upper and lower
velocity is inversely proportional to the length of the
parts of the circle.
conductor.
14. Statement 1: The equivalent resistance in series
4. Statement 1: Current flows in a conductor only
combination is larger than even the largest individual
when there is an electric field within the conductor.
resistance.
Statement 2: The drift velocity of electrons in the
presence of electric field decreases. Statement 2: The equivalent resistance of the
parallel combination is smaller than even the smallest
5. Statement 1: A piece of copper and other of
resistance.
germanium are cooled from room temperature to 100
15. Statement 1: When a battery is supplying power to a
K conductivity of copper increases and that of
circuit, work done by electrostatic forces on
germanium decreases.
electrolyte ions inside the battery is positive
Statement 2: Copper has positive temperature ·
Statement 2: Electric field is directed from positive
coefficient where as germanium has 'negative
to negative electrode inside a battery.
temperature coefficient.

www.puucho.com
Anurag Mishra Electricity and Magnetism with www.puucho.com

AN8WER8

Passage-1: 1. (a) 2. (c) 3. (c) Passage-2: 1. (b) 2. (b) 3. (a, c)

Passage-3: 1. (b) 2.(b) 3. (d) Passage-4: 1. (c) 2. C•l 3. (c)

Passage-5: 1. (c) 2. (a) 3. (b) Passage-6: 1. (d) 2. (c) 3. (d)

Passage-7: 1. (a) 2. (d) 3. (c) Passage-8: .1. (c) 2. (d) 3. (b)

Passage-9: 1. (b) 2. (a) 3. (b)

= f:ia'tcliihg Txpe P~obJ~in~


1. (a)-p; (b)- p; (c)-p; (d)- p 2. (a)-q; (b)- p; (c)- p
3. (a)-p; (b)-p; (c)- q 4. (a)-r; (b)- t; (c)- p; (d)- q
5. (a) s; (b)- q; (c)- q; (d) -q
0
6. (a)-r; (b)-p, q; (c)-s; (d)-p
7. (a)-p, q; (b)-q, r; (c)-s; (d)-p 8. (a)-r; (b)-q; (c)-q; (d)-p
9. (a)- q; (b)-p, q, r; (c)-q; (d)-p, q, r, s 10. (a)-p; (b)-q; (c)-q; (d)-r
11. (a)-p; (b)-p; (c)-q; (d)-q, s 12. (a)-q, s, t; (b)-q, s, t; (c)-p, t; (d)-t
13. (a)-s; (b)-r; (c)-p; (d)-q 14. (a)-q; (b)-p, q, r; (c)-q; (d)-p, q, r, s
15. (a)-q; (b)-r; (c)-r; (d)-r 16. (a)-r; (b)-s; (c)-q, s; (d)-p
17. (a)-r; (b)-p, q, r, s; (c)-s; (d)-s 18. (a)-q, s; (b)-p, r, s; (c)-p, r, s; (d)-q, s

1. (a) 2. (b) 3. (a) 4. (c) 5. (a) 6. (d) 7. (c) 8. (d)


9. (c) 10. (c) 11. t'(d) 12. (a). 13. (a)· 14. 1 (b) 15. (d)

www.puucho.com
Anurag Mishra Electricity and Magnetism with www.puucho.com

= ~evel-3:.C.?mpr_ehension Based Pr~le~~ :----._°"


Passage-1
1. [a] In first case batteries was connected through point Passage-2
=3x-x-

&=4V
16 2
3 5
323

A and in second case battery was connected 1. [b] Vp -VQ =2&


through point B VA =Vp -&
Let length of wire AB = l In length I potential difference 2&
2
P 15V 2/30 Q =} In length .i_ potential difference = &
n n
2&
=} Vs =Vp - -
n
• VA=Vp-&
A R
B
Resistance of length J. will be Ro
n n
Vs-VA =&(1-¾)
Resistance of AB = R
Accordingly I= ammeter reading=(1-;)( &Ro)
When battery was connected through the point A r+-
15 R n
&=--x-x80 ... (i) 2. [b] From above I min = 0 for n=2
~+R l
3 =} for PS = i_ reading will be 0
2
When battery was connected through the point B
15 3. [a, c]
& = - x~x(l-120)
5
... (ii) For maximum reading potential differenc~Vs - VA I
-+R l should be maximum
3
for PS = 0, l reading is maximum .
Dividing both
l = 200 cm
2. [c] From third condition when battery was connected
to point R.
;I Matchin~ Type Proble'm >,,,:-:-,,,,
6. (a)-r; (b)-p, q; (c)-s; (d)-p
efa
Vd=-
m
If Vis doubled, then E gets double and hence v d.
R
If I is doubled, then E gets half and hence v d
A B
decreases. If dis doubled, then there is no effect on
vd.
& If temperature increases then relaxation time -1- and
hence vd .
& = -15
- X [ l + -RX (200-180) ] ... (iii)
~+R 200
3
7. (a)-p, q; (b)-q, r; (c)-s; (d)-p
Divide (i) by (iii), As PA > PB >Pc and the rated voltage is same for
· all, their resistances will satisfy the relation
l+~ = Rx 80
10 200 RA <RB <Re,
2R R 3R For (a) As all A, B and C are in series, current
1=---=- through them would be same to I 2 R is maximum
5 10 10
10 for C and minimum for A. C is brightest and A is
R= n dimmest.
3
For (b) If current through A and B be I I and I 2
3. [c] Put 10 . eqn. ·c·)
R =-m
3
1 then current through C is I 1 + I 2 • So I 2R is
10 maximum for C and as A and B have same applied
We get
15 3
&=--x-x80 voltage across them so -
V,2
1
-
V,2
> - 1- so power
5 10 200 RA RB
-+-
3 3 delivered to B is minimum.

www.puucho.com
Anurag Mishra Electricity and Magnetism with www.puucho.com

_ELECTRICITY & MA(;NETISM :


For (c) As voltage across all the.bulbs is -same so R-8+28>0 => & >4-~
2
2
.!:'R.:_ is maximum for A and minimum,for
, . C. Depending upon the value of R, & can take any
For (d) Here bulb A is shorted so nci:cur~ent flows value from zero to infinity.
through it, while Band C are'in piu'allei so power 11. (a)-p; (b)-p; (c)-q; (d)-q, s
delivered to B is greater than to C. So, B is glowing R,q. decreases => 1 t => Vt
brightest and A is dimmest. 13. (a)-s; (b)-r; (c)-p; (d)-q
8. (a)-r; (b)-q; (c)-q; (d)-p (a) Effective resistance of the_ circuit =41J
When a steady state is reached, no current passes (b) Potential difference across· 31J=20V-8V =12V
through the capacitor or the branch CE. (c) Find currents in resistors using Ohm's law & series
Considering the loop ABEFA, parallel and then use Junction law to find current
in ammeter
C
,.. - (d) Effective resistance of the circuit =10/J

1. (a) Both statement are correct and Statement 2 is


12V D , , correct explanation of Stafement 1. Due to
·-- --··· - - '-·-- -- - _.,___ ->
Sx (1 1 +I 2 ) =10 decreasing cross-section area, the E increases and
or I 1 +I 2 =2A hence drift velocity as one moves from A to B.
Considering the loop BCDEB : 2. (b) When the battery is undergoing charging processes .
4I 2 =12-10=2 then,
I 2 =0.SA V=&+Ir >&
So, I 1 =2-0.5=1.SA So, Statement 1 is correct.
To find the charge on capacitor, we must known Statement 2 is also correct but not explaining
potential difference across the plates. Statement 1.
Consider the loop CEDC: pl V 1
-12+4I 2 +3x0-Vc +8=0 3. (a) V=IR=> V=neAva-=>vd =--=>vd oc-
. A nepl I
or Ve =-2V. So charge on capacitorQ=CV=lOµC
4. (c) Drift velocity is directly proportional to electric
9. (a)-q; (b)-p, q, r; (c)-q; (d)-p, q, r, s
field. If there is no electric field, then no drifting of
LoopFEDCF: &-6=RI 1 -4I 2 ... (i)
electrons in a particular direction, hence no current
Loop AFCBA: 6-4=4I 2 +2([ 1 +I 2 )
in the condnctor.
r .._____ - -- ..
2=2I1 +6I2 ... (ii)
-~l
5. (a) Copper is a conductor and germanium is a
8 C semi-conductor.
I 11+ 12
\ 6. (a) In series, current in both resistances will be same.
!2 4Q Ri' For same current, more is resistance more is the
I
I 1, potential drop.
I 6V
'
1,'
IF I 7. (c) Smaller is the resistance, more is the current in
A I --'=---,11--~E] parallel.
4V s
n "·"· ·~w-.-- 8. (d) It is true that. resistance of a wire is directly
Solving them we get: proportional to its length, but here when length is
I _3e-14 R+6-& doubled, area of cross-section decreases as the
i - 4+3R' 4+3R volume remains constant. Finally, resistance
(a) I 2 =0 => &=R+6 becomes n 2 times.
& > 6V (:. R ,e 0) 9. (c) The resistance ofa wire is R = £i (1)...
(b) For current from F to C direction A
I 2 >0 => R+6>& Hence, when the diameter is halved the resistance
&<R+6 of the wire is
1

(~r
possible for any finite value of&, because R is finite R oc--=16R
(c) For current from F to C direction
I2 <0 => &>R+6
(d) For current in 2/J from B to A direction Hence, its resistance will become 16 times.
_R-8+2& O
I 1 +I 2-----> Again from eqn. (1),
4+3R

www.puucho.com
Anurag Mishra Electricity and Magnetism with www.puucho.com

ELECTRIC CURRENT 32sl


---- l
l z2 12. (a) v d ,;,,_!_,_; Ifi radius is doubled, A becomes 4 times
Roc- or R oc- or R oc 12
A Al neA
Therefore, on increasing the length the resistance ahl:I 'hence· v d becomes one fourth.
increases. 13. (a) Bo~. R/2 are in parallel, so their equivalent
10. (c) On increasing the temperature of metals, the ~esi~t.an_ce i~ Rf 4.
resistance of metal increases. Therefore, the
temperature coefficient of resistance of metals is '·
positive.
On increasing the temperature of insulators, the I, • ---'----I
.•
resistance decreases. Therefore, temperature
coefficient of resistance of insulators is negative.
11. (d) Resistivity or specific resistance is a material R/2
property. So, it does not change on bending the
14. (b) Both the ~tatements are correct, but independent
insulated wire.
of each other.'
On bending, the cross-sectional area of wire
15. (d) When' battery is supplying power,
changes but drift velocity of electron does not
inside battery positive charge
depend on area of cross-section so it does not
moves opposite to electric field. So I-
change.
work done by electrostatic forces
is negative. Electro_static field

' ...
',,

www.puucho.com

S-ar putea să vă placă și